NCLEX Gastrointestinal Review

¡Supera tus tareas y exámenes ahora con Quizwiz!

What is the #1 complication of pancreatitis? #2? #3?

#1= hemorrhoids (incre pulse), #2= DM (incre BS), and #3= calcium wasting.

Hiatal Hernia Patho

-Hole in the diaphragm -This is when the hole in the diaphragm is too large so the stomach moves up into the thoracic cavity

Pancreatitis Patho

-Scar tissue or gallstones can occlude the pancreatic duct. If the digestive enzymes can't get out and they become activated inside the pancreas, then they eat away. When they get stuck, they activate inside the pancreas and they begin to autodigest the pancreas

Diagnosis of Cirrhosis

-US -CT, MRI -Liver Biopsy

Hemoglobin norms

1 3.2-1 7.3 g/dL (male) 1 1 .7-1 5.5 g/dL (female) 1 2.6-1 7.4 g/dL (male, older adult) 1 1 .7-1 6.1 g/dL (female, older adult)

sucralfate is a gastric protectant and is given

1 hour before meals and at bedtime

Globulin norms

2.3-3.4 g/dL

How far apart should bowel sounds be?

5 to 20 seconds.

What is dehiscence?

A separation of the wound incision

A nurse is assisting with preoperative care for a client who requires an appendectomy. The nurse is aware that the surgery will involve which abdominal quadrant? A) RLQ B) RUQ C) LLQ D) LUQ

A) RLQ The appendix is in the right lower quadrant.

The nurse is to obtain a stool specimen from a client who reported that he is taking iron supplements. The nurse would expect the stool to be which color? a) Red b) Black c) Dark brown d) Green

Black Explanation: Ingestion of iron can cause the stool to turn black. Meat protein causes stool to appear dark brown. Ingestion of large amounts of spinach may turn stool green while ingestion of carrots and beets may cause stool to turn red.

Fetor

Breath smells like ammonia (nail polish remover)

A nurse is teaching an elderly client about good bowel habits. Which statement by the client indicates to the nurse that additional teaching is required? a) "I need to drink 2 to 3 liters of fluids every day." b) "I should exercise four times per week." c) "I need to use laxatives regularly to prevent constipation." d) "I should eat a fiber-rich diet with raw, leafy vegetables, unpeeled fruit, and whole grain bread."

C) "I need to use laxatives regularly to prevent constipation." The client requires more teaching if he states that he'll use laxatives regularly to prevent constipation. The nurse should teach this client to gradually eliminate the use of laxatives because using laxatives to promote regular bowel movements may have the opposite effect. A high-fiber diet, ample amounts of fluids, and regular exercise promote good bowel health.

A nurse is preparing a client for surgery. During preoperative teaching, the client asks where is bile stored. The nurse knows that bile is stored in the: a) Cystic duct b) Duodenum c) Gallbladder d) Common bile duct

C) Gallbladder The gallbladder functions as a storage depot for bile.

Following administration of a dose of metoclopramide (Reglan) to the patient, the nurse determines that the medication has been effective when which of the following is noted? A) Decreased blood pressure B) Absence of muscle tremors C) Relief of nausea and vomiting D) No further episodes of diarrhea

C) Relief of nausea and vomiting. (Metoclopramide is classified as a prokinetic and antiemetic medication. If it is effective, the patient's nausea and vomiting should resolve).

The nurse is assessing a client for constipation. Which of the following is the first review that the nurse should conduct in order to identify the cause of constipation? Choose the correct option. a) Review the client's current medications b) Review the client's alcohol consumption c) Review the client's usual pattern of elimination d) Review the client's activity levels

C) Review the client's usual pattern of elimination Constipation has many possible reasons; assessing the client's usual pattern of elimination is the first step in identifying the cause.

A nurse is teaching a group of middle-aged men about peptic ulcers. When discussing risk factors for peptic ulcers, the nurse should mention: a) alcohol abuse and a history of acute renal failure. b) a history of hemorrhoids and smoking. c) a sedentary lifestyle and smoking. d) alcohol abuse and smoking.

D) Alcochol abuse and smoking The nurse should mention that risk factors for peptic (gastric and duodenal) ulcers include alcohol abuse, smoking, and stress. A sedentary lifestyle and a history of hemorrhoids aren't risk factors for peptic ulcers. Chronic renal failure, not acute renal failure, is associated with duodenal ulcers.

What are medium-length nasoenteric tubes are used for? a) Aspiration b) Emptying c) Decompression d) Feeding

D) Feeding Placement of the tube must be verified prior to any feeding. A gastric sump and nasoenteric tube are used for gastrointestinal decompression. Nasoenteric tubes are used for feeding. Gastric sump tubes are used to decompress the stomach and keep it empty.

Which of the following is the major carbohydrate that tissue cells use as fuel? a) Proteins b) Fats c) Chyme d) Glucose

D) Glucose Glucose is the major carbohydrate that tissue cells use as fuel. Proteins are a source of energy after they are broken down into amino acids and peptides. Chyme stays in the small intestine for 3 to 6 hours, allowing for continued breakdown and absorption of nutrients. Ingested fats become monoglycerides and fatty acids by the process of emulsification.

The nurse is providing care to a client who has had a percutaneous liver biopsy. The nurse would monitor the client for which of the following? a) Intake and output b) Passage of stool c) Return of the gag reflex d) Signs and symptoms of bleeding

D) Signs and symptoms of bleeding A major complication after a liver biopsy is bleeding so it would be important for the nurse to monitor the client for signs and symptoms of bleeding. Return of the gag reflex would be important for the client who had an esophagogastroduodenoscopy to prevent aspiration. Monitoring the passage of stool would be important for a client who had a barium enema or colonoscopy. Monitoring intake and output is a general measure indicated for any client. It is not specific to a liver biopsy.

What drugs are given for peptic ulcer disease?

Carafate (coats ulcer 1 hr b4 eating), Tagament (blocks hydrochloric acid with food), and Antacid-Maalox (Fights acid 1 hr after eating). CTMA (containing too much acid).

Patient had an abdominal peroneal ressection and his stitches have torn. What is your first action?

Cover the wound with sterile gauze wetted with NS.

What must you do immediately for a patient with esophageal varices?

Establish IV access, give pt FFP IV to decrease bleeding. To prevent acute alcohol withdrawal administer chlordiazepoxide HCL (librium).

Who is at risk for cholecystitis?

Fair skin, Fat, 40, Female, and Farty

What are the 3 expected outcomes of TPN?

Fluid replacement (moist membranes), calorie adequacy (albumin 3.5-5), normal electrolyte levels.

What are the two types of peptic ulcer disease (PUD)?

Gastric ulcers and Duodenal ulcers

How should meds be administered via a gastrostomy tube?

Give liquid medication when possible (don't need order to change consistency), dilute with at least 30 ml water, semi fowlers during and 30 mins after, VERIFY TUBE PLACEMENT, if giving more than one drug flush between each with 15-30 cc water, flush when done and clamp. DON'T MIX MEDS WITH FORMULA OR FOOD.

Following an X-ray (with barium contrast) of the patient's upper GI, what nursing measure would you take?

Give the patient a laxative to stimulate peristalsis (barium is constipating).

Asterixis

Hand Flap, Liver flap

Patient has a sliding hernia that is causing reflux. What is a primary s/sx?

Heartburn.

What should a patient with cholecystitis do to prevent complications?

Incentive spirometer to prevent atelectasis.

What is the diet for late liver disease?

Liver is a rock so NO LEGUMES. Diet: decreased protein bc converts to ammonia -> coma

What is a lower GI (barium enema)? Teach?

Look at rectum, colon, and large intestine. Clear liquid diet before test and laxative after test to eliminate barium.

What foods should be avoided with diverticulosis?

No Corn, Kiwi, raisins, cucumbers, and strawberries (seeds). Chicken, fish, or rice=good choices.

You discover the patient's TPN is running at the wrong rate and is 2 hours behind schedule. The nursing action would be?

Notify doctor.

Which of the following is an enzyme secreted by the gastric mucosa? a) Pepsin b) Trypsin c) Bile d) Ptyalin

Pepsin Explanation: Pepsin is secreted by the gastric mucosa. Trypsin is secreted by the pancreas. The salivary glands secrete ptyalin. The liver and gallbladder secrete bile.

What can a ruptured diverticulum lead to?

Peritonitis, sepsis, and hypovolemic shock. Immediate surgery is needed.

Esophageal varices are the most common complication of what?

Portal hypertension

What are S&S of cholecystitis?

Right upper quadrant or epigastric pain (NO MORPHINE), increased pain with deep breath, N&V, abd distention, and jaundice.

What are the signs and symptoms of appendicitis?

Rt lower quadrant pain at McBurney's point, N&V, low grade fever, and diarrhea (several days).

What change occurs in late liver disease? What increases LOC?

Scar tissue replaces normal tissue. Liver Lactulose medication gets rid of urea which converts to ammonia= increased LOC (normal ammonia levels= 15-45). (liver lactulose level of consciousness).

Eating what type of food places you at risk for stomach CA?

Smoked foods

Early ambulation

Soon after the client's abdominal surgery the nurse includes in the plan of care which of the follwing interventions, which is essential for promoting peristalsis? A. Consumption of a high-fiber diet B. Early ambulation C. Restriction of fluid intake D. Administration of large doses of opioids

Who is at risk for dumping syndrome?

Stomach stapling, partial or total gastrectomy.

Crohn's patient is on TPN. What are the nursing responsibilities?

Weigh daily, monitor IV infusion rate hourly, tape all tubing securely, vitals q4hrs, CVC dressing change 1x per week (unless signs of infection, drainage, etc.).

Aluminum hydroxide is an antacid that treats

hyperphosphatemia

Aluminum hydroxide is an antacid that causes

hypophosphatemia

A 61-year-old patient with suspected bowel obstruction has had a nasogastric tube inserted at 4:00 am. The nurse shares in the morning report that the day shift staff should check the tube for patency at which of the following times? A) 7:00 am, 10:00 am, and 1:00 pm B) 8:00 am and 12:00 pm C) 9:00 am and 3:00 pm D) 9:00 am, 12:00 pm, and 3:00 pm

B) 8:00 am and 12:00 pm A nasogastric tube should be checked for patency routinely at 4-hour intervals. Thus if the tube were inserted at 4:00 am, it would be due to be checked at 8:00 am and 12:00 pm.

Patients diagnosed with esophageal varices are at risk for hemorrhagic shock. Which of the following is a sign of potential hypovolemia? a) Bradycardia b) Hypotension c) Polyuria d) Warm moist skin

B) Hypotension Signs of potential hypovolemia include cool, clammy skin, tachycardia, decreased blood pressure, and decreased urine output.

What part of the GI tract begins the digestion of food? a) Stomach b) Mouth c) Duodenum d) Esophagus

B) Mouth Food that contains starch undergoes partial digestion when it mixes with the enzyme salivary amylase, which the salivary glands secrete.

What kind of feeding should be administered to a client who is at the risk of diarrhea due to hypertonic feeding solutions? a) Bolus feeding b) Intermittent feeding c) Cyclic feeding d) Continuous feedings

D) Continuous feedings. Continuous feedings should be administered to a client who is at the risk of diarrhea due to hypertonic feeding solutions.

What should you have your patient do prior to an abdominal assessment?

Empty their bladder and lie supine with pillow under knees.

True or False? Obsruction of the biliary tract is indicated by increased unconjugated (indirect) bilirubin levels in the blood.

False Obsruction of the biliary tract is indicated by increased conjugated (direct) bilirubin levels in the blood.

What might show you that someone's crohn's disease is not under control?

They may have an excoriated buttocks (if not then under control).

fetor hepaticus

fruity, musty breath odor of severe chronic liver disease

NCLEX bowel elimination questions

...

lesterol. High-density lipoproteins (HDLs) are the "good" cholesterol.

...

The nurse caring for a client with hemolytic jaundice anticipates which of the following findings on the laboratory results? 1. Elevated serum indirect bilirubin 2. Decreases serum protein 3. Elevated urine bilirubin 4. Decreased urine pH

1 Hemolytic jaundice is caused by excessive breakdown of red blood cells, and the amount of bilirubin produced exceeds the ability of the liver to conjugate it, so there is an increase in indirect bilirubin. Unconjugated bilirubin is insoluble in water and is not found in the urine Core issue: Knowledge of clinical indicators of hemolytic jaundice.

The client with a duodenal ulcer asks the nurse why an antibiotic is part of the treatment regime. Which information should the nurse include in the response? 1. Antibiotics decrease the likelihood of infection 2. Many duodenal ulcers are caused by the Helicobacter pylori organism 3. Antibiotis are used in an attempt to sterilize the stomach 4. Many people have Clostridium difficile, which can lead to ulcer formation

2 H. pylori infection is a major cause of peptic ulcers. Treatment includes eradicating H. pylori with antibiotics. The other answers are incorrect. Core issue: Knowledge of etiology of peptic ulcers, including duodenal ulcers.

A client is scheduled for a fecal fat exam. In planning client education, the nurse includes that which dietary modification is necessary before the test? 1. Eat a fat-free diet the day before the exam 2. Eat a high-fat meal right before the exam. 3. Eat a diet containing 35 grams of fat for 36 hours before the test. 4. Eat at least 100grams of fat for 3 days before and during the test.

4 It is suggested that adults consume at least 100 grams of fat per day for 3 days prior to test and throughout specimen collection. The other responses provide incorrect information. Core issue: Ability to provide correct information when teaching a client about proper preparation for fecal fat examination.

A client was admitted to the hospital with cholelithiasis the previous day. Which of the following new assessment findings indicates to the nurse that the stone has probably obstructed the common bile duct? 1. Nausea 2. Elevated cholesterol level 3. Right upper quadrant (RUQ) pain 4. Jaundice

4 Nausea and RUQ pain occur in cystic duct disease, but obstruction of the common bile duct results in reflux of bile into the liver, which produces jaundice. alkaline phosphatase increases with biliary obstruction but cholesterol level does not increase. Core issue: Knowledge of clinical indicators of common bile duct obstruction. Think about the pathophysiology of blocked bile drainage and use the process of elimination to make a decision.

The nurse has an order to administer sulfasalazine (Azulfidine) 2 g. The medication is available in 500-mg tablets. How many tablets should the nurse administer? ________________________ tablets.

4 tablets To administer 2 g sulfasalazine (Azulfidine), the nurse will need to administer 4 tablets.

Amoxicillin trihydrate (Amoxil) 300 mg P.O. has been prescribed for a client with an oral infection. The medication is available in a liquid suspension that is available as 250 mg/ 5 mL. How many milliliters would the nurse administer? ________________________ mL.

6 mL. To administer 300 mg P.O., the nurse will need to administer 6 mL. The following formula is used to calculate the correct dosage: 300 mg/ X mL = 250 mg/ 5 mL.

What is evisceration?

A protrusion of the abdominal viscera (dehis before evis)

After assessing a client with peritonitis, the nurse most likely would document the client's bowel sounds as: a) Absent. b) High-pitched. c) Mild. d) Hyperactive.

A) Absent Since lack of bowel motility typically accompanies peritonitis, bowel sounds are absent. Therefore, the nurse will not observe mild, high-pitched, or hyperactive bowel sounds.

Medication treatment for Cirrhosis

-Antacids -Vitamins -Diuretics -Avoid narcotics because the liver can't metabolize when it is sick

Diagnosing UC and Crohn's

-CT -Colonoscopy -Barium Enema

Gastric Ulcers

-Person has a malnourished appearance; laboring person. -Pain is usually half hour to 1 hour after meals -food does not help, but vomiting does -vomit blood

Treatment for Appendicitis

-Surgey: most done via laparoscope unless perforated

Patient admitted for a bowel obstruction and has N/V, abd pain, and cramping. List priority of nursing actions.

1. Assist with ambulation (to try to induce peristalsis) 2. Give Lactated Ringer's IV 3. NG tube 4. TPN

The hydrogen breath test was developed to evaluate which type of absorption? a) Protein b) Vitamin B12 c) Carbohydrate d) Fat

Carbohydrate Explanation: The hydrogen breath test that is used to evaluate carbohydrate absorption is performed if carbohydrate malabsorption is suspected. The hydrogen test does not evaluate fat, protein, or vitamin B12 absorption.

In what order should you listen to the abdominal quadrants?

RLQ then clockwise to RUQ, LUQ, and LLQ.

What occurs in liver disease?

The liver is unable to excrete bile. Blood going trough liver picks up bile bc excess.

asterixis

nonrhythmic coarse tremor, flexing of the wrist and fingers

Signs and Symptoms of Peptic Ulcers

-Burning pain usually on the mid-epigastric area/back -heartburn

Treatment of Bleeding Esophageal Varices (non-medication)

-Replace blood -VS -CVP -Oxygen -Cleansing enema to get rid of blood -Saline lavage to get blood out of the stomach -Sengstaken-Blakemore Tube

True or False? The structure that prevents reflux of stomach contents into the esophagus is the upper esophageal sphincter.

False The structure that prevents reflux of stomach contents into the esophagus is the lower esophageal sphincter.

sclerotherapy

injection of sclerosising agent

Treatment of Hepatic Coma medications

-Lactulose (Lactulax, Duphalac) (this med decreases serum ammonia)

➤ Nutrients must be consumed at least at a minimal level in order to meet the body's physiological needs. This level is called Dietary Reference Intake (DRI).

...

Which class of laxative acts by causing the stool to absorb water and swell? A)Bulk-forming B)Emollient C)Lubricant D)Stimulant

A)Bulk-forming Emollients Lubricate the stool; Lubricants soften the stool, making it easier to pass: and Stimulants promotes peristalsis by irritating the intestinal mucosa or stimulating nerve ending in the intestinal wall

A home care nurse is caring for a client with complaints of epigastric discomfort who is scheduled for a barium swallow. Which statement by the client indicates an understanding of the test? a) "I'll avoid eating or drinking anything 6 to 8 hours before the test." b) "I'll drink full liquids the day before the test." c) "I'll take a laxative to clear my bowels before the test." d) "There is no need for special preparation before the test."

"I'll avoid eating or drinking anything 6 to 8 hours before the test." Explanation: The client demonstrates understanding of a barium swallow when he states that he must refrain from eating or drinking for 6 to 8 hours before the test. No other preparation is needed. Before a lower GI series, the client should eat a low-residue or clear liquid diet for 2 days and take a potent laxative and an oral liquid preparation.

Folacin (Folate, folic acid)

*Functions*: Cellular metabolism, Neurotransmitter synthesis, Cell division, DNA synthesis, Hemoglobin formation *Sources*: Green leafy vegetables, asparagus, liver, yeast, eggs, beans, fruits, enriched cereals *Effects of Deficiency*: Megaloblastic anemia, Neural tube defects *Symptoms of Excess*: Increased seizure activity, hives, respiratory distress, itching ■ rash

GERD

- Backflow of gastric and duodenal contents into esophagus - Heartburn, dyspepsia, hypersalivation - Int: Avoid peppermint, fried or fatty foods, etc; eat low fat high fiber diet; avoid anticholinergics and NSAIDs; use antacids, H2 receptor antagonists, or PPIs

Upper Gastrointestinal Fiberscopy

- Sedation. Endoscope passed down eso to view gastric wall, sphincters, and duodenum - Pre: NPO 6-12 hrs before, local anesthetic spray and Versed, Atropine sulfate may before to relax muscle, pt lt side - Post: NPO until gag reflex returns, bedrest until alert

Bariatric Surgery

- Surgical reduction of gastric capacity - Pt at ^ risk for pulmonary and thromboembolic complications and death - Gastric restrictive- vertical band - Gastric bypass- stapling - Postop: avoid alcohol, high protein foods, and high sugar and fat foods; eat slowly and chew well; progress food types and amts; nutritional supplements; complications such as dehydration

Paracentesis

- Transabdominal removal of fluid - Pre: void before, measure abd girth, wt and VS; Fowlers - Can lead to vasodilation and shock bc of rapid removal of fluid - Post: Measure fluid, dry sterile dressing, hypovolemia and mental status changes, hematuria - Steps: understands, VS and wt, pt void, assist Dr, apply dressing, monitor VS, measure fluid, label fluid, document

Bleeding Esophageal Varices Patho

-High blood pressure in the liver (portal hypertension) forces collateral circulation to form (extra circulation) -This circulation forms in 3 places: stomach, esophagus, and rectum -When you see an alcoholic client that is GI bleeding, it is usually esophageal varices and is usually not a problem until they rupture -High pressure in the liver and all extra circulation. Protruding vessels under pressure start dripping blood to the stomach and then the patient can vomit blood

Post-op ileostomy care

-It's going to drain liquid all the time -avoid foods that are hard to digest; rough foods increase motility -gatorade in the summer -At risk for kidney stones (always a little dehydrated)

Patho of Cirrhosis

-Liver cells are destroyed and are replaced with connective/scar tissue which alters the circulation in the liver. -The BP in the liver goes up and this is called portal hypertension

Treatment of hepatic coma non-medications

-cleaning enemas to get blood out -decrease protein in the diet -monitor serum ammonia

Colonoscopy (notes about it, prep, etc)

-clear liquid diet for 12-24 hours -NPO 6-8 hours prior -Avoid NSAIDS -Laxatives or enemas until clear -Go-Lytely -Sedated for procedure -post op watch for perforation (pain, discomfort), We are going to assume the worst!!

Client teaching for Peptic Ulcer disease

-decrease stress in life -stop smoking -Eat what you can tolerate; avoid temp extremes and extra spicy foods -Avoid caffeine -Need to be followed for 1 year

Which colostomies are you allowed to irrigate?

-descending and sigmoid (semi-formed and formed stools) -Irrigate same time every day after a meal -If there is a cramp during irrigation, stop the irrigation and lower the bag

➤ Nutritional requirements of older adults are similar to those for all adults, with a slight reduction in the need for calories, and the more likely need for nutrient supplementation. Nutritional problems are also similar, but the incidence is higher among older adults.

...

➤ Potential complications associated with enteral feedings include aspiration, infection, diarrhea, and electrolyte imbalance.

...

The client attends two sessions with the dietitian to learn about diet modifications to minimize gastroesophageal reflux. The teaching would be considered successful if the client says that she will decrease her intake of which of the following foods? 1. Fats. 2. High-sodium foods. 3. Carbohydrates. 4. High-calcium foods.

1. Fats are associated with decreased esophageal sphincter tone, which increases reflux. Obesity contributes to the development of hiatal hernia, and a low-fat diet might also aid in weight loss. Carbohydrates and foods high in sodium or calcium do not affect gastroesophageal reflux.

Which of the following would be an expected outcome for a client with peptic ulcer disease? The client will: 1. Demonstrate appropriate use of analgesics to control pain. 2. Explain the rationale for eliminating alcohol from the diet. 3. Verbalize the importance of monitoring hemoglobin and hematocrit every 3 months. 4. Eliminate contact sports from his or her lifestyle.

2. Alcohol is a gastric irritant that should be eliminated from the intake of the client with peptic ulcer disease. Analgesics are not used to control ulcer pain; many analgesics are gastric irritants. The client's hemoglobin and hematocrit typically do not need to be monitored every 3 months, unless gastrointestinal bleeding is suspected. The client can maintain an active lifestyle and does not need to eliminate contact sports as long as they are not stress-inducing.

Which goal for the client's care should take priority during the first days of hospitalization for an exacerbation of ulcerative colitis? 1. Promoting self-care and independence. 2. Managing diarrhea. 3. Maintaining adequate nutrition. 4. Promoting rest and comfort.

2. Diarrhea is the primary symptom in an exacerbation of ulcerative colitis, and decreasing the frequency of stools is the first goal of treatment. The other goals are ongoing and will be best achieved by halting the exacerbation. The client may receive antidiarrheal agents, antispasmodic agents, bulk hydrophilic agents, or anti-inflammatory drugs.

A client with inflammatory bowel disease is receiving total parenteral nutrition (TPN). The basic component of the client's TPN solution is most likely to be: 1. An isotonic dextrose solution. 2. A hypertonic dextrose solution. 3. A hypotonic dextrose solution. 4. A colloidal dextrose solution.

2. The TPN solution is usually a hypertonic dextrose solution. The greater the concentration of dextrose in solution, the greater the tonicity. Hypertonic dextrose solutions are used to meet the body's calorie demands in a volume of fluid that will not overload the cardiovascular system. An isotonic dextrose solution (e.g., 5% dextrose in water) or a hypotonic dextrose solution will not provide enough calories to meet metabolic needs. Colloids are plasma expanders and blood products and are not used in TPN.

A client who is scheduled for an ileostomy has an order for oral neomycin (Mycifradin) to be administered before surgery. The intended outcome of administering oral neomycin before surgery is to: 1. Prevent postoperative bladder infection. 2. Reduce the number of intestinal bacteria. 3. Decrease the potential for postoperative hypostatic pneumonia. 4. Increase the body's immunologic response to the stressors of surgery.

2. The rationale for the administration of oral neomycin is to decrease intestinal bacteria and thereby decrease the potential for peritonitis and wound infection postoperatively. Neomycin will not alter the client's potential for developing a urinary or respiratory infection. Neomycin does not affect the body's immune system.

A client has returned to the medical surgical unit after having surgery to create an ileostomy. Which goal has the highest priority at this time? 1. Providing relief from constipation. 2. Assisting the client with self-care activities. 3. Maintaining fluid and electrolyte balance. 4. Minimizing odor formation.

3. A high-priority outcome after ileostomy surgery is the maintenance of fluid and electrolyte balance. The client will experience continuous liquid to semiliquid stools. The client should be engaged in self-care activities, and minimizing odor formation is important; however, these goals do not take priority over maintaining fluid and electrolyte balance.

A physician orders spironolactone (Aldactone), 50 mg by mouth four times daily, for a client with fluid retention caused by cirrhosis. Which finding indicates that the drug is producing a therapeutic effect? a) Loss of 2.2 lb (1 kg) in 24 hours b) Serum potassium level of 3.5 mEq/L c) Blood pH of 7.25 d) Serum sodium level of 135 mEq/L

A) Loss of 2.2 lb (1 kg) in 24 hours Daily weight measurement is the most accurate indicator of fluid status; a loss of 2.2 lb (1 kg) indicates loss of 1 L of fluid. Because spironolactone is a diuretic, weight loss is the best indicator of its effectiveness. This client's serum potassium and sodium levels are normal. A blood pH of 7.25 indicates acidosis, an adverse reaction to spironolactone.

A nurse is caring for a client with active upper GI bleeding. What is the appropriate diet for this client during the first 24 hours after admission? a) Skim milk b) Nothing by mouth c) Regular diet d) Clear liquids

B) NPO Shock and bleeding must be controlled before oral intake, so the client should receive nothing by mouth. When the bleeding is controlled, the diet is gradually increased, starting with ice chips and then clear liquids. Skim milk shouldn't be given because it increases gastric acid production, which could prolong bleeding. A clear liquid diet is the first diet offered after bleeding and shock are controlled.

Mr. Jay has a fecal impaction. The nurse correctly administers an oil-retention Enema by doing which of the following? A) Administering a large volume solution 500 to 1000 ml B) Mixing milk and molasses and equal parts for an enema C) Instructing the patient to retain the enema for at least 30 seconds D) Administering the enema while the patient is sitting on a toilet

C) Instructing the patient to retain the enema for at least 30 seconds The usual amount of solution administered with a retention Enema is 150 to 200 mL for an adult. The milk and molasses mixture is a carminative enema That helps to expel flats, As does the Harrison flush procedure

A client is evaluated for severe pain in the right upper abdominal quadrant, which is accompanied by nausea and vomiting. The physician diagnoses acute cholecystitis and cholelithiasis. For this client, which nursing diagnosis takes top priority? a) Imbalanced nutrition: Less than body requirements related to biliary inflammation b) Anxiety related to unknown outcome of hospitalization c) Deficient knowledge related to prevention of disease recurrence d) Acute pain related to biliary spasms

D) Acute pain related to biliary spasms The chief symptom of cholecystitis is abdominal pain or biliary colic. Typically, the pain is so severe that the client is restless and changes positions frequently to find relief. Therefore, the nursing diagnosis of Acute pain related to biliary spasms takes highest priority. Until the acute pain is relieved, the client can't learn about prevention, may continue to experience anxiety, and can't address nutritional concerns.

When reviewing the history of a client with pancreatic cancer, the nurse would identify which of the following as a possible risk factor? a) Ingestion of a low-fat diet b) One-time exposure to petrochemicals c) Ingestion of caffeinated coffee d) History of pancreatitis

D) History of pancreatitis Pancreatitis is associated with the development of pancreatic cancer. Other factors that correlate with pancreatic cancer include diabetes mellitus, a high-fat diet, and chronic exposure to carcinogenic substances (i.e., petrochemicals). Although data are inconclusive, a relationship may exist between cigarette smoking and high coffee consumption (especially decaffeinated coffee) and the development of pancreatic carcinoma.

Crohn's disease is a condition of malabsorption caused by which of the following pathophysiological processes? a) Infectious disease b) Gastric resection c) Disaccharidase deficiency d) Inflammation of all layers of intestinal mucosa

D) Inflammation of all layers of intestinal mucosa Crohn's disease is also known as regional enteritis and can occur anywhere along the GI tract, but most commonly at the distal ileum and in the colon. Infectious disease causes problems such as small bowel bacterial overgrowth leading to malabsorption. Disaccharidase deficiency leads to lactose intolerance. Postoperative malabsorption occurs after gastric or intestinal resection.

A client with an esophageal stricture is about to undergo esophageal dilatation. As the bougies are passed down the esophagus, the nurse should instruct the client to do which action to minimize the vomiting urge? a) Hold his breath b) Bear down as if having a bowel movement c) Pant like a dog d) Take long, slow breaths

D) Take long, slow breaths During passage of the bougies used to dilate the esophagus, the client should take long, slow breaths to minimize the vomiting urge. Having the client hold the breath, bear down as if having a bowel movement, or pant like a dog is neither required nor helpful.

Which of the following symptoms characterizes regional enteritis? a) Severe diarrhea b) Diffuse involvement c) Exacerbations and remissions d) Transmural thickening

D) Transmural thickening Transmural thickening is an early pathologic change of Crohn's disease. Later pathology results in deep, penetrating granulomas. Regional enteritis is characterized by regional discontinuous lesions. Severe diarrhea is characteristic of ulcerative colitis while diarrhea in regional enteritis is less severe. Regional enteritis is characterized by a prolonged and variable course while ulcerative colitis is characterized by exacerbations and remissions.

Digestive enzymes contribute to the breakdown of foods for the body's use. Stomach contractions mix the food and gastric secretions and, eventually, the chyme moves to the small intestine. Which nutrient actually delays feeling hungry? a) Fats b) Minerals c) Carbohydrates d) Proteins

Fats Explanation: Fats delay stomach emptying.

Ketones and Insulin

Fats are converted directly into an alternative fuel called *ketones*; ketones raise the acidity of the blood and can lead to acid-base imbalance. Fats are used for fuel in persons with diabetes, whose cells cannot use glucose for energy. Carbohydrates enhance insulin secretion, increase satiety (feeling of fullness and satisfaction), and improve absorption of sodium and excretion of calcium. Insulin is a pancreatic hormone that promotes the movement of glucose into the cells for use.

A client is diagnosed with megaloblastic anemia caused by vitamin B12 deficiency. The physician begins the client on cyanocobalamin (Betalin-12), 100 mcg I.M. daily. Which substance influences vitamin B12 absorption? a) Intrinsic factor b) Histamine c) Liver enzyme d) Hydrochloric acid

Intrinsic factor Explanation: Vitamin B12 absorption depends on intrinsic factor, which is secreted by parietal cells in the stomach. The vitamin binds with intrinsic factor and is absorbed in the ileum. Hydrochloric acid, histamine, and liver enzymes don't influence vitamin B12 absorption.

Constipation

The nurse is instructing the client about the use of opioids for pain relief. Included in the teaching is the fact that opioids may cause: A. Headaches. B. Constipation C. Hypertension D. Muscle weakness

GERD

reflux, incompetent sphincter, pyloric stenosis --antacids, H agonists, proton pump inhibitors --avoid cholinergics

Patients with chronic liver dysfunction have problems with insufficient vitamin intake. Which of the following may occur as a result of vitamin C deficiency? a) Beriberi b) Scurvy c) Night blindness d) Hypoprothrombinemia

B) Scurvy Scurvy may result from a vitamin C deficiency. Night blindness, hypoprothrombinemia, and beriberi do not result from a vitamin C deficiency.

A client is scheduled for an esophagogastroduodenoscopy (EGD) to detect lesions in the gastrointestinal tract. The nurse would observe for which of the following while assessing the client during the procedure? a) Signs of perforation b) Gag reflex c) Client's tolerance for pain and discomfort d) Client's ability to retain the barium

C) Client's tolerance for pain and discomfort The nurse has to assess the client's tolerance for pain and discomfort during the procedure. The nurse should assess the signs of perforation and the gag reflex after the procedure of EGD and not during the procedure. Assessing the client's level for retaining barium is important for a diagnostic test that involves the use of barium. EGD does not involve the use of barium.

Which of the following would be the least important assessment in a patient diagnosed with ascites? a) Measurement of abdominal girth b) Palpation of abdomen for a fluid shift c) Foul-smelling breath d) Weight

C) Foul smelling breath Foul-smelling breath would not be considered an important assessment for this patient. Measurement of abdominal girth, weight, and palpation of the abdomen for a fluid shift are all important assessment parameters for the patient diagnosed with ascites.

What is the diet for both types of peptic ulcer disease?

NO CAFFEINE, NO SPICES, NO NICOTINE, AND NO ALCOHOL.

PUD

ulceration in mucosal wall of esophagus, stomach or intestine from h. pylori tx normal --h agonists, antacids, prostaglandins, anticholinerigs tx bleed (EMERGENCY) --ng lavage, tranfusion, vasopressin, surgery

A client with extreme weakness, pallor, weak peripheral pulses, and disorientation is admitted to the emergency department. His wife reports that he has been "spitting up blood." A Mallory-Weiss tear is suspected, and the nurse begins taking a client history from the client's wife. The question by the nurse that demonstrates her understanding of Mallory-Weiss tearing is: a) "Is your partner being treated for tuberculosis?" b) "Has your partner recently fallen or injured his chest?" c) "Has your partner had recent forceful vomiting?" d) "What spices and condiments does your spouse use on food?"

"Has your partner had recent forceful vomiting?" Explanation: A Mallory-Weiss tear is associated with massive bleeding after a tear occurs in the mucous membrane at the junction of the esophagus and stomach. There is a strong relationship between forceful vomiting, and a Mallory-Weiss tear. The bleeding is coming from the stomach, not from the lungs as would be true in some cases of tuberculosis. A Mallory-Weiss tear doesn't occur from chest injuries or falls and isn't associated with eating spicy foods.

The nurse is assessing a client following laparoscopy. The client states that his stomach looks bloated and asks if this is normal. How will the nurse respond? a) "I am not sure about this. Let me get another nurse." b) "Yes, your abdomen may appear larger as a result of the injection of carbon dioxide for visualization." c) "Do you need to use the restroom? You may have to have a bowel movement." d) "No, this should not occur. I will call the physician right away."

"Yes, your abdomen may appear larger as a result of the injection of carbon dioxide for visualization." Explanation: During a laparoscopic procedure, a pneumoperitoneum is used to inject carbon dioxide into the peritoneal cavity to separate the intestines from the pelvic organs. Gas (carbon dioxide) is insufflated into the peritoneal cavity to create a working space for visualization.

Mechanically altered diet

*Indications* ■ Provides foods that have been mechanically altered in texture to require minimal chewing ■ Used for clients who have difficulty chewing but can tolerate more variety in texture than a liquid diet offers ■ Used for clients who have dental problems, surgery of the head or neck, or dysphagia (requires swallowing evaluation and may require thickened liquids) *Nursing considerations* ■ Degree of texture modification depends on individual need, including pureed, mashed, ground or chopped ■ Foods to be avoided in mechanically altered diets include nuts, dried fruits, raw fruits and vegetables, fried foods, tough, smoked, or salted meats, and foods with coarse textures

Duodenal Ulcers

- Break in mucosa of duodenum - Int: bland diet, tx h pylori, admin H2 receptor antagonists or PPIs

Cholecysteography

- Detects gallstones - Pre: Allergies to iodine, NPO after contrast agent - Post: Dysuria is common, normal diet may be resumed

Laparoscopy

- Direct visualization of organs and structures c/in abd

Percutaneous Transhepatic Cholangiography

- Dye into biliary tree - Pre: NPO, Sedation - Post: VS, peritonitis and septicemia, report pain immediately

Hiatal Hernia

- Esophageal or diaphragmatic hernia - Assess: regurg, dysphagia, feeling of fullness - Int: small freq meals, DONT recline for 1 hr after eating, avoid anticholinergics

Upper GI Notes

-Looks at the esophagus and stomach with dye -NPO past midnight -No smoking, chewing gum, or mints. Remove the nicotine patch too. (smoking increases stomach secretions which will affect the test and smoking increases stomach motility)

What diet should a person with UC or Crohn's have?

-Low fiber (limit GI motility to save fluid) -Avoid cold or hot foods (increase motility) -Avoid smoking

Non-medication treatment for Cirrhosis

-No more alcohol -I&O and daily weights -Rest -Prevent bleeding (bleeding precautions) -Measure abdominal girth for ascites issues -Paracentesis -Monitor jaundice (good skin care) -Diet: decrease protein and low sodium

Medication treatment of Bleeding esophageal varices

-Octreotide (Sandostatin) lowers BP in the liver but can cause vascoconstriction on other arteries, so be careful -Lactulose (Neo-Fradin)-to decrease ammonia

Appendicitis Patho

-Related to a low fiber diet -First thing that we worry about is rupture

Treatment for hiatal hernia

-Small frequent meals -sit up for 1 hour after eating (most symptomatic when they lay down...sit them up) -Elevate HOB -surgery -teach lifestyle changes and healthy diet (treat the cause)

When you are changing the tubing on a central line, how can you avoid getting air in the line?

-clamp it off -valsalva -take a deep breath and hummmmm (increases thoracic pressure to keep the air bubbles out

Causes of hiatal hernia

-congenital abnormalities (diaphragm does not fully form) -trauma -surgery

What are the main jobs of the liver?

-detoxifies the body -helps your blood clot -helps to metabolize (break down) drugs -synthesizes albumin

After abdominal surgery, what position should the patient be in?

-elevate head of bed (flat puts a lot of pressure on the suture)

S/S of Appendicitis

-generalized pain initially (eventually localizes in the right lower quadrant...McBurney's point) -rebound tenderness -n/v -get a good history -anorexia -Abdominal pain first, then n/v

S/S of hiatal hernia

-heartburn -fullness after eating -regurgitation -dysphagia (difficulty swallowing)

paracentesis

-removal of fluid from the peritoneal cavity (ascites) -have patient void (low puncture) -position (sitting up, fowlers) -vital signs (baseline, during, and after)

➤ A food record is the most accurate method of obtaining data about a client's actual food intake.

...

➤ Carbohydrates include simple sugars called monosaccharides and disaccharides and complex carbohydrates called polysaccharides.

...

➤ Government programs provide some assistance for clients who cannot afford to buy food.

...

➤ Interventions for underweight/undernutrition include measures to improve the patient's appetite, assisting with meals, and providing enteral and parenteral nutrition.

...

➤ Laboratory or biochemical indicators of nutritional status include blood glucose and serum protein levels or indices such as albumin, urea, and hemoglobin.

...

➤ Many people must follow a modified diet to assist in managing their illness. In addition, all inpatients must have a diet prescribed by their primary care provider.

...

➤ Reliable guidelines for designing a nutritious diet include the Dietary Reference Intakes (DRIs), Food Guide Pyramids, the USDA Dietary Guidelines, Canada's Food Guide for Healthy Eating, and the Nutrition Facts panel found on packaged foods in the United States.

...

➤ Saturated fats and trans-fats are less healthful lipid choices than mono- and polyunsaturated fats. Saturated fats and trans-fats raise LDL cholesterol levels in the blood.

...

➤ The body mass index (BMI), skinfold measurements, and body part circumferences are estimates of body composition.

...

➤ The essential fatty acids omega-6 and omega-3 help protect against heart disease when they replace transfats in the diet. They are found mainly in vegetable oils, nuts, seeds, and fatty fish.

...

➤ The fat-soluble vitamins are A, D, E, and K. Water-soluble vitamins are C and the B-complex vitamins.

...

➤ The most accurate way to check enteral tube placement is by radiographic (x-ray) verification; but this is too expensive and time consuming for ongoing bedside use.

...

➤ You must follow meticulous sterile technique when administering parenteral nutrition because the solution is delivered into a large central vein; if infection occurs, sepsis is almost immediate.

...

Which of the following dietary measures would be useful in preventing esophageal reflux? 1. Eating small, frequent meals. 2. Increasing fluid intake. 3. Avoiding air swallowing with meals. 4. Adding a bedtime snack to the dietary plan.

1. Esophageal reflux worsens when the stomach is overdistended with food. Therefore, an important measure is to eat small, frequent meals. Fluid intake should be decreased during meals to reduce abdominal distention. Avoiding air swallowing does not prevent esophageal reflux. Food intake in the evening should be strictly limited to reduce the incidence of nighttime reflux, so bedtime snacks are not recommended.

In the case of abdominal tenderness what should be the order of physical assessment?

1st inspect, 2nd auscultate, 3rd percuss, and 4th palpate

The nurse caring for a client with uncomplicated cholelithiasis anticipates that the client's laboratory test results will show an elevation in which of the following? 1. Serum amylase 2. Alkaline phosphatase 3. Mean corpuscular hemoglobin concentration (MCHC) 4. Indirect bilirubin

2 Obstructive biliary disease causes a significant elevation in alkaline phosphatase. Obstruction in the biliary tract causes an elevation in direct bilirubin, not indirect bilirubin. Options 1 and 3 are unrelated Core issue: Use nursing knowledge and process of elimination to make a selection

A client with peptic ulcer disease reports that he has been nauseated most of the day and is now feeling light-headed and dizzy. Based upon these findings, which nursing actions would be most appropriate for the nurse to take? Select all that apply. 1. Administering an antacid hourly until nausea subsides. 2. Monitoring the client's vital signs. 3. Notifying the physician of the client's symptoms. 4. Initiating oxygen therapy. 5. Reassessing the client in an hour.

2, 3. The symptoms of nausea and dizziness in a client with peptic ulcer disease may be indicative of hemorrhage and should not be ignored. The appropriate nursing actions at this time are for the nurse to monitor the client's vital signs and notify the physician of the client's symptoms. To administer an antacid hourly or to wait 1 hour to reassess the client would be inappropriate; prompt intervention is essential in a client who is potentially experiencing a gastrointestinal hemorrhage. The nurse would notify the physician of assessment findings and then initiate oxygen therapy if ordered by the physician.

The nurse is caring for a client who has had a gastroscopy. Which of the following signs and symptoms may indicate that the client is developing a complication related to the procedure? Select all that apply. 1. The client has a sore throat. 2. The client has a temperature of 100 ° F (37.8 ° C). 3. The client appears drowsy following the procedure. 4. The client has epigastric pain. 5. The client experiences hematemesis.

2, 4, 5. Following a gastroscopy, the nurse should monitor the client for complications, which include perforation and the potential for aspiration. An elevated temperature, complaints of epigastric pain, or the vomiting of blood (hematemesis) are all indications of a possible perforation and should be reported promptly. A sore throat is a common occurrence following a gastroscopy. Clients are usually sedated to decrease anxiety and the nurse would anticipate that the client will be drowsy following the procedure.

The nurse is teaching the client how to care for her ileostomy. The client asks the nurse how long she can wear her pouch before changing it. The nurse responds: 1. "The pouch is changed only when it leaks." 2. "You can wear the pouch for about 4 to 7 days." 3. "You should change the pouch every evening before bedtime." 4. "It depends on your activity level and your diet."

2. Unless the pouch leaks, the client can wear her ileostomy pouch for about 4 to 7 days. If leakage occurs, it is important to promptly change the pouch to avoid skin irritation. It is not necessary to change the pouch daily or in the evening. Diet and activity typically do not affect the schedule for changing the pouch.

The physician prescribes sulfasalazine (Azulfidine) for the client with ulcerative colitis to continue taking at home. Which instruction should the nurse give the client about taking this medication? 1. Avoid taking it with food. 2. Take the total dose at bedtime. 3. Take it with a full glass (240 mL) of water. 4. Stop taking it if urine turns orange-yellow.

3. Adequate fluid intake of at least 8 glasses a day prevents crystalluria and stone formation during sulfasalazine therapy. Sulfasalazine can cause gastrointestinal distress and is best taken after meals and in equally divided doses. Sulfasalazine gives alkaline urine an orange-yellow color, but it is not necessary to stop the drug when this occurs.

Three weeks after the client has had an ileostomy, the nurse is following up with instruction about using a skin barrier around the stoma at all times. The client has been applying the skin barrier correctly when: 1. There is no odor from the stoma. 2. The client is adequately hydrated. 3. There is no skin irritation around the stoma. 4. The client only changes the ostomy pouch once a day.

3. Because of high concentrations of digestive enzymes, ileostomy effluent is irritating to skin and can cause excoriation and ulceration. Some form of protection must be used to keep the effluent from contacting the skin. A skin barrier does not decrease odor formation; odor is controlled by diet. The barrier does not affect the client's hydration status, and the nurse can encourage the client to have an adequate daily intake of fluids. Pouches are usually worn for 4 to 7 days before being changed.

A client who has ulcerative colitis has persistent diarrhea. He is thin and has lost 12 lb since the exacerbation of his ulcerative colitis. Which of the following will be most effective in helping the client meet his nutritional needs? 1. Continuous enteral feedings. 2. Following a high-calorie, high-protein diet. 3. Total parenteral nutrition (TPN). 4. Eating six small meals a day.

3. Food will be withheld from the client with severe symptoms of ulcerative colitis to rest the bowel. To maintain the client's nutritional status, the client will be started on TPN. Enteral feedings or dividing the diet into six small meals does not allow the bowel to rest. A high-calorie, high-protein diet will worsen the client's symptoms.

A client who has a history of Crohn's disease is admitted to the hospital with fever, diarrhea, cramping, abdominal pain, and weight loss. The nurse should monitor the client for: 1. Hyperalbuminemia. 2. Thrombocytopenia. 3. Hypokalemia. 4. Hypercalcemia.

3. Hypokalemia is the most expected laboratory finding owing to the diarrhea. Hypoalbuminemia can also occur in Crohn's disease; however, the client's potassium level is of greater importance at this time because a low potassium level can cause cardiac arrest. Anemia is an expected development, but thrombocytopenia is not. Calcium levels are not affected.

What should you expect to drain from a T tube?

300-500 ml of thick blood tinged, bright yellow to dark green bile the first 24 hours. After 4 days the amount will be less than 200 ml/day.

When caring for a client who has cirrhosis, the nurse notices flapping tremors of the wrist and fingers. How should the nurse chart this finding? 1. "Trousseau's sign noted" 2. "Caput medusa noted" 3. "Fetor hepaticus noted" 4. "Asterixis noted"

4 Asterixis, also called liver flap, is the flapping tremor of the hands when the arms are extended. Option 1 reflects hypocalcemia. Option 2 refers to spiderlike abdominal veins that are also commonly found in clients with cirrhosis who have portal hypertension as a complication. Option 3 is a specific odor noted in liver failure Core issue: Knowledge of typical assessment findings in a client with cirrhosis.

A client with a peptic ulcer has been instructed to avoid intense physical activity and stress. Which strategy should the client incorporate into the home care plan? 1. Conduct physical activity in the morning so that he can rest in the afternoon. 2. Have the family agree to perform the necessary yard work at home. 3. Give up jogging and substitute a less demanding hobby. 4. Incorporate periods of physical and mental rest in his daily schedule.

4. It would be most effective for the client to develop a health maintenance plan that incorporates regular periods of physical and mental rest in the daily schedule. Strategies should be identified to deal with the types of physical and mental stressors that the client needs to cope with in the home and work environments. Scheduling physical activity to occur only in the morning would not be restful or practical. There is no need for the client to avoid yard work or jogging if these activities are not stressful.

A client is admitted to the hospital after vomiting bright red blood and is diagnosed with a bleeding duodenal ulcer. The client develops a sudden, sharp pain in the midepigastric region along with a rigid, boardlike abdomen. These clinical manifestations most likely indicate which of the following? 1. An intestinal obstruction has developed. 2. Additional ulcers have developed. 3. The esophagus has become inflamed. 4. The ulcer has perforated.

4. The body reacts to perforation of an ulcer by immobilizing the area as much as possible. This results in boardlike abdominal rigidity, usually with extreme pain. Perforation is a medical emergency requiring immediate surgical intervention because peritonitis develops quickly after perforation. An intestinal obstruction would not cause midepigastric pain. The development of additional ulcers or esophageal inflammation would not cause a rigid, boardlike abdomen.

Which of the following is an accurate statement regarding cancer of the esophagus? a) Chronic irritation of the esophagus is a known risk factor. b) It is three times more common in women in the U.S. than men. c) It is seen more frequently in Caucasian Americans than in African Americans. d) It usually occurs in the fourth decade of life.

A) Chronic irritation of the esophagus is a known risk factor In the United States, cancer of the esophagus has been associated with the ingestion of alcohol and the use of tobacco. In the United States, carcinoma of the esophagus occurs more than three times more often in men as in women. It is seen more frequently in African Americans than in Caucasian Americans. It usually occurs in the fifth decade of life

A client is admitted for suspected GI disease. Assessment data reveal muscle wasting, a decrease in chest and axillary hair, and increased bleeding tendency. The nurse suspects the client has: a) cirrhosis. b) cholelithiasis. c) appendicitis. d) peptic ulcer disease.

A) Cirrhosis Muscle wasting, a decrease in chest and axillary hair, and increased bleeding tendencies are all symptoms of cirrhosis. The client may also have mild fever, edema, abdominal pain, and an enlarged liver. Clients with peptic ulcer disease complain of a dull, gnawing epigastric pain that's relieved by eating. Appendicitis is characterized by a periumbilical pain that moves to the right lower quadrant and rebound tenderness. Cholelithiasis is characterized by severe abdominal pain that presents several hours after a large meal.

After administering a dose of promethazine (Phenergan) to a patient with nausea and vomiting, the nurse explains that which of the following may be experienced as a common temporary adverse effect of the medication? A) Drowsiness B) Reduced hearing C) Sensation of falling D) Photosensitivity

A) Drowsiness (Although being given to this patient as an antiemetic, promethazine also has sedative and amnesic properties. For this reason, the patient is likely to experience drowsiness as an adverse effect of the medication.)

After administering a dose of promethazine (Phenergan) to a patient with nausea and vomiting, the nurse explains that which of the following may be experienced as a common temporary adverse effect of the medication? A) Drowsiness B) Reduced hearing C) Sensation of falling D) Photosensitivity

A) Drowsiness Although being given to this patient as an antiemetic, promethazine also has sedative and amnesic properties. For this reason, the patient is likely to experience drowsiness as an adverse effect of the medication.

A nurse is preparing a client with Crohn's disease for a barium enema. What should the nurse do the day before the test? a) Encourage plenty of fluids. b) Order a high-fiber diet. c) Serve dairy products. d) Serve the client his usual diet.

A) Encourage plenty of fluids The nurse should encourage plenty of fluids because adequate fluid intake is necessary to avoid dehydration that may be caused by the bowel preparation and to prevent fecal impaction after the procedure. The client may be placed on a low-residue diet 1 to 2 days before the procedure to reduce the contents in the GI tract. Fiber intake is limited in a low-residue diet. Because dairy products leave a residue, they aren't allowed the evening before the test. Clear liquids only are allowed the evening before the test.

Two days following a colectomy for an abdominal mass, a patient reports gas pains and abdominal distention. The nurse plans care for the patient based on the knowledge that the symptoms occur as a result of which of the following? A) Impaired peristalsis B) Irritation of the bowel C) Nasogastric suctioning D) Anastomosis site inflammation

A) Impaired peristalsis Until peristalsis returns to normal following anesthesia, the patient may experience slowed gastrointestinal motility leading to gas pains and abdominal distention.

The nurse is caring for a 68-year-old patient admitted with abdominal pain, nausea, and vomiting. The patient has an abdominal mass and a bowel obstruction is suspected. The nurse auscultating the abdomen listens for which of the following types of bowel sounds that is consistent with the patient's clinical picture? A) Low pitched and rumbling above the area of obstruction B) High pitched and hypoactive below the area of obstruction C) Low pitched and hyperactive below the area of obstruction D) High pitched and hyperactive above the area of obstruction

A) Low pitched and rumbling above the area of obstruction Early in intestinal obstruction, the patient's bowel sounds are hyperactive and high pitched, sometimes referred to as "tinkling" above the level of the obstruction. This occurs because peristaltic action increases to "push past" the area of obstruction. As the obstruction becomes complete, bowel sounds decrease and finally become absent.

Which outcome indicates effective client teaching to prevent constipation? a) The client reports engaging in a regular exercise regimen. b) The client limits water intake to three glasses per day. c) The client verbalizes consumption of low-fiber foods. d) The client maintains a sedentary lifestyle.

A) The client reports engaging in a regular exercise regimen. The client having a regular exercise program indicates effective teaching. A regular exercise regimen promotes peristalsis and contributes to regular bowel elimination patterns. A low-fiber diet, a sedentary lifestyle, and limited water intake would predispose the client to constipation.

When caring for a patient with a biliary obstruction, the nurse will anticipate administering which of the following vitamin supplements (select all that apply)? A) Vitamin A B) Vitamin D C) Vitamin E D) Vitamin K E) Vitamin B

A,B,C,D Biliary obstruction prevents bile from entering the small intestine and thus prevents the absorption of fat-soluble vitamins. Vitamins A, D, E, and K are all fat soluble and thus would need to be supplemented in a patient with biliary obstruction.

What should you never do with TPN?

ADD medication to or piggyback with TPN.

A morbidly obese client is prescribed an anorexiant medication. The nurse should prepare to teach the client about which medication? A. Diazepam (Valium) B. Dexfenfluramine (Redux) C. Sibutramine (Meridia) D. Pemoline (Cylert)

ANS: C The nurse should teach the client that sibutramine (Meridia) is an anorexiant medication prescribed for morbidly obese clients. The mechanism of action in the control of appetite appears to occur by inhibiting the neutotransmitters serotonin and norepinephrine. Withdrawal from anorexiants can result in rebound weight gain, lethargy, and depression.

McBurney's point

Appendicitis -Localizes in the lower right quadrant

A patient who is administering a bisacodyl (Dulcolax) suppository asks the nurse how long it will take to work. The nurse replies that the patient will probably need to use the bedpan or commode within which of the following time frames after administration? A) 2-5 Minutes B) 15-60 Minutes C) 2-4 Hours D) 6-8 Hours

B) 15-60 minutes Bisacodyl suppositories usually are effective within 15 to 60 minutes of administration, so the nurse should plan accordingly to assist the patient to use the bedpan or commode.

Patients with chronic liver dysfunction have problems with insufficient vitamin intake. Which of the following may occur as a result of vitamin C deficiency? a) Hypoprothrombinemia b) Scurvy c) Beriberi d) Night blindness

B) Scurvy Scurvy may result from a vitamin C deficiency. Night blindness, hypoprothrombinemia, and beriberi do not result from a vitamin C deficiency.

Cullen's Sign

Bruising around the umbilical area

A patient is scheduled to receive "Colace 100 mg PO." The patient asks to take the medication in liquid form, and the nurse obtains an order for the interchange. Available is a syrup that contains 150 mg/15 ml. How many milliliters does the nurse administer? A) 3 B) 5 C) 10 D) 12

C) 10 mL The concentration of the syrup is 10 mg/ml. Therefore, a 100-mg dose necessitates 10 ml.

A nurse is receiving report from the emergency room regarding a new client being admitted to the medical-surgical unit with a diagnosis of peptic ulcer disease. The nurse expects the age of the client will be between a) 20 and 30 years b) 15 and 25 years c) 40 and 60 years d) 60 and 80 years

C) 40 to 60 years Peptic ulcer disease occurs with the greatest frequency in people 40 to 60 years old. It is relatively uncommon in women of childbearing age, but it has been observed in children and even in infants.

A client is scheduled for magnetic resonance imaging (MRI). During the client teaching, the nurse will discuss which of the following? a) "The examination will take only 15 minutes." b) "You must be NPO for the day before the examination." c) "Do you experience any claustrophobia?" d) "You must remove all jewelry but can wear your wedding ring."

C) Do you experience any claustrophobia? MRI is a noninvasive technique that uses magnetic fields and radio waves to produce images of the area being studied. Clients must be NPO for 6 to 8 hours before the study and remove all jewelry and other metals. The examination takes 60 to 90 minutes and can induce feelings of claustrophobia, because the scanner is close fitting.

Which of the following represents the medication classification of a proton (gastric acid) pump inhibitor? a) Famotidine (Pepcid) b) Metronidazole (Flagyl) c) Omeprazole (Prilosec) d) Sucralfate (Carafate)

C) Omeprazole Omeprazole decreases gastric acid by slowing the hydrogen-potassium adenosine triphosphatase pump on the surface of the parietal cells. Sucralfate is a cytoprotective drug. Famotidine is a histamine-2 receptor antagonist. Metronidazole is an antibiotic, specifically an amebicide.

The patient receiving chemotherapy rings the call bell and reports an onset of nausea. The nurse should prepare a prn dose of which of the following medications? A) Morphine sulfate B) Zolpidem (Ambien) C) Ondansetron (Zofran) D) Dexamethasone (Decadron)

C) Ondansetron (Zofran) Ondansetron is a 5-HT3 receptor antagonist antiemetic that is especially effective in reducing cancer chemotherapy-induced nausea and vomiting.

The patient receiving chemotherapy rings the call bell and reports an onset of nausea. The nurse should prepare a prn dose of which of the following medications? A) Morphine sulfate B) Zolpidem (Ambien) C) Ondansetron (Zofran) D) Dexamethasone (Decadron)

C) Ondansetron (Zofran) (Ondansetron is a 5-HT3 receptor antagonist antiemetic that is especially effective in reducing cancer chemotherapy-induced nausea and vomiting.)

A client presents with complaints of blood in her stools. Upon inspection, the nurse notes streaks of bright red blood visible on the outer surface of formed stool. Which of the following will the nurse further investigate with this client? a) Ingestion of cherry soda b) Ingestion of cocoa c) Presence or history of hemorrhoids d) Recent barium studies

C) Presence or history of hemorrhoids Stool is normally light to dark brown. Blood in the stool can present in various ways and must be investigated. Lower rectal or anal bleeding is suspected if there is streaking of blood on the surface of the stool or blood is noted on toilet tissue.

The nurse is caring for a client with cirrhosis. Which assessment findings indicate that the client has deficient vitamin K absorption caused by this hepatic disease? a) Ascites and orthopnea b) Gynecomastia and testicular atrophy c) Purpura and petechiae d) Dyspnea and fatigue

C) Purpura and petechiae A hepatic disorder, such as cirrhosis, may disrupt the liver's normal use of vitamin K to produce prothrombin (a clotting factor). Consequently, the nurse should monitor the client for signs of bleeding, including purpura and petechiae. Dyspnea and fatigue suggest anemia. Ascites and orthopnea are unrelated to vitamin K absorption. Gynecomastia and testicular atrophy result from decreased estrogen metabolism by the diseased liver.

A group of students are studying for an examination on the gastrointestinal (GI) system and are reviewing the structures of the esophagus and stomach. The students demonstrate understanding of the material when they identify which of the following as the opening between the stomach and duodenum? a) Hypoharyngeal sphincter b) Cardiac sphincter c) Pyloric sphincter d) Ileocecal valve

C) Pyloric Sphincter The pyloric sphincter is the opening between the stomach and duodenum. The cardiac sphincter is the opening between the esophagus and the stomach. The hypopharyngeal sphincter or upper esophageal sphincter prevents food or fluids from re-entering the pharynx. The ileocecal valve is located at the distal end of the small intestine and regulates flow of intestinal contents into the large intestine.

Following bowel resection, a patient has a nasogastric tube to suction, but complains of nausea and abdominal distention. The nurse irrigates the tube prn as ordered, but the irrigating fluid does not return. Which of the following should be the priority action by the nurse? A) Notify the physician. B) Auscultate for bowel sounds. C) Reposition the tube and check for placement. D) Remove the tube and replace it with a new one.

C) Reposition the tube and check for placement The tube may be resting against the stomach wall. The first action by the nurse, since this intestinal surgery (not gastric surgery), is to reposition the tube and check it again for placement.

A patient who has undergone colostomy surgery is experiencing constipation. Which of the following interventions should a nurse consider for such a patient? a) Instruct the patient to keep a record of food intake b) Instruct the patient to avoid prune or apple juice c) Suggest fluid intake of at least 2 L per day d) Assist the patient regarding the correct diet or to minimize food intake

C) Suggest fluid intake of at least 2 L per day For constipation the nurse should suggest a fluid intake of at least 2L per day. The nurse should also offer prune or apple juice because they promote elimination. The nurse should encourage the patient to eat regular meals. Dieting or fasting can decrease stool volume and slow elimination. The nurse should instruct the patient to keep a record of food intake in case of diarrhea because this helps identify specific foods that irritate the GI tract.

An elderly client asks the nurse how to treat chronic constipation. What is the best recommendation the nurse can make? a) Administer a tap-water enema weekly. b) Take a mild laxative such as magnesium citrate when necessary. c) Take a stool softener such as docusate sodium (Colace) daily. d) Administer a phospho-soda (Fleet) enema when necessary.

C) Take a stool softener such as docusate sodium (Colace) daily Stool softeners taken daily promote absorption of liquid into the stool, creating a softer mass. They may be taken on a daily basis without developing a dependence. Dependence is an adverse effect of daily laxative use. Enemas used daily or on a frequent basis can also lead to dependence of the bowel on an external source of stimulation.

A nurse is caring for a client with cirrhosis. The nurse assesses the client at noon and discovers that the client is difficult to arouse and has an elevated serum ammonia level. The nurse should suspect which situation? a) The client didn't take his morning dose of lactulose (Cephulac). b) The client is relaxed and not in pain. c) The client's hepatic function is decreasing. d) The client is avoiding the nurse.

C) The client's hepatic function is decreasing The decreased level of consciousness caused by an increased serum ammonia level indicates hepatic disfunction. If the client didn't take his morning dose of lactulose, he wouldn't have elevated ammonia levels and decreased level of consciousness this soon. These assessment findings don't indicate that the client is relaxed or avoiding the nurse.

Which of the following is the best method for determining nasogastric tube placement? a) Placement of external end of tube under water b) Testing of pH of gastric aspirate c) X-ray d) Observation of gastric aspirate

C) X-ray Radiologic identification of tube placement in the stomach is the most reliable method. Gastric fluid may be grassy green, brown, clear, or odorless while an aspirate from the lungs may be off-white or tan. Hence, checking aspirate is not the best method of determining nasogastric tube placement in the stomach. Gastric pH values are typically lower or more acidic than that of the intestinal or respiratory tract, but not always. Placement of external end of tube under water and watching for air bubbles is not a reliable method for determining nasogastric tube placement in the stomach.

Classification of Body Mass Index Values

Classification BMI (kg/m2) Risk of Comorbidities Underweight* <18.5 Low Normal weight 18.5-24.9 Average Pre-obese 25.0-29.9 Mildly increased Class I obesity 30.0-34.9 Moderate Class II obesity 35.0-39.9 Severe Class III obesity ≥40.0 Very severe

What is the preparation needed prior to a colonoscopy?

Clear the colon of stool with Golytely (visualization so decrease risk of bowel perf), no solid food for 24 hours prior to exam but can have clear liquids, nothing 4 hours prior, if takes aspirin, anticoagulants, or iron he should discontinue 4 to 7 days beforehand, arrange for a ride home.

A bowel training program includes which of the following? A) Using a diet that is low in bulk B) Decreasing fluid intake to 1000 mL C) Administering an enema once a day to stimulate peristalsis D) Allowing ample time for Evacuation

D) Allowing ample time for Evacuation For a bowel training program to be effective, the patient must have ample time for evacuation usually 20 to 30 minutes. Fluid intake is increased to 2500 to 3000 mL, food high in bulk is recommended as part of the program: and a daily enema is not administered in bowel training program. A cathartic Suppository maybe use 30 minutes before the patients usually defecation time to stimulate peristalsis

The nurse is preparing to administer a scheduled dose of docusate sodium (Colace) when the patient complains of an episode of loose stool and does not want to take the medication. Which of the following is the appropriate action by the nurse? A) Write an incident report about this untoward event. B) Attempt to have the family convince the patient to take the ordered dose. C) Withhold the medication at this time and try to administer it later in the day. D) Chart the dose as not given on the medical record and explain in the nursing progress notes.

D) Chart the dose as not given on the medical record and explain in the nursing progress notes. Whenever a patient refuses medication, the dose should be charted as not given. An explanation of the reason should then be documented in the nursing progress notes. In this instance, the refusal indicates good judgment by the patient.

The nurse asks a 68-year-old patient scheduled for colectomy to sign the operative permit as directed in the physician's preoperative orders. The patient states that the physician has not really explained well what is involved in the surgical procedure. Which of the following is the most appropriate action by the nurse? A) Ask family members whether they have discussed the surgical procedure with the physician. B) Have the patient sign the form and state the physician will visit to explain the procedure before surgery. C) Explain the planned surgical procedure as well as possible, and have the patient sign the consent form. D) Delay the patient's signature on the consent and notify the physician about the conversation with the patient.

D) Delay the patient's signature on the consent and notify the physician about the conversation with the patient. The patient should not be asked to sign a consent form unless the procedure has been explained to the satisfaction of the patient. The nurse should notify the physician, who has the responsibility for obtaining consent.

The client is experiencing swallowing difficulties and is now scheduled to receive a gastric feeding. She has the following oral medications prescribed: furosemide (Lasix), digoxin, enteric coated aspirin (Ecotrin), and vitamin E. The nurse withholds a) furosemide b) digoxin c) vitamin E d) enteric coated aspirin

D) Enteric coated aspirin Simple compressed tablets (furosemide, digoxin) may be crushed and dissolved in water. Soft gelatin capsules filled with liquid (vitamin E) may be opened, and the contents squeezed out. Enteric coated tablets (enteric coated aspirin) are not to be crushed and a change in the form of the medications is required.

Which type of jaundice seen in adults is the result of increased destruction of red blood cells? a) Obstructive b) Nonobstructive c) Hepatocellular d) Hemolytic

D) Hemolytic Hemolytic jaundice results because, although the liver is functioning normally, it cannot excrete the bilirubin as quickly as it is formed. Obstructive jaundice is the result of liver disease. Nonobstructive jaundice occurs with hepatitis. Hepatocellular jaundice is the result of liver disease.

Which of the following is a parasympathetic response in the GI tract? a) Blood vessel constriction b) Decreased gastric secretion c) Decreased motility d) Increased peristalsis

D) Increased peristalsis Increased peristalsis is a parasympathetic response in the GI tract. Decreased gastric secretion, blood vessel constriction, and decreased motility are sympathetic responses in the GI tract.

The nurse is preparing to administer a dose of bisacodyl (Dulcolax). In explaining the medication to the patient, the nurse would state that it acts in which of the following ways? A) Increases bulk in the stool B) Lubricates the intestinal tract to soften feces C) Increases fluid retention in the intestinal tract D) Increases peristalsis by stimulating nerves in the colon wall

D) Increases peristalsis by stimulating nerves in the colon wall Bisacodyl is a stimulant laxative that aids in producing a bowel movement by irritating the colon wall and stimulating enteric nerves. It is available in oral and suppository forms.

A client is diagnosed with megaloblastic anemia caused by vitamin B12 deficiency. The physician begins the client on cyanocobalamin (Betalin-12), 100 mcg I.M. daily. Which substance influences vitamin B12 absorption? a) Hydrochloric acid b) Histamine c) Liver enzyme d) Intrinsic factor

D) Intrinsic factor Vitamin B12 absorption depends on intrinsic factor, which is secreted by parietal cells in the stomach. The vitamin binds with intrinsic factor and is absorbed in the ileum. Hydrochloric acid, histamine, and liver enzymes don't influence vitamin B12 absorption.

A nurse is preparing a client for a protcosigmoidoscopy. Identify the quadrant on which this diagnostic test will focus. A) RUQ B) RLQ C) LUQ D) LLQ

D) LLQ The sigmoid colon is in the left lower quadrant. Proctosigmoidoscopy is examination of the rectum and sigmoid colon using a rigid endoscope inserted anally about 10 inches.

A physician has ordered a liver biopsy for a client whose condition is deteriorating. Which of the following places the client at high risk due to her altered liver function during the biopsy? a) Low hemoglobin b) Decreased prothrombin time c) Low sodium level d) Low platelet count

D) Low platelet count Certain blood tests provide information about liver function. Prolonged prothrombin time (PT) and low platelet count place the client at high risk for hemorrhage. The client may receive intravenous (IV) administration of vitamin K or infusions of platelets before liver biopsy to reduce the risk of bleeding.

A patient reports having dry mouth and asks for some liquid to drink. The nurse reasons that this symptom can most likely be attributed to a common adverse effect of which of the following medications? A) Digoxin (Lanoxin) B) Cefotetan (Cefotan) C) Famotidine (Pepcid) D) Promethazine (Phenergan)

D) Promethazine (Phenergan) A common adverse effect of promethazine, an antihistamine antiemetic agent, is dry mouth; another is blurred vision.

A client being treated for pancreatitis faces the risk of atelectasis. Which of the following interventions would be important to implement to minimize this risk? a) Withhold oral feedings for the client. b) Instruct the client to avoid coughing. c) Monitor pulse oximetry every hour. d) Reposition the client every 2 hours.

D) Reposition the client every 2 hours Repositioning the client every 2 hours minimizes the risk of atelectasis in a client who is being treated for pancreatitis. The client should be instructed to cough every 2 hours to reduce atelectasis. Monitoring the pulse oximetry helps show changes in respiratory status and promote early intervention, but it would do little to minimize the risk of atelectasis. Withholding oral feedings limits the reflux of bile and duodenal contents into the pancreatic duct.

A client is admitted with a diagnosis of acute appendicitis. When assessing the abdomen, the nurse would expect to find rebound tenderness at which location? a) Left lower quadrant b) Left upper quadrant c) Right upper quadrant d) Right lower quadrant

D) Right lower quadrant The pain of acute appendicitis localizes in the right lower quadrant (RLQ) at McBurney's point, an area midway between the umbilicus and the right iliac crest. Often, the pain is worse when manual pressure near the region is suddenly released, a condition called rebound tenderness.

After teaching a group of students about the various organs of the upper gastrointestinal tract and possible disorders, the instructor determines that the teaching was successful when the students identify which of the following structures as possibly being affected? a) Large intestine b) Ileum c) Liver d) Stomach

D) Stomach The upper gastrointestinal (GI) tract begins at the mouth and ends at the jejunum. Therefore, the stomach would be a component of the upper GI tract. The lower GI tract begins at the ileum and ends at the anus. The liver is considered an accessory structure.

After teaching a group of students about the various organs of the upper gastrointestinal tract and possible disorders, the instructor determines that the teaching was successful when the students identify which of the following structures as possibly being affected? a) Ileum b) Liver c) Large intestine d) Stomach

D) Stomach The upper gastrointestinal (GI) tract begins at the mouth and ends at the jejunum. Therefore, the stomach would be a component of the upper GI tract. The lower GI tract begins at the ileum and ends at the anus. The liver is considered an accessory structure.

A nurse is providing care for a client recovering from gastric bypass surgery. During assessment, the client exhibits pallor, perspiration, palpitations, headache, and feelings of warmth, dizziness, and drowsiness. The client reports eating 90 minutes ago. The nurse suspects: a) Peritonitis b) A normal reaction to surgery c) Dehiscence of the surgical wound d) Vasomotor symptoms associated with dumping syndrome

D) Vasomotor symptoms associated with dumping syndrome Early manifestations of dumping syndrome occur 15 to 30 minutes after eating. Signs and symptoms include vertigo, tachycardia, syncope, sweating, pallor, palpitations, diarrhea, nausea, and the desire to lie down. Dehiscence of the surgical wound is characterized by pain and a pulling or popping feeling at the surgical site. Peritonitis presents with a rigid, boardlike abdomen, tenderness, and fever. The client's signs and symptoms aren't a normal reaction to surgery.

A client with complaints of right lower quadrant pain is admitted to the emergency department. Blood specimens are drawn and sent to the laboratory. Which laboratory finding should be reported to the physician immediately? a) Hematocrit 42% b) Serum potassium 4.2 mEq/L c) Serum sodium 135 mEq/L d) White blood cell (WBC) count 22.8/mm3

D) White blood cell (WBC) count 22.8/mm3 The nurse should report the elevated WBC count. This finding, which is a sign of infection, indicates that the client's appendix might have ruptured. Hematocrit of 42%, serum potassium of 4.2 mEq/L, and serum sodium of 135 mEq/L are within normal limits. Alterations in these levels don't indicate appendicitis.

If a new bag of TPN were unavailable what should be substituted?

D10W at same rate to prevent hypoglycemia.

Seepage of stool around the impaction

Diarrhea that occurs with a fecal impaction is the result of: A. A clear liquid diet B. Irritation of the intestinal mucosa C. Inability of the client to form a stool D. Seepage of stool around the impaction

Lactose intorlerance

During the nursing assessment the client revels that he has diarrhea and cramping every time he eats ice cream. He attributes this to the cold termperature of the food. However, the nurse begins to suspect the these symptoms might be associated with. A. Food allergy B. Irritable bowel C. Lactose intolerance D. Increased peristalsis

What will be done if FFP doesn't control bleeding from alcohol induced esophageal varices?

Endoscopic band ligation, esophagogastric balloon tamponade, and transjugular intrahepatic portosystemic shunting.

What is characteristic of a duodenal ulcer?

Epigastric pain at night, burning cramping mid-epigastric pain, pain 2-4 hrs after meal (eating decreases pain), Weight gain, N&V.

True or False? A drug that clocks the release of secretions from the stomach's chief cells will decrease gastric acidity.

False A drug that clocks the release of secretions from the stomach's parietal cells will decrease gastric acidity.

What should you do if you see an NA feeding a patient with esophageal varices ice chips?

Intervene because ice chips are sharp and can open hemorrhoids or the varices.

What are causes of a hiatal hernia?

Obesity (increased abd pressure); heavy lifting; pregnancy; old age.

What are the S&S of a ruptured diverticulum?

Pale, diaphoretic, rebound tenderness, muscular rigidity, spasm, hypoactive & high pitched bowel sounds, dizziness, severe LLQ pain and N&V.

What positive value indicates stomach CA?

Positive CEA (cancer, embryonic agent).

What should you worry about if someone has high pitched bowel sounds?

Possible obstruction- may go from high pitched to silent.

normal ammonia process..

Protein breaks down into ammonia, the Liver converts ammonia to urea, the kidneys excrete urea in urine

What are the later clinical manifestations of cirrhosis?

Prune like liver= portal hypertension= ascites & hemorrhoids, esophageal VARICES, and enlarged veins on abdomen, palmar erythema, spider angiomas.

Sengstaken-Blakemore Tube

Purpose of this tube is to hold pressure on the bleeding varices

What should you do if evisceration occurs?

Put on sterile gloves and cover the protruding viscera with a warm sterile NaCl covered dressing. Notify physician. Prepare for IV insertion and surgery. Watch for signs of shock.

A client has recently obtained a set of dentures. Which of the following will the nurse do during a complete physical examination? a) Remove the plates and rinse under hot water to remove bacteria. b) Brush the dentures before examining the oral cavity. c) Remove the plates to visualize the oral cavity. d) Have the client rinse with warm salt water before assessing the oral cavity.

Remove the plates to visualize the oral cavity. Explanation: A complete physical examination includes an assessment of the mouth. It is necessary to remove the dentures to allow good visualization of the entire oral cavity.

What are the S&S of esophageal varices?

Tachycardia, pale mucous membranes, JVD, distended abdomen with ascites, and engorged collateral veins in the abdominal wall, and spider veins on chest wall.

What should be done after exam?

Tell pt mild cramping, abdominal pressure, and flatulence are all normal and usually resolve in 24 hours (gas from injected air). Complications include severe abdominal pain (bowel perforation-surgery required).

What is important to know about traditional cholecystectomy surgery?

The incision is under the ribs, pain=atelectasis risk w/NG/Ttube

Glaucoma Cardiovasular disease Risk for increased intracranial pressure

To prevent the client from performing Vlsalva maneuver, the nurse might request a stool softener for a client with which of the following conditions? (Select all that apply.) A. Glaucoma B. Hypotension C. Cardiovasular disease D. Risk for increased intracranial pressure

True or False? The nurse encourages that patient with chronic constipation to attempt defecation after the first meal of the day because gastrocolic and duodenocolic reflexes increase colon peristalsis at that time.

True

A patient with GERD is given urecholine. What adverse effects would you anticipate?

Urinary frequency, diarrhea, increased saliva, hypotension (this drug is a cholinergic...used to increase gastric emptying).

Patient underwent a gastrectomy. The OR nurse must report to the PACU nurse. What is the best way to report?

Use individualized, printed check list (these nurses are in two different places in the hospital, so face-to-face hand-off won't happen).

What is an upper GI (AKA barium swallow)? Teach?

Used to look at the esophagus and stomach, NPO 8 hours before test, drink barium, take laxative after to eliminate barium.

What are S&S of bowel obstruction?

Vomiting that turns malodorous with fecal smell, abdominal distention, constipation with failure to pass flatus, decreased electrolytes, hypovolemia, increased bowel sounds that are high pitched at first and then go silent.

A cone-tipped irrigator

WHen irrigating a colostomy, the nurse is sure to use which of the follwoing equipment?: A. An enema set B. A cone-tipped irrigator C. A 50 mL irrigation syringe D. A 16-French Foley catheter with a 30 mL balloon

What complication do magnesium based antacids have?

Women (Mag) have DIARRHEA of the mouth. Watch for dehydration, hypokalemia, and hyponatremia.

Is it inappropriate for a patient to ask their spouse to take care of their colostomy?

YES

cholecystitis

acute or chronic inflammation of gall bladder Murphy's sign -- cannot take deep breath when fingers passed below hepatic margine RUQ pain up to scapula 2-4 hrs post fatty foods guarding, mass in RUQ jaundice, dark orange foamy urine steathorrea with clay color tx npo, analgesics, antispasmodics, small low fat low gas meals, surgery (cholecystectomy)

Why do liver people tend to be GI bleeders?

blood is a protein. It goes to the GI tract and breaks down into ammonia

CNS effects w/ Cinetidine (Tagament) occur in elderly; signs?

confusion

Pancrelipase is a pancreatic enzyme used w/ patients w/ pancreatitis as a

digestive aid to reduce fatty stool & improve nutritional status

Misoprostyl (Cytotec) & Sucralfate (Carafate) are

gastric protectants

duodenal ulcer

break in duodenum mucosa from h. pylori tx bland diet, h agonists, avoid NSAIDs, aspirin, alcohol

Stool Specimens

- Consistency, color, and occult blood, urobilirubin, fat, N, parasites

PUD

- Ulceration in mucosal wall of stomach, pylorus, duodenum, or esophagus. - Can be gastric, duodenal, or esophageal

Prealbumin norms

1 2-42 mg/dL (age 6 years-adult)

Serum albumin norms

3.4-4.8 g/dL

What is regional enteritis (CROHN's DISEASE)?

An inflammatory condition with familial tendency, that peaks at age 20-40, and causes diarrhea (worse with emotional upset).

If cullens or turners signs appear what should you do?

Don't expect so call MD

How often should the TPN dressing be changed? Technique?

Every 48 hours and PRN aseptic technique is used.

What is the diet for early liver disease?

Increased protein and vitamin C (LEGUMES)

What is asterixis?

Involuntary jerking movements of the hands.

classes of antacids

alminum, MG, NA, and Ca compounds

Loperamide is an

antidiarrheal med

melena

black, tarry, bloody feces

Antacids are used for

peptic ulcer disease & GERD

gastric protectant: creates a protective barrier against acid & pepsin

sucralfate (Carafate)

Barium Enema

-BE or lower GI -Done if colonoscopy was incomplete

Procedure for Liver Biopsy

-Clotting studies pre-PT and PTT -vital signs pre-procedure -Position client supine with right arm out of the way -Exhale and hold breath to get the lungs and diaphragm out of the way -After the procedure, lie the patient on the right side (and also could roll a towel under them) to prevent bleeding -Take vital signs

Patho of Peptic Ulcers

-Common cause of GI bleed -Can be in the esophagus, stomach, or duodenum -Mainly in males -erosion is present

S/S of UC and Crohn's

-Diarrhea -Rectal bleeding -weight loss -vomiting -cramping -dehydration -blood in stools -anemia -rebound tenderness -fever

Duodenal ulcer

-Executives; well-nourished -Night time pain is common and 2-3 hours after meals -food helps (could be overweight) -blood in stools

S/S of Dumping Syndrome

-Fullness -Palpitations -faintness -weakness -cramping -diarrhea

Diagnose Peptic Ulcers

-Gastroscopy (EGD, endoscopy) -Upper GI

Treatment for pancreatitis (non-meds)

-Goal is to control pain -Keep them empty and dry -Decrease gastric secretions (NPO, NGT to suction, bed rest)...want the stomach empty and dry -Maintain fluid and electrolyte balance -maintain nutritional status...ease into the diet slowly -Give insulin if needed (pancreas is sick, steroids inhibit insulin, TPN is packed with sugar) -daily weights -eliminate alcohol (cause) -Refer to AA if this is the cause

Notes on assisting the MD in inserting a central line

-Have saline available for flush; do not start fluids until positive confirmation of placement (CXR) -Position the patient trendelenburg to distend the veins -Xray is done post-insertion to make sure that it is in the right place and patient does not have a pneumothorax

Nursing Considerations for TPN

-Keep refrigerated; warm for administration; let it sit out for a few minutes prior to hanging -Central line needed -filter needed -DO NOT MIX ANYTHING! Nothing else should go through this line. Just TPN -discontinued gradually to avoid hypoglycemia -daily weights (total nutrition...should not be losing weight) -May have to start taking insulin -Blood glucose monitoring q6h -Check urine for ketones and glucose -Do not mix ahead...mixture changes every day according to electrolytes -Can only be hung 24 hours -Change tubing with each new bag -IV bag may be covered with dark bag to prevent chemical breakdown -Needs to be on a pump -Home TPN-emphasize hand washing

Surgery for Crohn's

-May remove only the affected area -the client may end up with an ileostomy or a colostomy. It just depends on the area affected.

Signs and Symptoms of Hepatic Coma

-Monitor mental changes/motor problems -difficult to wake -asterixis -handwriting changes -reflexes will decrease (ammonia is sedative) -EEG slower -Fetor (breath smells like ammonia) -liver people tend to be GI bleeders

Gastroscopy (EGD, endoscopy)

-NPO pre -Sedated -NPO until gag reflex returns -Watch for signs of perforation (pain, bleeding, or difficulty swallowing)

Signs and Symptoms of Pancreatitis

-Pain (increases with eating because each time you eat, the pancreatic enzymes are released) -Abdominal distension and ascites (losing protein-rich foods like enzymes and blood in the abdomen) leading to ascites -Abdominal mass (swollen pancreas) -Rigid board-like abdomen (guarding or bleeding) peritonitis -Bruising around the umbilical area (Cullen's sign) -Bruising around flank area (Gray Turner's sign) -Fever (inflammation) -N/V-filled with toxins -Jaundiced (liver involved) -Hypotension=internal bleeding or ascites

Medication treatment for pancreatitis

-Pain Medications: PCA narcotics morphine sulfate (Morphine), hydromorphone (Dilaudid), Fentanyl patches (Duragesic) -Steroids for inflammation -Anticholinergics: Benztropine (Cogentin), Diphenoxylate/Atropine (Lonox). To dry up secretions -Pantoprazole (Protonix) proton pump inhibitor -Ranitidine HCI (Zantac), Famotidine (Pepcid) (H2 receptor antagonist) -Antacids

What if air gets in the line when inserting a central line?

-Position the client in trendelenburg and on the left side -When an air embolus is suspected in the heart, the patient may be taken to the cath lab for the removal of air

Treatment for Dumping Syndrome

-Semi-recumbent with meals -lie down after meals -No liquid with meals (drink in between meals) -decrease carbs (they empty first) -Lay on left side to keep food in the stomach (stomach is angled to the left)

Diagnosis for Pancreatitis

-Serum lipase and amylase will increase (should be in small intestine and not in the blood) -WBCs are increased -Blood sugar increases (pancrease makes insulin) -ALT, AST-liver enzymes increase -PT, PTT increase (liver helps us clot) -Serum bilirubin increase -H/H go up or down because you are bleeding, but the blood is very concentrated

Dumping Syndrome Patho

-The stomach empties too quickly and the client experiences many uncomfortable to severe side effects...usually secondary to gastric bypass, gastrectomy, or gall bladder disease

Surgery for UC

-Total Colectomy (ileostomy formed) -Kock's ileostomy or a J Pouch (no external bag) A Kock's pouch has a nipple valve that opens and closes to empty intestines. The J pouch procedure removes the colon and attaches the ileum into the rectum (anastimosis) You can wear normal clothing with a Kock's or J Pouch

Diagnosis of Appendicitis

-WBCs increase -US -CT definitive -Do NOT do enemas because you don't want it to rupture!

Signs and Symptoms of Cirrhosis

-firm, nodular liver -Abdominal pain-liver capsule has stretched -Chronic dyspepsia (GI upset) -Change in bowel habits -ascites -splenomegaly (immune system now sick) -decreased serum albumin (this is why liver people have ascites) -Increased ALT and AST -Anemia as a result of bleeding -Can progress to hepatic encephalopathy/coma, ammonia builds up and acts like a sedative

Hepatic Coma Patho

-when you eat protein, it transforms into ammonia, and the liver converts it to urea. Urea can be excreted through the liver without difficulty -when the liver becomes impaired then it can't make this conversion, so ammonia builds up in the body -Ammonia decreases the LOC (sedative) -Liver is sick so you can't use dietary protein to make albumin because ammonia will build up.

➤ Energy nutrients are carbohydrates, proteins, and fats (lipids). Other important nutrients are water and the micronutrients: fat-soluble vitamins, water0soluble vitamins, and minerals.

...

➤ Factors infl uencing a client's nutritional status include developmental stage, pregnancy and lactation, education, lifestyle choices, vegetarianism, dieting for weight loss, culture, religion, disease processes, functional limitations, economic factors, and modified or special diets.

...

➤ Glucose, a monosaccharide, is the primary source of energy for the brain, as well as for the body during moderate-to-intense physical activity.

...

➤ Lipids, including fats and oils, are classifi ed as glycerides, sterols, and phospholipids. They are essential components of cells, fuel the body at rest and during light activity, aid in absorption of fat-soluble vitamins, and promote a sense of satiety when eaten. Low-density lipoproteins (LDLs) are the "bad" cho-

...

➤ Minerals (e.g., calcium, iron, magnesium) are important for transmitting nerve impulses, regulating fluid balance, strengthening bones, and producing energy.

...

➤ MyPyramid is a food guide that illustrates healthful dietary choices. It stresses six concepts for healthy eating: activity, moderation, personalization, proportionality, variety, and gradual improvement.

...

➤ Nasogastric tubes are inserted to lavage the stomach; collect a specimen of the stomach contents, or prevent nausea, vomiting, and gastric distention. They are also placed to facilitate enteral feeding.

...

➤ Nurses can help support nutrition for patients who have Self-Care Deficits or Impaired Swallowing by assisting patients with meals and providing swallowing therapy.

...

➤ Nutritional imbalance can range from either inadequate or excessive amounts of a single nutrient relative to overall caloric intake.

...

➤ Overweight and obesity are a major problem in the United States, both in children and adults.

...

➤ Proteins are made up of amino acids and are required for cell and tissue growth, maintenance, and repair. They also act as buffers in acid-base balance. Proteins are an energy source.

...

➤ The basal metabolic rate (BMR) is a measure of the energy required by resting tissue to maintain basic function.

...

➤ To check enteral tube placement at the bedside, you should use a combination of techniques: aspirate stomach contents to observe color and measure the pH (normally 1 to 5.5 for stomach contents); the "whoosh test"; serial observations for respiratory distress, residual volume, length of tube extending outside the body; and capnometry. Do not rely on a single technique.

...

➤ Vitamin and mineral supplementation may be appropriate for some people, but they do not replace the need for a balanced diet.

...

➤ When examining an individual's nutritional balance, it is important to screen for nutritional problems. When risk factors are present, thoroughly assess nutritional history, physical findings, anthropometric measurements, imaging techniques, and biochemical values.

...

Creatinine norms

0.5-1 mg/dL (female) 0.6-1 .2 mg/dL (male)

Lower Gastrointestinal Tract Study

- Fluoroscopic and radiographic exam of large intestine (barium enema) - Pre: low fiber diet, clear liquid diet and laxative night before, NPO after midnight - Post: ^ fluid intake, mild laxative, monitor stools

Upper Gastrointestinal Tract Study

- Fluoroscopy after pt drinks barium sulfate (barium swallow) - Pre: NPO after midnight - Post: Laxative, ^ fluids, monitor stools (chalky white)

Signs of Bowel Perforation

- Guarding of abd - ^ fever and chills - Pallor - Abd distention and pain - Restlessness - Tachycardia and tachypnea

Vitamin B12 Deficiency

- Inadequate intake of Vit B12 or lack of absorption - Results in pernicious anemia - Assess: severe pallor, smooth beefy red tongue, wt loss, slight jaundice - Int: ^ Vit B12 intake (citrus fruits, dried beans, green leafy vegs, liver, nuts, organ meats), admin Vit B12 injections (lifelong)

Cholecystitis

- Inflammation of gallbladder - Assess: belching, flatulence, epigastric pain, RUQ pain, guarding, murphys sign (cannot take deep breath), ^ T - Biliary obstruction: jaundice, dark orange and foamy urine, steatorrhea - Int: NPO, NG decompression, small low fat meals

Gastritis

- Inflammation of stomach or gastric mucosa - Acute: anorexia, hiccupping - Chronic: anorexia, belching, heartburn after eating, sour taste in mouth, vit b12 deficiency - Int: monitor tachycardia, hypotension, hematemesis; avoid spicy foods; use antibiotics and antacids

Gastric Analysis

- Nasogastric tube into stomach - Aspirate contents, test pH, appearance, volume - q 15 for 1 hr - Histamine or pentagastrin before - Pre: fasting 8-12 hrs - Post: resume normal activities

Liver Biopsy

- Needle through abd wall to liver to get tissue - Pre: Assess Pt, PTT, platelet; sedative; pt supine or lt lateral - Post: bleeding, peritonitis, bedrest, place pt on rt side c pillow to decr risk of hemorrhage, avoid coughing and straining, avoid lifting 1 wk

The nurse is educating the client with gastroesophageal reflux disease (GERD) about ways to minimize symptoms. Which information in the client's history should the nurse address as an indicator that needs to be changed? 1. Lifting weights for exercise 2. Being a vegetarian 3. Having a body mass index of 23 4. Taking calcium carbonate tablets

1 Lifestyle modifications can minimize symptoms of GERD. Anything that increases intra-abdominal pressure should be avoided, such as lifting weights. Obesity also aggravates symptoms, but a body mass index of 23 is normal. Being a vegetarian does not increase risk, and calcium carbonate tablets often aid in symptom relief. Core issue: Ability to identify risk factors that aggravate GERD.

The nurse should evaluate results of which of the following to plan for safe care? 1. Prothrombin time 2. Urinalysis 3. Serum lipase 4. Serum troponin

1 Many clotting factors are produced in the liver, including fibronigen (factor I), prothrombin (factor II), factor V, serum prothrombin conversion accelerator (factor VII), factor IX, and factor X. The client's ability to form these factors may be impaired with cirrhosis, putting the client at risk for bleeding. The prothrombin time will evaluate blood clotting ability, the others will not. Core issue: The critical word in the questions is "safe". With this in mind, the correct answer is one that could lead to detect a complication of cirrhosis.

Which of the following assessments made by the nurse could indicate the development of portal hypertension in a client with cirrhosis? 1. Hemorrhoids 2. Bleeding gums 3. Muscle wasting 4. Hypothermia

1 Obstruction to portal blood flow causes a rise in portal venous pressure resulting in splenomegaly, ascites, and dilation of collateral venous channels predominantly in the paraumbilical and hemorrhoidal veins, the cardia of the stomach, and extending into the esophagus. Bleeding gums would indicate insufficient vitamin K production in the liver. Muscle wasting commonly accompanies the poor nutritional intake commonly seen in clients with cirrhosis. Hypothermia is an unrelated finding. Core issue: Knowledge of associated findings in a client with portal hypertension

Magnesium

*Functions*: Aids thyroid hormone secretion, maintains normal basal metabolic rate, activates enzymes for carbohydrate and protein metabolism, nerve and muscle function, cardiac function *Sources*: Whole grains, nuts, legumes, green leafy vegetables, lima beans, broccoli, squash, potatoes *Effects of deficiency*: Tremor, spasm, convulsions, weakness, muscle pain, poor cardiac function *Symptoms of excess*: Weakness, nausea, malaise

Calcium

*Functions*: Bone and teeth formation, blood clotting, nerve conduction, muscle contraction, cellular metabolism, heart action *Sources*: Dairy products, sardines, green leafy vegetables, broccoli, whole grains, egg yolks, legumes, nuts, fortified products *Effects of deficiency*: Bone loss, tetany, rickets, osteoporosis *Symptoms of excess*: Kidney stones, constipation, intestinal gas

Potassium

*Functions*: Intracellular fluid control, acid-base balance, nerve transmission, muscle contraction, glycogen formation, protein synthesis, energy metabolism, blood pressure regulation *Sources*: Unprocessed foods, especially fruits, any vegetables, meats, potatoes, avocados, legumes, milk, molasses, shellfish, dates, figs *Effects of deficiency*: Muscle weakness (including weakness of heart and respiratory muscles), weak pulse, fatigue, abdominal distention. (Rarely occurs as a result of inadequate dietary intake. More likely due to losses from prolonged vomiting, diarrhea, or some diuretic drugs.) *Symptoms of excess*: Cardiac dysrhythmias, cardiac arrest, weakness, abdominal cramps, diarrhea, anxiety, paresthesia

Full Liquid Diet

*Indication* ■ May be used as a transition diet after clear liquids following surgery or for clients who have difficulty swallowing or tollerating solid foods *Nursing considerations* ■ A full liquid diet is nutritionally deficient in energy (calories) and many *nutrients* ■ The diet included clear and opaque liquid foods, and those that are liquid at body temperature ■ Foods include all clear liquids and items such as plain ice cream, sherbert, breakfast drinks, milk, pudding and custard, soups that are strained, refined cooked cereals, fruit juices, strained vegetable juices ■ Use of a complete nutritional liquid supplement is often necessary to meet nutrient needs for clietns on a full liquid diet for more than 3 days */!\* Provide nuturitional supplements such as those high in protein, as prescribed for the client on a liquid diet */!\*

Endoscopic Retrograde Cholangiopancreatography

- Examine hepatobiliary system - Pre: NPO, sedation - Post: return of gag reflex, signs of perforation

Fiberoptic Colonoscopy

- Examines lining of large intestine - Pt lt side c knee-chest - Pre: clear liquid diet, NPO after midnight, mild sedative, glucagon may be admin to relax muscle - Post: bedrest, signs of bowel perforation

Dumping Syndrome

- Rapid emptying of gastric contents into small intestine following gastric resection - Assess: 30 mins after meals, palpitations, tachycardia, perspiration, borborygmi (loud gurgles) - Prevent: avoid sugar, salt, and milk; eat high protein high fat and low carb; small meals and avoid fluids c meals; lie down after meals; take antispasmodic

Gastric Ulcers

- Ulceration of mucosal lining in stomach - Predisp: stress, smoking, corticosteroids, NSAIDs, alcohol, fam hx - Gnawing sharp pain occurs 30 to 60 mins after meal, hematemesis is more common - Int: small frequent bland meals, H2 receptor antagonists or PPIs, antacids, anticholinergics, mucosal barrier protectants, prostaglandins - Total gastrectomy- removal of stomach; Vagotomy- division of vagus nerve; Gastric resection- removal of lower half of stomach; Billroth I- remaining section connected to duodenum; Billroth II- remaining section connected to jejunum; Pyloroplasty- enlargement of pylorus - Following gastric surgery DONT irrigate NG tube

The client with diverticular disease is scheduled for a sigmoidoscopy. He suddenly complains of severe abdominal pain. On examination, the nurse notes a rigid abdomen with guarding. What action should the nurse take next? 1. Notify the physician 2. Place the client in a more comfortable position 3. Keep the client distracted until the procedure begins 4. Tell the client that the test will show what is causing his problem

1 Perforation of an obstructed diverticulum can cause abscess formation or generalized peritonitis. The manifestations of peritonitis are abdominal guarding and rigidity and pain.. Sigmoidoscopy is contraindicated in cases of perforation. Because treatment of this complication is beyond the scope of independent nursing practice, the physician must be notified. Core issue: Ability to identify the occurrence of peritonitis as a complication of diverticular disease and determine the appropriate course of action.

The client returning from a colonoscopy has been given a diagnosis of Crohn's disease. The oncoming shift nurse expects to note which of the following manifestations in the client? 1. Steatorrhea 2. Firm, rigid abdomen 3. Constipation 4. Enlarged hemorrhoids

1 Steatorrhea is often present in the client with Crohn's disease. Diarrhea is also key feature, but unlike ulcerative colitis, the loose stool usually does not contain blood and is usually less frequent in number of episodes. Core issue: Knowledge and identification of common symptoms of Crohn's disease.

A client is being evaluated for possible duodenal ulcer. The nurse assess the client for which of the following manifestations that would support this diagnosis? 1. Epigastric pain relieved by food 2. History of chronic aspirin use 3. Distended abdomen 4. Positive fluid wave

1 The pain of a gastric ulcer is dull and aching, occurs after eating, and is not relieved by food as is the pain from a duodenal ulcer. The pancreatic juices that are high in bicarbonate are released with food intake and relieve duodenal ulcer pain when the client eats. Chronic aspirin use is irritating to the stomach. The manifestations in 2,3 are unrelated. Core issue: Expected assessment findings in duodenal ulcer. Recall the effect of pancreatic juices on the duodenal ulcer surface and use the process of elimination to make a selection.

The post-cholecystectomy client asks the nurse when the T-tube will be removed. Which of the following responses by the nurse would be appropriate? 1. "When your stool turns to a normal brown color, the tube can be removed" 2. "The tube will be removed at the same time as your staples" 3. "When the tube stops draining, it will be removed" 4. The tube is usually removed the day after surgery"

1 When T-tube drainage declines and the stools return to a normal brown color, the tube can be clamped 1 to 2 hours before and after meals in preparation for tube removal. If the client tolerates clamping, the tube will then be removed. Core issue: Appropriate timeframe for use of a T-tube following gallbladder surgery

A client with a peptic ulcer reports epigastric pain that frequently awakens her during the night. The nurse should instruct the client to do which activities? Select all that apply. 1. Obtain adequate rest to reduce stimulation. 2. Eat small, frequent meals throughout the day. 3. Take all medications on time as ordered. 4. Sit up for one hour when awakened at night. 5. Stay away from crowded areas.

1, 2, 3, 4. The nurse should encourage the client to reduce stimulation that may enhance gastric secretion. The nurse can also advise the client to utilize health practices that will prevent recurrences of ulcer pain, such as avoiding fatigue and elimination of smoking. Eating small, frequent meals helps to prevent gastric distention if not actively bleeding and decreases distension and release of gastrin. Medications should be administered promptly to maintain optimum levels. After awakening during the night, the client should eat a small snack and return to bed, keeping the head of the bed elevated for an hour after eating. It is not necessary to stay away from crowded areas.

The client who has ulcerative colitis is scheduled for an ileostomy. When the client asks the nurse what to expect related to bowel function and care after surgery, what response should the nurse make? 1. "You will be able to have some control over your bowel movements." 2. "The stoma will require that you wear a collection device all the time." 3. "After the stoma heals, you can irrigate your bowel so you will not have to wear a pouch." 4. "The drainage will gradually become semisolid and formed."

2 A client with an ileostomy has no control over bowel movements and must always wear a collections device. The drainage tends to be liquid but becomes pastelike with intake of specific foods. Core issue: Knowledge of stool characteristics and associated stoma appliance needs following ileostomy

When planning care for a client with ulcerative colitis who is experiencing an exacerbation of symptoms, which client care activities can the nurse appropriately delegate to an unlicensed assistant? Select all that apply. 1. Assessing the client's bowel sounds. 2. Providing skin care following bowel movements. 3. Evaluating the client's response to antidiarrheal medications. 4. Maintaining intake and output records. 5. Obtaining the client's weight.

2, 4, 5. The nurse can delegate the following basic care activities to the unlicensed assistant: providing skin care following bowel movements, maintaining intake and output records, and obtaining the client's weight. Assessing the client's bowel sounds and evaluating the client's response to medication are registered nurse activities that cannot be delegated.

The nurse is developing a plan of care for a client with Crohn's disease who is receiving total parenteral nutrition (TPN). Which of the following interventions should the nurse include? Select all that apply. 1. Monitoring vital signs once a shift. 2. Weighing the client daily. 3. Changing the central venous line dressing daily. 4. Monitoring the I.V. infusion rate hourly. 5. Taping all I.V. tubing connections securely.

2, 4, 5. When caring for a client who is receiving TPN, the nurse should plan to weigh the client daily, monitor the I.V. fluid infusion rate hourly (even when using an I.V. fluid pump), and securely tape all I.V. tubing connections to prevent disconnections. Vital signs should be monitored at least every 4 hours to facilitate early detection of complications. It is recommended that the I.V. dressing be changed once or twice per week or when it becomes soiled, loose, or wet.

A nurse is caring for a client who has just returned from surgery to treat a fractured mandible. Which of the following items should always be available at this client's bedside? Select all that apply. 1. Nasogastric tube. 2. Wire cutters. 3. Oxygen cannula. 4. Suction equipment. 5. Code cart.

2, 4. Following surgery for a fractured mandible, the client's jaws will be wired. The nurse should be prepared to intervene quickly in case the client develops respiratory distress or begins to choke or vomit. Wire cutters or scissors should always be available in case the wires need to be cut in a medical emergency. Suction equipment should be available to help clear the client's airway if necessary. It is not necessary to keep a nasogastric tube or oxygen cannula at the client's bedside. Cardiopulmonary arrest is unlikely, so a code cart is not needed at the bedside.

A client with peptic ulcer disease tells the nurse that he has black stools, which he has not reported to his physician. Based on this information, which nursing diagnosis would be appropriate for this client? 1. Ineffective coping related to fear of diagnosis of chronic illness. 2. Deficient knowledge related to unfamiliarity with significant signs and symptoms. 3. Constipation related to decreased gastric motility. 4. Imbalanced nutrition: Less than body requirements related to gastric bleeding.

2. Black, tarry stools are an important warning sign of bleeding in peptic ulcer disease. Digested blood in the stool causes it to be black. The odor of the stool is very offensive. Clients with peptic ulcer disease should be instructed to report the incidence of black stools promptly to their primary health care provider. The data do not support the other diagnoses.

Which of the following diets would be most appropriate for the client with ulcerative colitis? 1. High-calorie, low-protein. 2. High-protein, low-residue. 3. Low-fat, high-fiber. 4. Low-sodium, high-carbohydrate.

2. Clients with ulcerative colitis should follow a well-balanced high-protein, high-calorie, low-residue diet, avoiding such high-residue foods as whole-wheat grains, nuts, and raw fruits and vegetables. Clients with ulcerative colitis need more protein for tissue healing and should avoid excess roughage. There is no need for clients with ulcerative colitis to follow low-sodium diets.

Which of the following interventions should the nurse include in the client's plan of care to prevent complications associated with TPN administered through a central line? 1. Use a clean technique for all dressing changes. 2. Tape all connections of the system. 3. Encourage bed rest. 4. Cover the insertion site with a moisture-proof dressing.

2. Complications associated with administration of TPN through a central line include infection and air embolism. To prevent these complications, strict aseptic technique is used for all dressing changes, the insertion site is covered with an air-occlusive dressing, and all connections of the system are taped. Ambulation and activities of daily living are encouraged and not limited during the administration of TPN.

A client has been placed on long-term sulfasalazine (Azulfidine) therapy for treatment of his ulcerative colitis. The nurse should encourage the client to eat which of the following foods to help avoid the nutrient deficiencies that may develop as a result of this medication? 1. Citrus fruits. 2. Green, leafy vegetables. 3. Eggs. 4. Milk products.

2. In long-term sulfasalazine therapy, the client may develop folic acid deficiency. The client can take folic acid supplements, but the nurse should also encourage the client to increase the intake of folic acid in his diet. Green, leafy vegetables are a good source of folic acid. Citrus fruits, eggs, and milk products are not good sources of folic acid.

A client with Crohn's disease has concentrated urine, decreased urinary output, dry skin with decreased turgor, hypotension, and weak, thready pulses. The nurse should do which of the following first? 1. Encourage the client to drink at least 1,000 mL per day. 2. Provide parenteral rehydration therapy ordered by the physician. 3. Turn and reposition every 2 hours. 4. Monitor vital signs every shift.

2. Initially, the extracellular fluid (ECF) volume with isotonic I.V. fluids until adequate circulating blood volume and renal perfusion are achieved. Vital signs should be monitored as parenteral and oral rehydration are achieved. Oral fluid intake should be greater than 1,000 mL/ day. Turning and repositioning the client at regular intervals aids in the prevention of skin breakdown, but it is first necessary to rehydrate this client.

The client asks the nurse whether he will need surgery to correct his hiatal hernia. Which reply by the nurse would be most accurate? 1. "Surgery is usually required, although medical treatment is attempted first." 2. "Hiatal hernia symptoms can usually be successfully managed with diet modifications, medications, and lifestyle changes." 3. "Surgery is not performed for this type of hernia." 4. "A minor surgical procedure to reduce the size of the diaphragmatic opening will probably be planned."

2. Most clients can be treated successfully with a combination of diet restrictions, medications, weight control, and lifestyle modifications. Surgery to correct a hiatal hernia, which commonly produces complications, is performed only when medical therapy fails to control the symptoms.

Which of the following lifestyle modifications should the nurse encourage the client with a hiatal hernia to include in activities of daily living? 1. Daily aerobic exercise. 2. Eliminating smoking and alcohol use. 3. Balancing activity and rest. 4. Avoiding high-stress situations.

2. Smoking and alcohol use both reduce esophageal sphincter tone and can result in reflux. They therefore should be avoided by clients with hiatal hernia. Daily aerobic exercise, balancing activity and rest, and avoiding high-stress situations may increase the client's general health and well-being, but they are not directly associated with hiatal hernia.

A client newly diagnosed with ulcerative colitis who has been placed on steroids asks the nurse why steroids are prescribed. The nurse shuld tell the client? 1. "Ulcerative colitis can be cured by the use of steroids." 2. "Steroids are used in severe flare-ups because they can decrease the incidence of bleeding." 3. "Long-term use of steroids will prolong periods of remission." 4.. "The side effects of steroids outweigh their benefits to clients with ulcerative colitis."

2. Steroids are effective in management of the acute symptoms of ulcerative colitis. Steroids do not cure ulcerative colitis, which is a chronic disease. Long-term use is not effective in prolonging the remission and is not advocated. Clients should be assessed carefully for side effects related to steroid therapy, but the benefits of short-term steroid therapy usually outweigh the potential adverse effects.

A client who has been diagnosed with gastroesophageal reflux disease (GERD) complains of heartburn. To decrease the heartburn, the nurse should instruct the client to eliminate which of the following items from the diet? 1. Lean beef. 2. Air-popped popcorn. 3. Hot chocolate. 4. Raw vegetables.

3. With GERD, eating substances that decrease lower esophageal sphincter pressure causes heartburn. A decrease in the lower esophageal sphincter pressure allows gastric contents to reflux into the lower end of the esophagus. Foods that can cause a decrease in esophageal sphincter pressure include fatty foods, chocolate, caffeinated beverages, peppermint, and alcohol. A diet high in protein and low in fat is recommended for clients with GERD. Lean beef, popcorn, and raw vegetables would be acceptable.

Which of the following should the nurse interpret as an indication of a complication after the first few days of TPN therapy? 1. Glycosuria. 2. A 1- to 2-pound weight gain. 3. Decreased appetite. 4. Elevated temperature.

4. An elevated temperature can be an indication of an infection at the insertion site or in the catheter. Vital signs should be taken every 2 to 4 hours after initiation of TPN therapy to detect early signs of complications. Glycosuria is to be expected during the first few days of therapy until the pancreas adjusts by secreting more insulin. A gradual weight gain is to be expected as the client's nutritional status improves. Some clients experience a decreased appetite during TPN therapy.

750 to 1000 mL

A cleansing enema is ordered for a 55 year-old client before intestinal surgery. The maximum amount of fluid used is: A. 150 to 200 mL B. 200 to 400 mL C. 400 to 750 mL D. 750 to 1000 mL

Room-temperature bouillon

A client who recently experience a bout of diarrhea is requesting something to drink. There is an order to force clear liqueids to prevent fluid and electrolyte imbalance. The nurse decides to give the client: A. Ice cream B. A cold fruit pop C. A cup of hot coffee D. Room-temperature bouillon

Irrigate the large lumen with saline

A client with a Salem sump tube begins to drain stomach contents from the blue "pigtail". Which nursing actions would be approprate for the nurse to impelemt at this time? ( Select all that apply.) A. Clamp the blue pigtail B. Attach suction to the blue pigtail. C. Irrigate the large lumen with saline D. Position the blu pigtail at the level of the client's ear.

A wound-ostomy-continence nurse

A nurse traned to care for ostomy clients is: A. A gastrointestinal therapist B. A nurse practitioner C. An ostomy practitioner D. A wound-ostomy-continence nurse

What is a colonoscopy?

A visual exam of the lining of the entire large intestine with a lighted, flexible fiber optic video endoscope.

A patient scheduled to undergo an abdominal ultrasonography is advised to do which of the following? a) Do not undertake any strenuous exercise for 24 hours before the test b) Restrict eating of solid food for 6 to 8 hours before the test. c) Avoid exposure to sunlight for at least 6 to 8 hours before the test d) Do not consume anything sweet for 24 hours before the test

B) Avoid eating of solid food for 6 to 8 hours before the test. For a patient who is scheduled to undergo an abdominal ultrasonography, the patient should restrict herself from solid food for 6 to 8 hours to avoid having images of her test obscured with gas and intestinal contents. Ultrasonography records the reflection of sound waves. Strenuous exercises, the consumption of sweets, and exposure to sunlight do not affect the results of the test in any way.

The nurse is assessing a client with a bleeding gastric ulcer. When examining the client's stool, which of the following characteristics would the nurse be most likely to find? a) Green color and texture b) Black and tarry appearance c) Clay-like quality d) Bright red blood in stool

B) Black and tarry appearance Black and tarry stools (melena) are a sign of bleeding in the upper gastrointestinal (GI) tract. As the blood moves through the GI system, digestive enzymes turn red blood to black. Bright red blood in the stool is a sign of lower GI bleeding. Green color and texture is a distractor for this question. Clay-like stools are a characteristic of biliary disorders

A client with inflammatory bowel disease undergoes an ileostomy. On the first day after surgery, the nurse notes that the client's stoma appears dusky. How should the nurse interpret this finding? a) The ostomy bag should be adjusted. b) Blood supply to the stoma has been interrupted. c) An intestinal obstruction has occurred. d) This is a normal finding 1 day after surgery.

B) Blood supply to the stoma has been interrupted. An ileostomy stoma forms as the ileum is brought through the abdominal wall to the surface skin, creating an artificial opening for waste elimination. The stoma should appear cherry red, indicating adequate arterial perfusion. A dusky stoma suggests decreased perfusion. The nurse should interpret this finding as an indication that the stoma's blood supply is interuppted, which may lead to tissue damage or necrosis. A dusky stoma isn't a normal finding 1 day after surgery. Adjusting the ostomy bag wouldn't affect stoma color, which depends on blood supply to the area. An intestinal obstruction also wouldn't change stoma color.

Which of the follow statements provide accurate information regarding cancer of the colon and rectum? a) There is no hereditary component to colon cancer. b) Cancer of the colon and rectum is the second most common type of internal cancer in the United States. c) Rectal cancer affects more than twice as many people as colon cancer. d) The incidence of colon and rectal cancer decreases with age.

B) Cancer of the colon and rectum is the second most common type of internal cancer in the US Cancer of the colon and rectum is the second most common type of internal cancer in the United States. Colon cancer affects more than twice as many people as does rectal cancer (94,700 for colon, 34,700 for rectum). The incidence increases with age (the incidence is highest in people older than 85). Colon cancer occurrence is higher in people with a family history of colon cancer.

Your patient complains of an excessive flatulence. When reviewing your patients dietary intake, which foods, if eaten regularly, would you identify as possibly responsible? A) Meet B) Cauliflower C) Potatoes D) Ice cream

B) Cauliflower Cauliflower is a gas producing food that relates in flatulence

A client comes into the emergency department with complaints of abdominal pain. Which of the following should the nurse ask first? a) Family history of ruptured appendix b) Characteristics and duration of pain c) Concerns about impending hospital stay d) Medications taken in the last 8 hours

B) Characteristics and duration of pain A focused abdominal assessment begins with a complete history. The nurse must obtain information about abdominal pain. Pain can be a major symptom of gastrointestinal disease. The character, duration, pattern, frequency, location, distribution, and timing of the pain vary but require investigation immediately.

A client is scheduled for several diagnostic tests to evaluate her gastrointestinal function. After teaching the client about these tests, the nurse determines that the client has understood the teaching when she identifies which test as not requiring the use of a contrast medium? a) Computer tomography b) Colonoscopy c) Small bowel series d) Upper GI series

B) Colonoscopy A colonoscopy is a direct visual examination of the entire large intestine. It does not involve the use of a contrast agent. Contrast medium may be used with a small bowel series, computed tomography, and upper GI series.

A client recovering from gastric bypass surgery accidentally removes the nasogastric tube. It is best for the nurse to a) Notify the surgeon about the tube's removal. b) Reinsert the nasogastric tube to the stomach. c) Document the discontinuation of the nasogastric tube. d) Place the nasogastric tube to the level of the esophagus.

A) Notify the surgeon about the tube's removal If the nasogastric tube is removed accidently in a client who has undergone esophageal or gastric surgery, it is usually replaced by the physician. Care is taken to avoid trauma to the suture line. The nurse will not insert the tube to the esophagus or to the stomach in this situation. The nurse needs to do more than just document its removal. The nurse needs to notify the physician who will make a determination of leaving out or inserting a new nasogastric tube.

The nurse is checking the residual content for a client who is receiving intermittent feedings. Which residual content, if obtained, would lead the nurse to delay the feeding? a) 120 mL b) 60 mL c) 30 mL d) 90 mL

A) 120 mL Feedings typically are delayed if the residual content measures more than 100 mL for intermittent feedings or 10% to 20% of the hourly amount of a continuous feeding. Thus a residual content of 120 mL would require the nurse to delay the feeding.

A client with liver and renal failure has severe ascites. On initial shift rounds, his primary nurse finds his indwelling urinary catheter collection bag too full to store more urine. The nurse empties more than 2,000 ml from the collection bag. One hour later, she finds the collection bag full again. The nurse notifies the physician, who suspects that a bladder rupture is allowing the drainage of peritoneal fluid. The physician orders a urinalysis to be obtained immediately. The presence of which substance is considered abnormal? a) Albumin b) Chloride c) Creatinine d) Urobilinogen

A) Albumin Albumin is an abnormal finding in a routine urine specimen. Ascites present in liver failure contain albumin; therefore, if the bladder ruptured, ascites containing albumin would drain from the indwelling urinary catheter because the catheter is no longer contained in the bladder. Creatinine, urobilinogen, and chloride are normally found in urine.

Which diagnostic test is used first to evaluate a client with upper GI bleeding? a) Hemoglobin levels and hematocrit (HCT) b) Endoscopy c) Arteriography d) Upper GI series

A) Hemoglobin levels and hematocrit Hemoglobin and HCT are typically performed first in clients with upper GI bleeding to evaluate the extent of blood loss. Endoscopy is then performed to directly visualize the upper GI tract and locate the source of bleeding. An upper GI series, or barium study, usually isn't the diagnostic method of choice, especially in a client with acute active bleeding who's vomiting and unstable. An upper GI series is also less accurate than endoscopy. Although an upper GI series might confirm the presence of a lesion, it wouldn't necessarily reveal whether the lesion is bleeding. Arteriography is an invasive study associated with life-threatening complications and wouldn't be used for an initial evaluation.

Diet therapy for patients diagnosed with IBS include which of the following? a) High-fiber diet b) Fluids with meals c) Caffeinated products d) Spicy foods

A) High fiber diet A high-fiber diet is prescribed to help control diarrhea and constipation. Foods that are possible irritants such as caffeine, spicy foods, lactose, beans, fried foods, corn, wheat, alcohol should be avoided. Fluids should not be taken with meals because this results in abdominal distention.

Patients diagnosed with esophageal varices are at risk for hemorrhagic shock. Which of the following is a sign of potential hypovolemia? a) Hypotension b) Bradycardia c) Polyuria d) Warm moist skin

A) Hypotension Signs of potential hypovolemia include cool, clammy skin, tachycardia, decreased blood pressure, and decreased urine output.

Patients diagnosed with esophageal varices are at risk for hemorrhagic shock. Which of the following is a sign of potential hypovolemia? a) Hypotension b) Bradycardia c) Warm moist skin d) Polyuria

A) Hypotension Signs of potential hypovolemia include cool, clammy skin, tachycardia, decreased blood pressure, and decreased urine output.

A physician plans to send a client home with supplies to complete a hemoccult test on all stools for 3 days. During the client education, the nurse informs the client to avoid which of the following medications while collecting stool for the test? a) ibuprofen (Advil) b) ciprofloxacin (Cipro XR) c) docusate sodium (Colace) d) acetaminophen (Tylenol)

A) Ibprofen (Advil) Fecal occult blood testing (FOBT) is one of the most commonly performed stool tests. FOBT can be done at the bedside, in the physician's office, or at home. The client is taught to avoid aspirin, red meats, nonsteroidal antiinflammatory agents, and horseradish for 72 hours prior to the examination. Advil is an anti-inflammatory drug and should be avoided with FOBT.

A nursing instructor tells the class that review of oral hygiene is an important component during assessment of the gastrointestinal system. One of the students questions this statement. Which of the following explanations from the nurse educator is most appropriate? a) "Injury to oral mucosa or tooth decay can lead to difficulty in chewing food." b) "Mouth sores are caused by bacteria that can thin the villi of the small intestine." c) "Decaying teeth secrete toxins that decrease the absorption of nutrients." d) "Bad breath will encourage ingestion of fatty foods to mask odor."

A) Injury to the oral mucosa or tooth decay can lead to difficulty in chewing food Poor oral hygiene can result in injury to the oral mucosa, lip, or palate; tooth decay; or loss of teeth. Such problems may lead to disruption in the digestive system. The ability to chew food or even swallow may be hindered.

After 20 seconds of auscultating for bowel sounds on a client recovering from abdominal surgery, the nurse hears nothing. Which of the following should the nurse do based on the assessment findings? a) Listen longer for the sounds. b) Call the physician to report absent bowel sounds. c) Document that the client is constipated. d) Return in 1 hour and listen again to confirm findings.

A) Listen longer for sounds Auscultation is used to determine the character, location, and frequency of bowel sounds. The frequency and character of sounds are usually heard as clicks and gurgles that occur irregularly and range from 5 to 35 per minutes. Normal sounds are heard about every 5 to 20 seconds, whereas hypoactive sounds can be one or two sounds in 2 minutes. Postoperatively, it is common for sounds to be reduced; therefore, the nurse needs to listen at least 3 to 5 minutes to verify absent or no bowel sounds.

A physician has ordered a liver biopsy for a client whose condition is deteriorating. Which of the following places the client at high risk due to her altered liver function during the biopsy? a) Low platelet count b) Low hemoglobin c) Decreased prothrombin time d) Low sodium level

A) Low platelet count Certain blood tests provide information about liver function. Prolonged prothrombin time (PT) and low platelet count place the client at high risk for hemorrhage. The client may receive intravenous (IV) administration of vitamin K or infusions of platelets before liver biopsy to reduce the risk of bleeding.

The nurse is conducting discharge teaching for a patient with metastatic lung cancer who was admitted with a bowel impaction. Which of the following instructions would be most helpful to prevent further episodes of constipation? A) Maintain a high intake of fluid and fiber in the diet. B) Reduce intake of medications causing constipation. C) Eat several small meals per day to maintain bowel motility. D) Sit upright during meals to increase bowel motility by gravity.

A) Maintain a high intake of fluid and fiber int he diet Increased fluid intake and a high-fiber diet reduce the incidence of constipation caused by immobility, medications, and other factors. Fluid and fiber provide bulk that in turn increases peristalsis and bowel motility.

A patient with type 2 diabetes and cirrhosis asks the nurse if it would be okay to take silymarin (milk thistle) to help minimize liver damage. The nurse responds based on knowledge that A) Milk thistle may affect liver enzymes and thus alter drug metabolism. B) Milk thistle is generally safe in recommended doses for up to 10 years. C) There is unclear scientific evidence for the use of milk thistle in treating cirrhosis. D) Milk thistle may elevate the serum glucose levels and is thus contraindicated in diabetes.

A) Milk thistle may affect liver enzymes and thus alter drug metabolism There is good scientific evidence for the use of milk thistle as an antioxidant to protect the liver cells from toxic damage in the treatment of cirrhosis. It is noted to be safe for up to 6 years, not 10 years, and it may lower, not elevate, blood glucose levels. It does affect liver enzymes and thus could alter drug metabolism. Therefore patients will need to be monitored for drug interactions.

Which of the following medications used for the treatment of obesity prevents the reuptake of serotonin and norepinephrine? a) Sibutramine hydrochloride (Meridia) b) Orlistat (Xenical) c) Bupropion hydrochloride (Wellbutrin) d) Fluoxetine hydrochloride (Prozac)

A) Sibutramine hydrochloride (Meridia) Sibutramine hydrochloride (Meridia) inhibits the reuptake of serotonin and norepinephrine. Meridia decreases appetite. Orlistat (Xenical) prevents the absorption of triglycerides. Side effects of Xenical may include increased bowel movements, gas with oily discharge, decreased food absorption, decreased bile flow, and decreased absorption of some vitamins. Bupropion hydrochloride (Wellbutrin) is an antidepressant medication. Fluoxetine hydrochloride (Prozac) has not been approved by the FDA for use in the treatment of obesity.

Which of the following digestive enzymes aids in the digesting of starch? a) Bile b) Trypsin c) Amylase d) Lipase

Amylase Explanation: Digestive enzymes secreted by the pancreas include trypsin, which aids in digesting protein; amylase, which aids in digesting starch; and lipase, which aids in digesting fats. Bile is secreted by the liver and is not considered a digestive enzyme.

A nurse assesses the abdomen of a newly admitted client. Which finding would necessitate further investigation? a) Flat appearance below the umbilicus b) Asymmetrical upper quadrants c) Striae of lateral abdomen d) Rounded contour

Asymmetrical upper quadrants Explanation: The client lies supine with knees flexed for the abdominal assessment. Upon inspection the nurse notes any skin changes, nodules, lesions, inflammation, or striae. Lesions are of particular importance and require further investigation, as do irregular contours or asymmetry of the abdomen.

The results of a pnt's recent endoscopy indicate the presence of peptic ulcer disease (PUD). Which of the following teaching points should the nurse provide to the pt in light of his new diagnosis? A) "You'll need to drink at least two to three glasses of milk daily." B) "It would likely be beneficial for you to eliminate drinking alcohol." C) "Many people find that a minced or pureed diet eases their sxs of PUD." D) "Your medications should allow you to maintain your present diet while minimizing symptoms."

B) "It would likely be beneficial for you to eliminate drinking alcohol." Although there is no specific recommended dietary modification for PUD, most patients find it necessary to make some sort of dietary modifications to minimize symptoms. Milk may exacerbate PUD and alcohol is best avoided because it can delay healing.

A client is evaluated for severe pain in the right upper abdominal quadrant, which is accompanied by nausea and vomiting. The physician diagnoses acute cholecystitis and cholelithiasis. For this client, which nursing diagnosis takes top priority? a) Anxiety related to unknown outcome of hospitalization b) Acute pain related to biliary spasms c) Imbalanced nutrition: Less than body requirements related to biliary inflammation d) Deficient knowledge related to prevention of disease recurrence

B) Acute pain related to biliary spams The chief symptom of cholecystitis is abdominal pain or biliary colic. Typically, the pain is so severe that the client is restless and changes positions frequently to find relief. Therefore, the nursing diagnosis of Acute pain related to biliary spasms takes highest priority. Until the acute pain is relieved, the client can't learn about prevention, may continue to experience anxiety, and can't address nutritional concerns.

A client with a history of alcohol abuse comes to the emergency department and complains of abdominal pain. Laboratory studies help confirm a diagnosis of acute pancreatitis. The client's vital signs are stable, but the client's pain is worsening and radiating to his back. Which intervention takes priority for this client? a) Providing mouth care b) Administering morphine I.V. as ordered c) Placing the client in a semi-Fowler's position d) Maintaining nothing-by-mouth (NPO) status

B) Administering morphine IV as ordered The nurse should address the client's pain issues first by administering morphine I.V. as ordered. Placing the client in a Semi-Fowler's position, maintaining NPO status, and providing mouth care don't take priority over addressing the client's pain issues.

A colectomy is scheduled for a 68-year-old woman with an abdominal mass, possible bowel obstruction, and a history of rectal polyps. The nurse should plan to include which of the following prescribed measures in the preoperative preparation of this patient? A) Instruction on irrigating a colostomy B) Administration of a cleansing enema C) A high-fiber diet the day before surgery D) Administration of IV antibiotics for bowel preparation

B) Administration of a cleansing enema Preoperative preparation for bowel surgery typically includes bowel cleansing with antibiotics, such as oral neomycin and cleansing enemas, including Fleet enemas.

If the patient was instructed to avoid foods that may have a laxative effect, the nurse would advise the patient to avoid which of the following foods? A) Chinese B) Alcohol C) Eggs D) Pasta

B) Alcohol All the foods listed as such alcohol have a constipating effect

A nonresponsive client has a nasogastric tube to low intermittent suction due to gastrointestinal bleeding. It is most important for the nurse to a) Change the nasal tape every 2 to 3 days. b) Auscultate lung sounds every 4 hours. c) Inspect the nose daily for skin irritation. d) Apply water-based lubricant to the nares daily.

B) Auscultate lung sounds every 4 hours Pulmonary complications may occur as a result of nasogastric intubation. It is a high priority according to Maslow's hierarchy of needs and takes a higher priority over assessing the nose, changing nasal tape, or applying a water-based lubricant.

A client with a peptic ulcer is about to begin a therapeutic regimen that includes a bland diet, antacids, and famotidine (Pepcid). Before the client is discharged, the nurse should provide which instruction? a) "Eat three balanced meals every day." b) "Avoid aspirin and products that contain aspirin." c) "Stop taking the drugs when your symptoms subside." d) "Increase your intake of fluids containing caffeine."

B) Avoid aspirin and products that contain aspirin The nurse should instruct the client to avoid aspirin because it's a gastric irritant and should not be taken by clients with peptic ulcer to prevent further erosion of the stomach lining. The client should eat small, frequent meals rather than three large ones. Antacids and ranitidine prevent acid accumulation in the stomach; they should be taken even after symptoms subside. Caffeine should be avoided because it increases acid production in the stomach.

To prevent gastroesophageal reflux in a client with hiatal hernia, the nurse should provide which discharge instruction? a) "Lie down after meals to promote digestion." b) "Avoid coffee and alcoholic beverages." c) "Limit fluid intake with meals." d) "Take antacids with meals."

B) Avoid coffee and alcoholic beverages To prevent reflux of stomach acid into the esophagus, the nurse should advise the client to avoid foods and beverages that increase stomach acid, such as coffee and alcohol. The nurse also should teach the client to avoid lying down after meals, which can aggravate reflux, and to take antacids after eating. The client need not limit fluid intake with meals as long as the fluids aren't gastric irritants.

A nurse is caring for a client who underwent a subtotal gastrectomy. To manage dumping syndrome, the nurse should advise the client to: a) restrict fluid intake to 1 qt (1,000 ml)/day. b) drink liquids only between meals. c) don't drink liquids 2 hours before meals. d) drink liquids only with meals.

B) Drink liquids only between meals. A client who experiences dumping syndrome after a subtotal gastrectomy should be advised to ingest liquids between meals rather than with meals. Taking fluids between meals allows for adequate hydration, reduces the amount of bulk ingested with meals, and aids in the prevention of rapid gastric emptying. There is no need to restrict the amount of fluids, just the time when the client drinks fluids. Drinking liquids with meals increases the risk of dumping syndrome by increasing the amount of bulk and stimulating rapid gastric emptying. Small amounts of water are allowable before meals.

The nurse is caring for a patient treated with intravenous fluid therapy for severe vomiting. As the patient recovers and begins to tolerate oral intake, the nurse understands that which of the following food choices would be most appropriate? A) Ice tea B) Dry toast C) Warm broth D) Plain hamburger

B) Dry toast Dry toast or crackers may alleviate the feeling of nausea and prevent further vomiting. Extremely hot or cold liquids and fatty foods are generally not well tolerated.

The nurse is caring for a patient treated with intravenous fluid therapy for severe vomiting. As the pt recovers and begins to tolerate oral intake, the N understands that which of the following food choices would be most appropriate? A) Ice tea B) Dry toast C) Warm broth D) Plain hamburger

B) Dry toast (Dry toast or crackers may alleviate the feeling of nausea and prevent further vomiting. Extremely hot or cold liquids and fatty foods are generally not well tolerated.)

A nurse is preparing a client with Crohn's disease for a barium enema. What should the nurse do the day before the test? a) Serve dairy products. b) Encourage plenty of fluids. c) Serve the client his usual diet. d) Order a high-fiber diet.

B) Encourage plenty of fluids The nurse should encourage plenty of fluids because adequate fluid intake is necessary to avoid dehydration that may be caused by the bowel preparation and to prevent fecal impaction after the procedure. The client may be placed on a low-residue diet 1 to 2 days before the procedure to reduce the contents in the GI tract. Fiber intake is limited in a low-residue diet. Because dairy products leave a residue, they aren't allowed the evening before the test. Clear liquids only are allowed the evening before the test.

Which of the following medications requires the nurse to contact the pharmacist in consultation when the patient receives all oral medications by feeding tube? a) Buccal or sublingual tablets b) Enteric-coated tablets c) Soft gelatin capsules filled with liquid d) Simple compressed tablets

B) Enteric-coated tablets Enteric-coated tablets are meant to be digested in the intestinal tract and may be destroyed by stomach acids. A change in the form of medication is necessary for patients with tube feedings. Simple compressed tablets may be crushed and dissolved in water for patients receiving oral medications by feeding tube. Buccal or sublingual tablets are absorbed by mucous membranes and may be given as intended to the patient undergoing tube feedings. The nurse may make an opening in the capsule and squeeze out contents for administration by feeding tube.

A nurse is applying an ostomy appliance to the ileostomy of a client with ulcerative colitis. Which action is appropriate? a) Maintaining wrinkles in the faceplate so it doesn't irritate the skin b) Gently washing the area surrounding the stoma using a facecloth and mild soap c) Scrubbing fecal material from the skin surrounding the stoma d) Cutting the faceplate opening no more than 2? larger than the stoma

B) Gently washing the area surrounding the stoma using a facecloth and mild soap For a client with an ostomy, maintaining skin integrity is a priority. The nurse should gently wash the area surrounding the stoma using a facecloth and mild soap. Scrubbing the area around the stoma can damage the skin and cause bleeding. The faceplate opening should be no more than 1/8? to 1/6? larger than the stoma. This size protects the skin from exposure to irritating fecal material. The nurse can create an adequate seal and prevent leakage of fecal material from under the faceplate by applying a thin layer of skin barrier and smoothing out wrinkles in the faceplate. Eliminating wrinkles in the faceplate also protects the skin surrounding the stoma from pressure.

3.The pt who is admitted with a diagnosis of diverticulitis and a history of irritable bowel disease and gastroesophageal reflux disease (GERD) has received a dose of Mylanta 30 ml PO. The nurse would evaluate its effectiveness by questioning the patient as to whether which of the following sxs has been resolved? A) Diarrhea B) Heartburn C) Constipation D) Lower abdominal pain

B) Heartburn (Mylanta is an antacid that contains both aluminum and magnesium. It is indicated for the relief of GI discomfort, such as with heartburn associated with GERD.)

The patient who is admitted with a diagnosis of diverticulitis and a history of irritable bowel disease and gastroesophageal reflux disease (GERD) has received a dose of Mylanta 30 ml PO. The nurse would evaluate its effectiveness by questioning the patient as to whether which of the following symptoms has been resolved? A) Diarrhea B) Heartburn C) Constipation D) Lower abdominal pain

B) Heartburn Mylanta is an antacid that contains both aluminum and magnesium. It is indicated for the relief of GI discomfort, such as with heartburn associated with GERD.

The nurse is planning care for a 68-year-old patient with an abdominal mass and suspected bowel obstruction. Which of the following factors in the patient's history increases the patient's risk for colorectal cancer? A) Osteoarthritis B) History of rectal polyps C) History of lactose intolerance D) Use of herbs as dietary supplements

B) History of rectal polyps A history of rectal polyps places this patient at risk for colorectal cancer. This tissue can degenerate over time and become malignant. The other factors identified do not pose additional risk to the patient.

A client presented with gastrointestinal bleeding 2 days ago and continues to have problems. The physician has ordered a visualization of the small intestine via a capsule endoscopy. Which of the following will the nurse include in the client education about this procedure? a) "An x-ray machine will use a capsule ray to follow your intestinal tract." b) "You will need to swallow a capsule." c) "The physician will use a scope called a capsule to view your intestine." d) "A capsule will be inserted into your rectum."

B) You will need to swallow a capsule A capsule endoscopy allows for noninvasive visualization of the small intestinal mucosa. The technique consists of the client swallowing a capsule that is embedded with a wireless miniature camera, which is propelled through the intestine by peristalsis. The capsule passes from the rectum in 1 to 2 days.

What is heartburn/GERD? Prevention?

Backflow of stomach contents into the esophagus. Avoid smoking/alcohol, exercise, decrease intake of fats and spices, do lie down or bend over shortly after eating (eat 3 hrs before bed), avoid restrictive clothing, eat frequent small meals, chew food well, reduce acidic food intake, and cut down on caffeine.

Why can't liver problem people eat too much protein?

Because the breakdown of protein leads to ammonia production

PUD surgeries

Billroth gastric resection (remove part stomach) gastrectomy (remove stomach) vagotomy (divide vagus to eliminate vagal impulse) pylorplasty (enlarge pylorus) postop watch dumping syndrome

If your liver is sick, what is your number 1 concern?

Bleeding

Gray Turner's sign

Bruising around the flank area

A 24-year-old athlete is admitted to the trauma unit following a motor-vehicle collision. The client is comatose and has developed ascites as a result of the accident. You are explaining the client's condition to his parents. In your education, what do you indicate is the primary function of the small intestine? a) Digest proteins b) Digest fats c) Absorb nutrients d) Absorb water

C) Absorb nutrients The primary function of the small intestine is to absorb nutrients from the chyme.

A 54-year-old patient admitted with diabetes mellitus, malnutrition, osteomyelitis, and alcohol abuse has a serum amylase level of 280 U/L and a serum lipase level of 310 U/L. To which of the following diagnoses does the nurse attribute these findings? A) Malnutrition B) Osteomyelitis C) Alcohol abuse D) Diabetes mellitus

C) Alcohol Abuse The patient with alcohol abuse could develop pancreatitis as a complication, which would increase the serum amylase (normal 30-122 U/L) and serum lipase (normal 31-186 U/L) levels as shown.

A longitudinal tear or ulceration in the lining of the anal canal is termed a (an) a) anorectal abscess. b) anal fistula. c) anal fissure. d) hemorrhoid.

C) Anal fissure Fissures are usually caused by the trauma of passing a large, firm stool or from persistent tightening of the anal canal secondary to stress or anxiety (leading to constipation). An anorectal abscess is an infection in the pararectal spaces. An anal fistula is a tiny, tubular, fibrous tract that extends into the anal canal from an opening located beside the anus. A hemorrhoid is a dilated portion of vein in the anal canal.

A longitudinal tear or ulceration in the lining of the anal canal is termed a (an) a) hemorrhoid. b) anorectal abscess. c) anal fissure. d) anal fistula.

C) Anal fissure Fissures are usually caused by the trauma of passing a large, firm stool or from persistent tightening of the anal canal secondary to stress or anxiety (leading to constipation). An anorectal abscess is an infection in the pararectal spaces. An anal fistula is a tiny, tubular, fibrous tract that extends into the anal canal from an opening located beside the anus. A hemorrhoid is a dilated portion of vein in the anal canal.

The nurse is preparing to measure the client's abdominal girth as part of the physical examination. At which location would the nurse most likely measure? a) At the lower border of the liver b) In the right upper quadrant c) At the umbilicus d) Just below the last rib

C) At the umbilicus Measurement of abdominal girth is done at the widest point, which is usually the umbilicus. The right upper quadrant, lower border of the liver, or just below the last rib would be inappropriate sites for abdominal girth measurement.

A client is prescribed tetracycline to treat peptic ulcer disease. Which of the following instructions would the nurse give the client? a) "Take the medication with milk." b) "Do not drive when taking this medication." c) "Be sure to wear sunscreen while taking this medicine." d) "Expect a metallic taste when taking this medicine, which is normal."

C) Be sure to wear sunscreen while taking this medicine Tetracycline may cause a photosensitivity reaction in clients. The nurse should caution the client to use sunscreen when taking this drug. Dairy products can reduce the effectiveness of tetracycline, so the nurse should not advise him or her to take the medication with milk. A metallic taste accompanies administration of metronidazole (Flagyl). Administration of tetracycline does not necessitate driving restrictions.

The nurse is to obtain a stool specimen from a client who reported that he is taking iron supplements. The nurse would expect the stool to be which color? a) Dark brown b) Red c) Black d) Green

C) Black Ingestion of iron can cause the stool to turn black. Meat protein causes stool to appear dark brown. Ingestion of large amounts of spinach may turn stool green while ingestion of carrots and beets may cause stool to turn red.

Which of the following terms is used to refer to intestinal rumbling? a) Diverticulitis b) Tenesmus c) Borborygmus d) Azotorrhea

C) Borborygmus Borborygmus is the intestinal rumbling that accompanies diarrhea. Tenesmus is the term used to refer to ineffectual straining at stool. Azotorrhea is the term used to refer to excess of nitrogenous matter in the feces or urine. Diverticulitis refers to inflammation of a diverticulum from obstruction (by fecal matter) resulting in abscess formation.

A nurse is caring for a client newly diagnosed with hepatitis A. Which statement by the client indicates the need for further teaching? a) "I'll wash my hands often." b) "How did this happen? I've been faithful my entire marriage." c) "I'll take all my medications as ordered." d) "I'll be very careful when preparing food for my family."

B) How did this happen? I've been faithful my entire marriage The client requires further teaching if he suggests that he acquired the virus through sexual contact. Hepatitis A is transmitted by the oral-fecal route or through ingested food or liquid that's contaminated with the virus. Hepatitis A is rarely transmitted through sexual contact. Clients with hepatitis A need to take every effort to avoid spreading the virus to other members of their family with precautions such as preparing food carefully, washing hands often, and taking medications as ordered.

The nurse is preparing to interview a client with cirrhosis. Based on an understanding of this disorder, which question would be most important to include? a) "What type of over-the-counter pain reliever do you use?" b) "How often do you drink alcohol?" c) "Have you had an infection recently?" d) "Does your work expose you to chemicals?"

B) How often do you drink alcohol? The most common type of cirrhosis results from chronic alcohol intake and is frequently associated with poor nutrition. Although it can follow chronic poisoning with chemicals or ingestion of hepatotoxic drugs such as acetaminophen, asking about alcohol intake would be most important. Asking about an infection or exposure to hepatotoxins or industrial chemicals would be important if the client had postnecrotic cirrhosis.

Which of the following would be the highest priority information to include in preoperative teaching for a 68-year-old patient scheduled for a colectomy? A) How to care for the wound B) How to deep breathe and cough C) The location and care of drains after surgery D) What medications will be used during surgery

B) How to deep breathe and cough Because anesthesia, an abdominal incision, and pain can impair the patient's respiratory status in the postoperative period, it is of high priority to teach the patient to cough and deep breathe. Otherwise, the patient could develop atelectasis and pneumonia, which would delay early recovery from surgery and hospital discharge.

An elderly patient diagnosed with diarrhea is taking digoxin (Lanoxin). Which of the following electrolyte imbalances should the nurse be alert to? a) Hyponatremia b) Hypokalemia c) Hypernatremia d) Hyperkalemia

B) Hypokalemia The older person taking digitalis must be aware of how quickly dehydration and hypokalemia can occur with diarrhea. The nurse teaches the patient to recognize the symptoms of hypokalemia because low levels of potassium intensify the action of digitalis, leading to digitalis toxicity.

Paul Cavanagh, a 63-year-old retired teacher, had oral cancer and had extensive surgery to excise the malignancy. While is surgery was deemed successful, it was quite disfiguring and incapacitating. What is essential to Mr. Cavanagh and his family? a) Knowing that everything will work out just fine b) Time to mourn, accept, and adjust to the loss c) Not giving in to anger d) Having a courageous attitude

B) Time to mourn, accept, and adjust to the loss The first time family members or clients see the effects of surgery, the experience usually is traumatic. The nurse needs to promote effective coping and therapeutic grieving at this time. Responses may range from crying or extreme sadness and avoiding contact with others to refusing to talk about the surgery or changes in appearance. Allowing the client time to mourn, accept, and adjust to losses is essential.

Mr. T is nervous about a colonoscopy scheduled for tomorrow. The nurse describes the test by explaining that it allows which of the following? A) Visual examination of the esophagus and stomach B) Visual examination of the large intestine C) Radiographic examination of the large intestine D) Fluoroscopic examination of the small intestine

B) Visual examination of the large intestine An esophagogastroduodenoscopy Allows visual examination of the esophagus and stomach. The radiographic examination of the large intestine refers to a barium enema, and a fluoroscopic Examination of the small intestines refers to an upper gastrointestinal series

A client with severe and chronic liver disease is showing manifestations related to inadequate vitamin intake and metabolism. He reports difficulty driving at night because he cannot see well. Which of the following vitamins is most likely deficient for this client? a) Vitamin K b) Vitamin A c) Riboflavin d) Thiamine

B) Vitamin A Problems common to clients with severe chronic liver dysfunction result from inadequate intake of sufficient vitamins. Vitamin A deficiency results in night blindness and eye and skin changes. Thiamine deficiency can lead to beriberi, polyneuritis, and Wernicke-Korsakoff psychosis. Riboflavin deficiency results in characteristic skin and mucous membrane lesions. Vitamin K deficiency can cause hypoprothrombinemia, characterized by spontaneous bleeding and ecchymoses.

When reviewing the history of a client with pancreatic cancer, the nurse would identify which of the following as a possible risk factor? a) Ingestion of caffeinated coffee b) Ingestion of a low-fat diet c) History of pancreatitis d) One-time exposure to petrochemicals

C) History of pancreatitis Pancreatitis is associated with the development of pancreatic cancer. Other factors that correlate with pancreatic cancer include diabetes mellitus, a high-fat diet, and chronic exposure to carcinogenic substances (i.e., petrochemicals). Although data are inconclusive, a relationship may exist between cigarette smoking and high coffee consumption (especially decaffeinated coffee) and the development of pancreatic carcinoma.

Which organ relies almost exclusively on glucose for energy? Which do not?

Carbohydrates, which are more easily and quickly digested than proteins and lipids, fuel strenuous short-term skeleton muscle activity and *provide nearly all the energy for the brain.* Humans store glucose in liver and skeletal muscle tissue as glycogen. *Glycogen* is converted back into glucose to meet energy needs. This process is called *glycogenolysis*. If carbohydrates are not available, proteins and lipids (fats) can also be used for energy. If glycogen stores are low (for instance, in a person who is undernourished), physical activity causes the breakdown of body stores of protein (*gluconeogenesis*) and lipids (fats) to use for en- ergy. But when proteins are used for energy, they are not available for their primary functions of tissue growth, maintenance, and repair.

What is the most accurate method of verifying NG placement?

Chest XRay

What is the appropriate way to change an ostomy appliance?

Clean gloves, empty pouch, remove, lift skin barrier, clean skin with warm water, observe stoma, choose appropriate appliance, cut the opening, place adhesive, snap on pouch.

A client is scheduled for several diagnostic tests to evaluate her gastrointestinal function. After teaching the client about these tests, the nurse determines that the client has understood the teaching when she identifies which test as not requiring the use of a contrast medium? a) Small bowel series b) Colonoscopy c) Upper GI series d) Computer tomography

Colonoscopy Explanation: A colonoscopy is a direct visual examination of the entire large intestine. It does not involve the use of a contrast agent. Contrast medium may be used with a small bowel series, computed tomography, and upper GI series.

Which of the following is an outcome of histamine 2 (H2)-receptor antagonists blocking the action of histamine in the stomach? a) Blood phosphate levels are elevated. b) Symptoms of gastroesophageal reflux are relieved. c) Acid indigestion is relieved. d) Acid secretion is reduced.

D) Acid secretion is reduced H2-receptor antagonists decrease the amount of hydrochloric acid that the stomach produces by blocking the action of histamine on histamine receptors of potential cells in the stomach.

The nurse who inserted a nasogastric tube for a 68-year-old patient with suspected bowel obstruction should write which of the following priority nursing diagnoses on the patient's problem list? A) Anxiety related to nasogastric tube placement B) Abdominal pain related to nasogastric tube placement C) Risk for deficient knowledge related to nasogastric tube placement D) Altered oral mucous membrane related to nasogastric tube placement

D) Altered oral mucous membrane related to nasogastric tube placement With nasogastric tube placement, the patient is likely to breathe through the mouth and may experience irritation in the affected nares. For this reason, the nurse should plan preventive measures based on this nursing diagnosis.

A client is admitted to the hospital for diagnostic testing to rule out colorectal cancer. Which intervention should the nurse include on the plan of care? a) Prepare the client for a gastrostomy tube placement. b) Administer morphine (Duramorph PF) routinely, as ordered. c) Test all stools for occult blood. d) Administer topical ointment to the rectal area to decrease bleeding.

C) Test all stools for occult blood Blood in the stools is one of the warning signs of colorectal cancer. The nurse should plan on checking all stools for both frank and occult blood. The blood in the stool is coming from the colon or rectum; administering an ointment wouldn't help decrease the bleeding. Preparing a client for a gastrostomy tube isn't appropriate when diagnosing colorectal cancer. Colorectal cancer is usually painless; administering opioid pain medication isn't needed.

Which of the following indicates an overdose of lactulose? a) Hypoactive bowel sounds b) Constipation c) Watery diarrhea d) Fecal impaction

C) Watery diarrhea The patient receiving lactulose is monitored closely for the development of watery diarrheal stool, which indicates a medication overdose.

A client with diabetes begins to have digestive problems and is told by the physician that they are a complication of the diabetes. Which of the following explanations from the nurse is most accurate? a) Insulin has an adverse effect of constipation. b) The nerve fibers of the intestinal lining are experiencing neuropathy. c) Elevated glucose levels cause bacteria overgrowth in the large intestine. d) The pancreas secretes digestive enzymes.

D) The pancreas secretes digestive enzymes While the pancreas has the well-known function of secreting insulin, it also secretes digestive enzymes. These enzymes include trypsin, amylase, and lipase. If the secretion of these enzymes are affected by a diseased pancreas as foundi with diabetes, the digestive functioning may be impaired.

The most significant complication related to continuous tube feedings is a) an interruption in fat metabolism and lipoprotein synthesis. b) a disturbance in the sequence of intestinal and hepatic metabolism. c) the interruption of GI integrity, d) the potential for aspiration,

D) The potential for aspiration Because the normal swallowing mechanism is bypassed, consideration of the danger of aspiration must be foremost in the mind of the nurse caring for the patient receiving continuous tube feedings. Tube feedings preserve GI integrity by intraluminal delivery of nutrients. Tube feedings preserve the normal sequence of intestinal and hepatic metabolism. Tube feedings maintain fat metabolism and lipoprotein synthesis.

What lab value would you expect to see in a patient with diverticulitis?

Elevated WBC count.

What is the hallmark of acute pancreatitis?

Elevated serum amylase and lipase levels.

A patient with GERD complains of heartburn. Which food would you instruct the patient to eliminate from his diet?

Eliminate hot chocolate (because it's fatty, chocolate, and caffeinated).

What is the proper way to manage a jackson pratt drain (closed wound drainage device)? Sequential order

Empty before full, use aseptic technique unplug, empty contents, clean spout with alcohol, COMPLETELY COMPRESS COLLECTION CHAMBER, replace plug, check for kinks, position below level of wound, and document.

Which of the following should be included as part of the preprocedure teaching for a patient scheduled for a proctosigmoidoscopy involving the lower GI structures? a) Consume at least three quarts of water 30 minutes before the test b) Follow the dietary and fluid restrictions and bowel preparation procedures c) Spray or gargle with a local anesthetic d) Do not void for at least 30 minutes before the test

Follow the dietary and fluid restrictions and bowel preparation procedures Explanation: For a patient due to undergo a proctosigmoidoscopy, it is essential that the patient follows the dietary and fluid restrictions and bowel preparation procedures if the examination involves the lower GI structures. For the patient undergoing an esophagogastroduodenoscopy (EGD), it is necessary for the patient to spray or gargle with a local anesthetic. The patient is not advised to consume three quarts of water and is not advised to void before the test. These interventions may be essential for tests that involve ultrasonographic procedures.

One or two bowel sounds in 2 minutes would be documented as which of the following? a) Hypoactive b) Hyperactive c) Normal d) Absent

Hypoactive Explanation: Hypoactive bowel sound is the description given to auscultation of one to two bowel sounds in 2 minutes. Normal bowel sounds are heard every 5 to 20 seconds. Hyperactive bowel sounds occur when 5 or 6 sounds are heard in less than 30 seconds. The nurse records that bowel sounds are absent when no sound is heard in 3 to 5 minutes.

One or two bowel sounds in 2 minutes would be documented as which of the following? a) Hyperactive b) Absent c) Normal d) Hypoactive

Hypoactive Explanation: Hypoactive bowel sounds is the description given to auscultation of 1 to 2 bowel sounds in 2 minutes. Normal bowel sounds are heard every 5 to 20 seconds. Hyperactive bowel sounds occur when 5 or 6 sounds are heard in less than 30 seconds. The nurse records that bowel sounds are absent when no sound is heard in 3 to 5 minutes.

A patient with ulcerative colitis has had s/sx present for > 1 week. For what is patient at greatest risk?

Hypokalemia (lose K and Na in diarrhea).

Crohn's patient is admitted to the hospital with fever, abd pain, wt loss, and diarrhea. For what is he at greatest risk?

Hypokalemia.

When is the liver considered enlarged?

If the edge extends more than 3/4 inch below the right costal region.

As part of a routine screening for colon cancer

In assessing a 55 year-old client who is in the clinic for a routine physical, the nurse instructs the client about the need to proved a stool specimen for guaiac fecal occult blood testing: A. If the client notices rectal bleeding B. If there is a family history of intestinal polyps C. As part of a routine screening for colon cancer D. If a palpable mass is detected on digital exam

What can be done to decrease the risk of diverticulitis?

Increase dietary fiber by eating more fruits and whole grains, drink 80 ounces of water/day, and avoid foods with seeds or undigestible roughage.

Gastrin has which of the following effects on gastrointestinal (GI) motility? a) Relaxation of the colon b) Contraction of the ileocecal sphincter c) Increased motility of the stomach d) Relaxation of gastroesophageal sphincter

Increased motility of the stomach Explanation: Gastrin has the following effects on GI motility: increased motility of the stomach, excitation of the colon, relaxation of ileocecal sphincter, and contraction of the gastroesophageal sphincter.

A home care nurse is caring for a client with complaints of epigastric discomfort who is scheduled for a barium swallow. Which statement by the client indicates an understanding of the test? a) "I'll drink full liquids the day before the test." b) "There is no need for special preparation before the test." c) "I'll take a laxative to clear my bowels before the test." d) "I'll avoid eating or drinking anything 6 to 8 hours before the test."

D) I'll avoid eating or drinking anything 6 to 8 hours before the test The client demonstrates understanding of a barium swallow when he states that he must refrain from eating or drinking for 6 to 8 hours before the test. No other preparation is needed. Before a lower GI series, the client should eat a low-residue or clear liquid diet for 2 days and take a potent laxative and an oral liquid preparation.

When planning care for a patient with cirrhosis, the nurse will give highest priority to which of the following nursing diagnoses? A) Imbalanced nutrition: less than body requirements B) Impaired skin integrity related to edema, ascites, and pruritus C) Excess fluid volume related to portal hypertension and hyperaldosteronism D) Ineffective breathing pattern related to pressure on diaphragm and reduced lung volume

D) Ineffective breathing pattern related to pressure on diaphragm and reduced lung volume Although all of these nursing diagnoses are appropriate and important in the care of a patient with cirrhosis, airway and breathing are always the highest priorities.

Which of the following terms describes a gastric secretion that combines with vitamin B12 so that it can be absorbed? a) Amylase b) Trypsin c) Pepsin d) Intrinsic factor

D) Intrinsic factor Intrinsic factor, secreted by the gastric mucosa, combines with dietary vitamin B12 so that the vitamin can be absorbed in the ileum. In the absence of intrinsic factor, vitamin B12 cannot be absorbed and pernicious anemia results. Amylase is an enzyme that aids in the digestion of starch. Pepsin, an important enzyme for protein digestion, is the end product of the conversion of pepsinogen from the chief cells. Digestive enzymes secreted by the pancreas include trypsin, which aids in digesting protein.

Regarding oral cancer, the nurse provides health teaching to inform the patient that a) most oral cancers are painful at the outset. b) blood testing is used to diagnose oral cancer. c) a typical lesion is soft and craterlike. d) many oral cancers produce no symptoms in the early stages.

D) Many oral cancers produce no symptoms in the early stages The most frequent symptom of oral cancer is a painless sore that does not heal. The patient may complain of tenderness, and difficulty with chewing, swallowing, or speaking as the cancer progresses. Biopsy is used to diagnose oral cancer. A typical lesion in oral cancer is a painless hardened ulcer with raised edges.

The nurse would question the use of which of the following cathartic agents in a patient with renal insufficiency? A) Bisacodyl B) Lubiprostone C) Cascara sagrada D) Milk of magnesia

D) Milk of Magnesia Milk of magnesia may cause hypermagnesemia in patients with renal insufficiency. The nurse should question this order with the health care provider before administration.

A nurse is caring for a client who is undergoing a diagnostic workup for a suspected GI problem. The client reports gnawing epigastric pain following meals and heartburn. The nurse suspects the client has: a) diverticulitis. b) peptic ulcer disease. c) appendicitis. d) ulcerative colitis.

D) Peptic Ulcer Disease Peptic ulcer disease is characterized by dull, gnawing pain in the midepigastrium or the back that worsens with eating. Ulcerative colitis is characterized by exacerbations and remissions of severe bloody diarrhea. Appendicitis is characterized by epigastric or umbilical pain along with nausea, vomiting, and low-grade fever. Pain caused by diverticulitis is in the left lower quadrant and has a moderate onset. It's accompanied by nausea, vomiting, fever, and chills.

Which client requires immediate nursing intervention? The client who: a) complains of epigastric pain after eating. b) complains of anorexia and periumbilical pain. c) presents with ribbonlike stools. d) presents with a rigid, boardlike abdomen.

D) Presents with a rigid, boardlike abdomen A rigid, boardlike abdomen is a sign of peritonitis, a possibly life-threatening condition. Epigastric pain occurring 90 minutes to 3 hours after eating indicates a duodenal ulcer. Anorexia and periumbilical pain are characteristic of appendicitis. Risk of rupture is minimal within the first 24 hours, but increases significantly after 48 hours. A client with a large-bowel obstruction may have ribbonlike stools.

A pt reports having dry mouth and asks for some liquid to drink. The nurse reasons that this symptom can most likely be attributed to a common adverse effect of which of the following medications? A) Digoxin (Lanoxin) B) Cefotetan (Cefotan) C) Famotidine (Pepcid) D) Promethazine (Phenergan)

D) Promethazine (Phenergan) A common adverse effect of promethazine, an antihistamine antiemetic agent, is dry mouth; another is blurred vision.

The patient is on a continuous tube feeding. The tube placement should be checked every a) 24 hours. b) 12 hours. c) hour. d) shift.

D) Shift Each nurse caring for the patient is responsible for verifying that the tube is located in the proper area for continuous feeding. Checking for placement each hour is unnecessary unless the patient is extremely restless or there is basis for rechecking the tube based on other patient activities. Checking for placement every 12 or 24 hours does not meet the standard of care due the patient receiving continuous tube feedings.

A client with acute liver failure exhibits confusion, a declining level of consciousness, and slowed respirations. The nurse finds him very difficult to arouse. The diagnostic information which best explains the client's behavior is: a) elevated liver enzymes and low serum protein level. b) subnormal clotting factors and platelet count. c) elevated blood urea nitrogen and creatinine levels and hyperglycemia. d) subnormal serum glucose and elevated serum ammonia levels.

D) Subnormal serum glucose and elevated serum ammonia levels. In acute liver failure, serum ammonia levels increase because the liver can't adequately detoxify the ammonia produced in the GI tract. In addition, serum glucose levels decline because the liver isn't capable of releasing stored glucose. Elevated serum ammonia and subnormal serum glucose levels depress the level of a client's consciousness. Elevated liver enzymes, low serum protein level, subnormal clotting factors and platelet count, elevated blood urea nitrogen and creatine levels, and hyperglycemia aren't as directly related to the client's level of consciousness.

Why should total parental nutrition (TPN) be used cautiously in clients with pancreatitis? a) Such clients can digest high-fat foods. b) Such clients are at risk for hepatic encephalopathy. c) Such clients are at risk for gallbladder contraction. d) Such clients cannot tolerate high-glucose concentration.

D) Such clients cannot tolerate high glucose concentration Total parental nutrition (TPN) is used carefully in clients with pancreatitis because some clients cannot tolerate a high-glucose concentration even with insulin coverage. Intake of coffee increases the risk for gallbladder contraction, whereas intake of high protein increases risk for hepatic encephalopathy in clients with cirrhosis. Patients with pancreatitis should not be given high-fat foods because they are difficult to digest.

An elderly client asks the nurse how to treat chronic constipation. What is the best recommendation the nurse can make? a) Take a mild laxative such as magnesium citrate when necessary. b) Administer a tap-water enema weekly. c) Administer a phospho-soda (Fleet) enema when necessary. d) Take a stool softener such as docusate sodium (Colace) daily.

D) Take a stool softener such as docuaste sodium (Colace) daily. Stool softeners taken daily promote absorption of liquid into the stool, creating a softer mass. They may be taken on a daily basis without developing a dependence. Dependence is an adverse effect of daily laxative use. Enemas used daily or on a frequent basis can also lead to dependence of the bowel on an external source of stimulation.

The nurse would instruct the patient to do which of the following to best enhance the effectiveness of a daily dose of docusate sodium (Colace)? A) Take a dose of mineral oil at the same time. B) Add extra salt to food on at least one meal tray. C) Ensure dietary intake of 10 g of fiber each day. D) Take each dose with a full glass of water or other liquid.

D) Take each dose with a full glass of water or other liquid Docusate lowers the surface tension of stool, permitting water and fats to penetrate and soften the stool for easier passage. The patient should take the dose with a full glass of water and should increase overall fluid intake, if able, to enhance effectiveness of the medication. Dietary fiber intake should be a minimum of 20 g daily to prevent constipation.

Blood shed in sufficient quantities into the upper GI tract, produces which color of stool? a) Bright red b) Milky white c) Green d) Tarry-black

D) Tarry-black Blood shed in sufficient quantities into the upper GI tract produces a tarry-black stool. Blood entering the lower portion of the GI tract or passing rapidly through it will appear bright or dark red. A milky white stool is indicative "of" a patient who received barium. A green stool is indicative of a patient who has eaten spinach.

A client with diabetes begins to have digestive problems and is told by the physician that they are a complication of the diabetes. Which of the following explanations from the nurse is most accurate? a) The pancreas secretes digestive enzymes. b) Elevated glucose levels cause bacteria overgrowth in the large intestine. c) The nerve fibers of the intestinal lining are experiencing neuropathy. d) Insulin has an adverse effect of constipation.

The pancreas secretes digestive enzymes. Explanation: While the pancreas has the well-known function of secreting insulin, it also secretes digestive enzymes. These enzymes include trypsin, amylase, and lipase. If the secretion of these enzymes are affected by a diseased pancreas as foundi with diabetes, the digestive functioning may be impaired.

What is characteristic of an ileal conduit? select all that apply

It collects urine, kidneys work constantly, never irrigate, urine must be clear not cloudy, mucus in urine is common, empty q3hrs, tampon or rolled gauze to get bag to stick on, the bag is worn for over a year and clean with vinegar.

What is the blue pigtail on an NG tube?

It is an airport used for draining air from the stomach.

What should you teach your patient about a colostomy?

It is relocation of your butt, bowels will need to be cleaned before surgery, neomycin and laxative results: death of e.coli, risk of bleeding, and vitamin K in OR, stoma should be red and beefy, clean with soap and water, "fart" in bag let out of bag or bust and make mess.

If delivering meds via gastrostomy and by gravity what should be done?

It should be 18 inches above body only.

What does the colonoscopy procedure include?

It takes 30-60 mins, an IV is inserted, if bowel spasms occur administer IV anticholinergics and glucagon.

What are S&S of liver disease?

Jaundice (icterus), yellow sclera (light skin), soft palate (dark skin), bile=urine bright orange, stool=clay colored d/t lack of bile in intestine.

A nurse is reviewing laboratory test results from a client. The report indicates that the client has jaundice. What serum bilirubin level must the client's finding exceed? Enter the correct number only.

Jaundice becomes clinically evident when the serum bilirubin level exceeds 2.5 mg/dL (43 fmol/L).

What is a good treatment for younger adults with crohns disease?

Kocks pouch (internal bag good bc don't want ppl to see/know).

What is the proper management of a nasoenteric compression tube?

Leave the tube anchored so peristaltic movement can advance into the intestine, reposition patient on right side to help pass tube through pyloric sphincter, stay for 2 hours then turn to left side, encourage walking, once in position aspirate fluid through tube and test pH (less 7=stomach, >7= intestine), provide oral and nasal hygiene at least once/shift.

2, 1, 3, 4, 6, 5, 7 Perform hand hygiene, and apply clean gloves. Remove the used pouch and skin barrier. Asses the stoma for color, swelling, and healing. Gently cleanse the preistomal skin with warm tap water. Cut an opening on the pouch 1/16 inch larger than the stoma. Apply nonallergenic tape around the pectin skin barrier. Press the adhesive bakcing of the pouch smoothly against the skin.

List the correct order in which to apply an ostomy pouch: 1. Remove the used pouch and skin barrier 2. Perform hand hygiene, and apply clean gloves. 3. Asses the stoma for color, swelling, and healing. 4. Gently cleanse the preistomal skin with warm tap water. 5. Apply nonallergenic tape around the pectin skin barrier. 6. Cut an opening on the pouch 1/16 inch larger than the stoma. 7. Press the adhesive bakcing of the pouch smoothly against the skin.

What kind of diet should someone with cholecystitis eat?

Low fat and calories, increased protein to re-build liver.

What are the two types of antacids?

Magnesium (Mag) (Ex: milk of mag) based and aluminum (Al) (Ex: amphogel) based.

What are nursing interventions for peritonitis?

Maintain fluid and electrolyte balance and decrease GI distention (NG SUCTION, IVs-NS/LR, K supplements), decrease infection risk, prevent immobility complications.

If a client has abdominal surgery and a portion of the small intestine is removed, the client is at risk for which of the following? a) Cirrhosis b) Gastric ulcers c) Constipation d) Malabsorption syndrome

Malabsorption syndrome Explanation: Absorption is the primary function of the small intestine. Vitamins and minerals are absorbed essentially unchanged. Nutrients are absorbed at specific locations in the small intestine.

What is a nasoenteric compression tube (AKA miller/abbott or cantar tube)?

May be used for patients with bowel obstruction to remove intestinal contents.

What is the correct way to insert an NG tube?

Measure distance from tip of nose to earlose and to xiphoid process. Mark the length with tape, lubricate tubing (4inches) with water soluble lube, give patient water with straw, have extend neck back, insert tube advance, have client place head forward and drink sips of water, verify correct tube placement by aspirating stomach contents, observing color and get an xray to confirm.

What can cause a bowel obstruction?

Mechanical blockage (scar tissue, tumor, endometriosis) or paralytic ileus (decreased bowel sounds).

What complication do aluminum based antacids have?

Men (Al) are full of poop (CONSTIPATION).

A client presents to the emergency department, reporting that he has been vomiting every 30 to 40 minutes for the past 8 hours. Frequent vomiting puts this client at risk for which imbalances? a) Metabolic alkalosis and hyperkalemia b) Metabolic acidosis and hypokalemia c) Metabolic alkalosis and hypokalemia d) Metabolic acidosis and hyperkalemia

Metabolic alkalosis and hypokalemia Explanation: Gastric acid contains large amounts of potassium, chloride, and hydrogen ions. Excessive vomiting causes loss of these substances, which can lead to metabolic alkalosis and hypokalemia. Excessive vomiting doesn't cause metabolic acidosis or hyperkalemia.

gastric protectant: an antisecretory med that enhances mucosal defenses & suppresses secretion of gastric acid & maintains submucosal blood flow by promoting vasodilation; used to prevent ulcers w/ NSAID use

Misoprostyl (cytotec)

What is the treatment for acute pancreatitis?

Monitor serum glucose/administer insulin, monitor serum calcium (low calcium=shaking), teach to take pancreatic enzymes with meals to aid digestion (Viokase), tell pt to avoid alcohol and caffeine, low fat, low protein, high carb diet.

Which of the following would be most important to ensure that a client does not retain any barium after a barium swallow? a) Observing the color of urine. b) Placing any stool passed in a specific preservative. c) Monitoring the stool passage and its color. d) Monitoring the volume of urine.

Monitoring the stool passage and its color. Explanation: Monitoring stool passage and its color will ensure that the client remains barium free following a barium swallow test. The white or clay color of the stool would indicate barium retention. The stool should be placed in a special preservative if the client undergoes a stool analysis. Observing the color and volume of urine will not ensure that the client is barium free because barium is not eliminated through urine but through stool.

What can be used to decrease risk of tearing the skin (dehiscence)?

Montgomery Straps

The small intestine

Most nutrients and electrolytes are absorbed in: A. The colon B. The stomach C. The esophagus D. The small intestine

What is the antidote for tylenol?

Mucomyst

What are the S&S of pancreatitis?

N&V, bloating, Cullens sign (bluish color around umbilicus) or flank (Turners sign) may signal hemorrhage.

Who should not manage a T tube?

NAs

Is someone with cholecystitis a priority patient?

NO

Should you ever use oil for an NG tube?

NO

What should the patient/nurse NOT DO for someone with appendicitis to prevent rupture?

NO bearing down and No enemas.

What is characteristic of Kock's pouch?

No bag is worn (internally placed), insert catheter and flush to empty.

4, 5, 2, 1, 3 Perform hand hygiene, and apply clean gloves. Insert tip of syringe into NG tube, and slowly inject 30 mL saline Reconnect the NG tube to suction. Slowly aspirate the syringe. Clamp and disconnect the NG tube

Number the steps to irrigating a nasogastric tube in correct order: 1. Slowly aspirate the syringe. 2. Reconnect the NG tube to suction. 3. Clamp and disconnect the NG tube 4. Perform hand hygiene, and apply clean gloves. 5. Insert tip of syringe into NG tube, and slowly inject 30 mL saline.

What should the nurse do for a patient undergoing paracentesis?

Obtain consent, assess baseline vital and abdominal girth, HAVE THE PT VOID AND POSITION HIM, observe for hypovolemia during removal and observe fluid characteristics, measure abdominal girth for comparison, document and monitor for fever, pain, hematuria, or bleeding from puncture site. DON'T LET FLUID SIT SEND ORDERS TO LAB IMMEDIATELY!

What is a balloon tamponade (blakemore tube)? Consideration?

One balloon in stomach and one in esophagus. Must have scissors available at bedside to snip top and remove air if need to deflate.

What is the correct way to manage a T-tube?

Put on sterile gloves clean around tube with NaCL, air dry then apply petroleum jelly around tube to protect skin, and put a sterile precut dressing around tube.

The client feels nauseated The client oozes liquid stool The client has continous bowel sounds

The nurse begins to suspect a fecal impaction in a client who has not had a stool in 10 days when which of the following occurs: A. The client feels nauseated B. The client oozes liquid stool C. The client has a rounded abdomen D. The client has continous bowel sounds.

Fish Raw vegetables

The nurse instructs the client to avoid which of the following foods, which could give a false reading on the fecal occult blood test? (Select all the apply.) A. Fish B. Lasagna C. Cranberry juice D. Raw vegetables

A nursing instructor tells the class that review of oral hygiene is an important component during assessment of the gastrointestinal system. One of the students questions this statement. Which of the following explanations from the nurse educator is most appropriate? a) "Bad breath will encourage ingestion of fatty foods to mask odor." b) "Injury to oral mucosa or tooth decay can lead to difficulty in chewing food." c) "Decaying teeth secrete toxins that decrease the absorption of nutrients." d) "Mouth sores are caused by bacteria that can thin the villi of the small intestine."

Injury to oral mucosa or tooth decay can lead to difficulty in chewing food." Explanation: Poor oral hygiene can result in injury to the oral mucosa, lip, or palate; tooth decay; or loss of teeth. Such problems may lead to disruption in the digestive system. The ability to chew food or even swallow may be hindered.

What should you do immediately for someone with a ruptured diverticulum?

Insert an NG tube and attach to low suction, insert large bore IV, and anticipate immediate surgery.

The nurse assesses a client who is reporting mild abdominal cramping. How will the nurse proceed with assessment of this client? a) Auscultation, inspection, percussion, palpation b) Inspection, auscultation, percussion, palpation c) Palpation, inspection, percussion, auscultation d) Inspection, percussion, auscultation, palpation

Inspection, auscultation, percussion, palpation Explanation: Accurate assessment skills are necessary to ensure client safety. Auscultation always precedes percussion and palpation, which may alter sounds. Inspection remains the first physical assessment skill the nurse should perform.

Which of the following sequence should be used to assess the abdomen? a) Palpation, inspection, percussion, auscultation b) Auscultation, inspection, percussion, palpation c) Inspection, auscultation, percussion, palpation d) Percussion, auscultation, palpation, inspection

Inspection, auscultation, percussion, palpation Explanation: Assessment begins with an overall visual inspection of the abdomen followed by auscultation, which always precedes percussion and palpation since manipulation of the abdomen may alter the frequency and intensity of bowel sounds. Inspection allows the nurse to visualize the skin, umbilicus, contour, and symmetry of abdomen and any movement or pulsations.

Aspirin Nonsteroidal anti-inflammatory drugs (NSAIDS)

The nurse is obtaining a client's medication history. Which of the follwing mediactions my cause gastrointestinal bleeding? (Select all that apply.) A. Aspirin B. Cathartics C. Antidiarrheal opiate agents D. Nonsteroidal anti-inflammatory drugs (NSAIDS)

Beans Onions Cauliflower

The nurse teaches clients with a new colostomy that they can eat whatever roods they like but that which of the follwing foods typically produce gas and should be consumed cautiously? (Select all that apply? ) A. Pasta B. Beans C. Garlic D. Onions E. Cauliflower

Upon review of a client's chart, the nurse notes the client has been receiving antiemetics every 6 to 8 hours. What in this client's history may necessitate such frequency? a) Pituitary tumor b) Adrenal gland removal 3 days ago c) Treatment for cancer d) Multiple leg fractures

Treatment for cancer Explanation: Antiemetics are used to treat nausea and vomiting. Common causes of nausea and vomiting include visceral afferent stimulation, peritoneal irritation, infections, radiation or chemotherapy therapy, increased intracranial pressure, and vestibular disorders. Irritation of the chemoreceptor trigger zone from cancer treatment can induce nausea and lead to vomiting.

When describing the role of the pancreas to a client with a pancreatic dysfunction, the nurse would identify which substance as being acted on by pancreatic lipase? a) Protein b) Glucose c) Triglycerides d) Starch

Triglycerides Explanation: Pancreatic lipase acts on lipids, especially triglycerides. Salivary amylase and pancreatic amylase act on starch. Pepsin and hydrochloric acid in the stomach and trypsin from the pancreas act on proteins. Insulin acts on glucose.

True or False? The secretion of hydrochloric acid and pepsinogen is stimulated by the sight, smell, and taste of food.

True

What drug should you never give to liver people?

Tylenol

What are the S&S of peritonitis?

Tympanic membrane, pain over area, REBOUND TENDERNESS, ABD DISTENTION, DECREASED BOWEL SOUNDS, ABD RIGIDITY (BOARD-LIKE), fever, N&V, dehydration, and increased pulse and BP.

what is the patho difference between Ulcerative Colitis and Crohn's disease?

UC: ulcerative inflammatory bowel disease; just in the large intestine Crohn's Disease: also called regional Enteritis; inflammation and erosion of the ilium; can be fouind anywhere, but most of the time it is in the small intestine

Understand the difference between positive and negative nitrogen balance. Give examples of different patient populations for each.

When amino acids are catabolized, the nitrogen-containing part is converted to ammonia (NH3) and excreted in the urine as urea. Therefore, nitrogen balance reflects how well body tissues are being maintained. Nitrogen balance occurs when intake and output of nitrogen are equal. A *positive nitrogen balance* exists when nitrogen intake exceeds output, making a pool of amino acids available for growth, pregnancy, and tissue maintenance and repair. *Negative nitrogen balance* exists when nitrogen intake is lower than nitrogen loss. This occurs in illness, injury (e.g., burns), and malnutrition.

Ondansetron is an antiemetic used for

post op N/V as well as N/V induced w/ chemotherapy

What does rebound tenderness mean?

push in...then let go...if there is pain on the let go This means peritoneal inflammation

dumping syndrome

rapid emptyingn of gastic contents into intestine post gastric resection 30 min post meal: n/v/d, fullness, borborygmi, cramp, tachy tx: no sugar, salt, milk eat low carb/high fat/protein, small meals w/o fluids, lie down post meal, antispasmodics

Infliximab is an immunodulator that

reduces inflammation in Crohns disease, thereb reducing diarrhea

bariatric surgery

reduction of gastric capacity limited to liquids, pureed foods for 6 weeks post surgery

A nurse is teaching a client with malabsorption syndrome about the disorder and its treatment. The client asks which part of the GI tract absorbs food. The nurse tells the client that products of digestion are absorbed mainly in the: a) large intestine. b) small intestine. c) rectum. d) stomach.

small intestine. Explanation: The small intestine absorbs products of digestion, completes food digestion, and secretes hormones that help control the secretion of bile, pancreatic juice, and intestinal secretions. The stomach stores, mixes, and liquefies the food bolus into chyme and controls food passage into the duodenum; it doesn't absorb products of digestion. Although the large intestine completes the absorption of water, chloride, and sodium, it plays no part in absorbing food. The rectum is the portion of the large intestine that forms and expels feces from the body; its functions don't include absorption.

hiatal hernia

stomach herniates into thorax regurg, full feeling, hearburn --antacids, H agonists, proton pump inhibitors --avoid cholinergics

triple therapy of h. pylori infection include

two antibacterial drugs and a proton pump inhibitor

variceal ligation

tying of varicies with elastic rubber band

Misoprostol is a gastric protectant and is given to prevent

ulcers w/ NSAID and aspirin use

Choice Multiple question - Select all answer choices that apply. An examiner is performing the physical assessment of the rectum, perianal region, and anus. While this examination can be uncomfortable for many clients, health care providers must approach it in a prepared, confident manner. Which of the following considerations will help this examination flow smoothly and efficiently for both provider and client? Select all that apply. a) Position the client on the right side with the knees up to the chest. b) Dim the lights to decrease the client's embarrassment. c) Cleanse gloved fingers with water to allow for easy insertion. d) Ask the client to produce a bowel movement after the procedure. e) Ask the client to bear down for visual inspection.

• Position the client on the right side with the knees up to the chest. • Ask the client to bear down for visual inspection. Explanation: While examination of the rectum, perineum, and anus may be uncomfortable for the client, it is necessary for a thorough examination. The examiner will position the client on the right side with the knees up. He or she will use a gloved finger lubricated with a water-soluble lubricant for ease of insertion. The health care provider will encourage deep breathing during the procedure and ask the client to bear down while inspecting the anal area. The examination requires appropriate lighting for thorough inspection.

Carbohydrates

■ *Carbohydrates* are the preferred source of energy. ■ Sugars, starches and cellulose provide 4 cal/g ■ Carbohydrates promote normal fat *metabolism* ■ Major food sources of carbohydrates include milk, grains, fruits and vegetables *Food Sources of Carbohydrates* *Cellulose*: Apples, Beans, Bran, Cabbage *Fructose*: Fruits, honey *Glucose*: Carrots, corn, dates, grapes, oranges *Lactose*: milk *Starch*: barley, beets, carrots, peas, corn, oats, potatoes and pasta, rye, wheat *Sucrose*: apricots, granulated table sugar, honeydew, cantelope, molasses, peaches, peas, corn, plums

Fats

■ *Fats* provide a concentrated source and a stored form of energy. ■ Fats protect internal organs and maintain body temperature ■ Fats enhance *absorption* of the fat-soluble vitamins. ■ Fats provide 9 cal/g. ■ Inadequate fat intake leads to clinical manifestations of sensitivity to cold, skin lesions, increased risk of infection, and amenorrhea in women. ■ Diets high in fat can lead to obesity and increase the risk of cardiovascular disease and some cancers. *Food sources of fats* *Cholesterol*: animal products, egg yolks, liver and organ meats, shelfish *Monounsaturated fats*: duck and goose, eggs, olive and peanut oils *Polyunsaturated fats*: corn, safflower, sunflower oils *Saturated fats*: beef, butter, hard yellow cheeses, luncheon meats

Proteins

■ Amino acids, which make up proteins, are critical to all aspects of growth and development of body tissues, and provide 4 cal/g ■ Proteins build and repair body tissues, regulate fluid balance, maintain acid-base balance, produce antibodies, provide energy, and produce enzymes and hormones ■ Essential amino acids are required in the diet because the body cannot manufacture them. ■ High-quality proteins or complete proteins such as eggs, dairy products, meat, fish, and poultry contain adequate amounts of essential amino acids. ■ Foods that do not contain the essential amino acids in sufficient amounts are lower quality or incomplete proteins. ■ Inadequate protein can cause protein energy *malnutrition* and severe wasting of fat and muscle tissue. *Food sources of protein* bread and cereal products dairy products dried beans eggs meats, fish, poultry

Minerals

■ Minerals are components of hormones, cells, tissues, and bones ■ Minerals act as catalysts for chemical reactions and enhancers of cell function ■ Almost all foods contain some form of minerals ■ A deficiency of minerals can occur in chronically ill or hospitalized patients ■ Electrolytes play a major role in osmolarity and body water regulation, acid-base balance, enzyme reactions, and neuromuscular activity. *Food Sources of Minerals* *Calcium*: broccoli, carrots, cheese, collard greens, green beans, milk, rhubarb, spinach, tofu, yogurt *Chloride*: salt *Magnesium*: avocado, canned white tuna, cauliflower, cooked rolled oats, green leafy vegetables, milk, peanut butter, peas, pork, beef, chicken, potatoes, raisins, yogurt *Phosphorus*: fish, nuts, organ meats, pork, beef, chicken, whole-grain breads and cereals *Potassium*: avocado, bananas, cantaloupe, carrots, fish, mushrooms, oranges, pork, beef, veal, potates, raisins, spinach, strawberries, tomatoes *Sodium*: american cheese, bacon, butter, canned food, cottage cheese, cured pork, hot dogs, ketchup, lunch meat, milk, mustard, processed food, snack food, soy sauce, table salt, white and whole-grain bread *Iron*: breads and cereals, dark green vegetables, egg yolk, liver, meats *Zinc*: eggs, leafy vegetables, meats, protein-rich foods

The nurse asks a client to point to where she feels pain. The client asks why this is important. The nurse's best response would be which of the following? a) "This determines the pain medication to be ordered." b) "The area may determine the severity of the pain." c) "If the doctor massages over the exact painful area, the pain will disappear." d) "Often the area of pain is referred from another area."

You selected: "Often the area of pain is referred from another area." Explanation: Pain can be a major symptom of disease. The location and distribution of pain can be referred from a different area. If a client points to an area of pain and has other symptoms associated with a certain disease, this is valuable information for treatment.

pancreatitis

acute or chronic inflammation of pancreas cullens sign (bluish discoloration of ab, umbilicus) turners sign (bluish discoloration of flanks) acute = ab pain, up post fatty meal, booze, lying down, elevated enzymes, billirubin, WBCs chronic = same plus jaundice, wasting, DM tx; demerol, antacids, h agonists, antispasmodics --avoid alcohol

A client is admitted to the healthcare facility suspected of having acute pancreatitis and undergoes laboratory testing. Which of the following would the nurse expect to find? a) Decreased white blood cell count b) Increased serum calcium levels c) Elevated urine amylase levels d) Decreased liver enzyme levels

c) Elevated urine amylase levels Elevated serum and urine amylase, lipase, and liver enzyme levels accompany significant pancreatitis. If the common bile duct is obstructed, the bilirubin level is above normal. Blood glucose levels and white blood cell counts can be elevated. Serum electrolyte levels (calcium, potassium, and magnesium) are low.

A patient receives atropine, an anticholinergic drug, in preparation for surgery. The nurse expects this drug to affect the GI tract by: a) increasing gastric emptying b) relaxing pyloric and ileocecal sphincters c) decreasing secretions and peristaltic action d) stimulating the nervous system of the GI tract

c) decreasing secretions and peristaltic action The parasympathetic nervous system increasing motility and secretions and relaxing sphincters to promote movement of contents. A drug that blocks this activity decreases secretions and peristalsis, slows gastric emptying, and contracts sphincters. The enteric nervous system of the GI tract is modulated by sympathetic and parasympathetic influence

When caring for a patient who has had most of the stomach surgically removed, the nurse plans to teach the client a) that extra iron will need to be taken to prevent anemia b) to avoid foods with lactose to prevent diarrhea and bloating c) that lifelong supplementation of cobalamin will be needed d) that, because of the absence of digestive enzymes, protein malnurition is likely

c) that lifelong supplementation of cobalamin will be needed The stomach secretes intrinsic factor necessary for cobalamin absorption in the intestine. In removal of part or all of the stomach, cobalamin must be supplemented for life.

cirrhosis

chronic, progressive disease of liver fetor hepaticus asterixis portal HTN, ascites, esophageal varicies, poor coagulation (K), jaundice, encephalopathy, hepatorenal failure, death,

A client is to have an upper GI procedure with barium ingestion and abdominal ultrasonography. While scheduling these diagnostic tests, the nurse must consider which factor? a) The client may eat a light meal before either test. b) Both tests need to be done before breakfast. c) The upper GI should be scheduled before the ultrasonography. d) The ultrasonography should be scheduled before the GI procedure.

d) The ultrasonography should be scheduled before the GI procedure Both an upper GI procedure with barium ingestion and an ultrasonography may be completed on the same day. The ultrasonography test should be completed first, because the barium solution could interfere with the transmission of the sound waves. The ultrasonography test uses sound waves that are passed into internal body structures, and the echoes are recorded as they strike tissues. Fluid in the abdomen prevents transmission of ultrasound.

After eating, a patient with an inflamed gallbladder experiences pain caused by contraction of the gallbladdder. The mechanism responsible for this action is a) production of bile by the liver b) production of secretin by the duodenum c) release of gastrin from the stomach antrum d) production of cholecystokinin by the duodenum

d) production of cholecystokinin by the duodenum Cholecystokinin is secreted by the duodenal mucosa when fats and amino acids enter the duodenum and stimulates the gallbladder to release bile and emulsify the fats for digestion. The bile is produced by the liver but stored in the gallbladder. Secretin is responsible for stimulating pancreatic bicarbonate secretion, and gastrin increases gastric motility and acid secretion.

esophageal varicies (distended veins)

dilated veins in esophagus from liver hypertension/cirrhosis bleed is EMERGENCY tx iced saline, vasopressin, sclerotherapy, ligation, shunt/anastamosis of splenic/renal veins

hemorrhoids

dilated veins in the anal canal from portal HTN, straining tx side lie, sitz bath, cold packs, fiber, fluids, stool softener, surgery (scleropathy,s stapling, resection

vitamin b12 deficiency

inadequate intake or lack of absorption beefy, red, tongue pernicious anemia tx b12 injections

What is the most frequent complication of TPN?

infection

gastritis

inflammation of stomach --antibiotic, bismuth salt (pepto) --avoid NSAIDs, aspirin, alcohol

diverticulitis

inflammation or herniation of intestinal mucosa LLQ pain, poss. peritonitis tx NPO then up liquids, fiber diet, antibiotics, antispasmodics, analgesics, surgery (colon resection) avoid straining

ulcerative colitis

inflammatory disease of bowel diarrhea, dehydration, k deficiency, anemia tx: bland and non-gassy diet, surgery (proctocolectomy, illeostomy)

crohns disease

inflammatory disease that occurs throughout GI tract cramp post meal, diarrhea tx: bland and non-gassy diet

Questron therapy is use not compliant due to

its terrible taste; encourage to drink w/ fruit juice

A client is to have an upper GI procedure with barium ingestion and abdominal ultrasonography. While scheduling these diagnostic tests, the nurse must consider which factor? a) The ultrasonography should be scheduled before the GI procedure. b) The upper GI should be scheduled before the ultrasonography. c) The client may eat a light meal before either test. d) Both tests need to be done before breakfast.

The ultrasonography should be scheduled before the GI procedure. Explanation: Both an upper GI procedure with barium ingestion and an ultrasonography may be completed on the same day. The ultrasonography test should be completed first, because the barium solution could interfere with the transmission of the sound waves. The ultrasonography test uses sound waves that are passed into internal body structures, and the echoes are recorded as they strike tissues. Fluid in the abdomen prevents transmission of ultrasound.

What should you teach your pt about a liver biopsy? (before, during, and after)

They should avoid aspirin, NSAIDs, and anticoagulants for at least 2 weeks prior and they should fast 6 hours before exam. During the procedure they will be positioned on their left side or supine with their right arm above head, the pt should exhale and hold breath while needle is inserted. After the procedure the patient should lie on his/her right side for 2 hours (plug).

How do you know if someone's crohn's is under control?

They will gain weight

Who is most at risk for stomach CA?

African American and oriental males.

What should you instruct patient with dumping syndrome to do to manage his condition?

After eating lie on left side (food remains trapped in stomach) and change diet (no fluids c meals, no salt, and no high carbs (potatoes/bread).

Blood glucose norms

70 mg/dL to 1 00 mg/dL. Capillary blood sugar is frequently assessed at the bedside with a simple fi ngerstick. Serum blood glucose levels are assessed by drawing a venous blood sample. The level is measured in the laboratory. The American Diabetes Association (2006) recommends for diabetic patients a preprandial plasma glucose level of 70 to 130 mg/dL.

How do you know if gastrostomy tube feedings are effective?

Albumin is 3.5-5

What is the nurses most accurate method of verifying NG?

Aspiration of stomach contents

A patient with GERD complains of a chronic cough. What potential complication does this signal?

Aspiration of stomach contents.

What should you do for an abdominal assessment?

Assess for any visible masses, bulging, or asymmetry, place the diaphragm over each quadrant listening for 5 minutes, document bowel sounds, listen with bell over aorta (with hypertension may hear a bruit (abnormal), lightly percuss (dull sounds over solid structures not lungs), palpate with the palm aspect of fingers of dominant hand light then deep (avoid deep palpation if your patient may have splenomegaly, appendicitis, or aneurysm), palpate the liver (have pt inhale during). Assessment should take 12-20 mins.

A nurse assesses the abdomen of a newly admitted client. Which finding would necessitate further investigation? a) Flat appearance below the umbilicus b) Rounded contour c) Asymmetrical upper quadrants d) Striae of lateral abdomen

Asymmetrical upper quadrants Explanation: The client lies supine with knees flexed for the abdominal assessment. Upon inspection the nurse notes any skin changes, nodules, lesions, inflammation, or striae. Lesions are of particular importance and require further investigation, as do irregular contours or asymmetry of the abdomen.

What are S&S?

Diarrhea, abdominal pain and tenderness (board-like), N&V, borborygmi (loud bowel sounds), and steatorrhea (d/t dec. bile salt absorption). Later S&S= dehydration, weight loss, electrolyte imbalance, anemia.

Milk of Magnesia is given for the patient's hiatal hernia. What is an adverse effect of taking MOM for too long?

Diarrhea.

Iron

*Function*: Synthesis of hemoglobin, general metabolism (e.g., of glucose), antibody production, drug detoxification in the liver *Sources:* Meats, eggs, spinach, seafood, broccoli, peas, bran, enriched breads, fortified cereals *Effects of deficiency*: Small, pale RBCs, anemia *Symptoms of excess*: Hemochromatosis

A client with diverticular disease undergoes a colonoscopy. When conducting an abdominal assessment, the nurse looks for which of the following as a sign of possible complication? 1. Diarrhea 2. Nausea and vomiting 3. Guarding and rebound tenderness 4. Redness and warmth of the abdominal skin

3 Bowel perforation is a possible result of a colonoscopy if the colonoscope accidentally pierces the bowel wall. Perforation could lead to symptoms of periotonitis, such as guarding and rebound tenderness. The other options are incorrect because 1,2 are signs of obstruction and 4 is not of concern. Core issue: Assessment data that correlates with complications of colonoscopy, such as peritonitis.

The nurse is conducting dietary teaching with a client who has dumping symdrome. The nurse encourages the client to avoid which of the foods that the client usually enjoys? 1. Eggs 2. Cheese 3. Fruit 4. Pork

3 Dumping syndrome, in which gastric contents rapidly enter the bowel, can occur following gastrectomy. Dietary fats and proteins are increased, and carbohydrates such as fruits, are reduced. This helps slow the GI transit time and reduce the GI cramping, diarrhea, and vasomotor symptoms associated with dumping syndrome. Core issue: Knowledge of foods to avoid when the client has dumping syndrome.

The nurse is caring for a client with a history of alcoholism. Which of the following findings would indicate that the client has possibly developed chronic pancreatitis? 1. Steady weight gain 2. Flank pain on the left side only 3. Fatty stools 4. Excessive hunger

3 Manifestations of chronic pancreatitis include vomiting, nausea, weight loss, flatulence, constipation, and steatorrhea that result from a decreases in pancreatic enzyme secretion. Weight gain is the opposite of what occurs with this disorder, while options 2 and 4 are unrelated. Core issue: Ability to identify assessment findings that are consistent with the development of chronic pancreatitis.

The nurse is caring for a client who has ascites and the health care provider prescribes spironolactone (Aldactone). The client asks why this drug is being used. Which is the best response by the nurse? 1. "This drug will help increase the level of protein in your blood" 2. "The drug will cause an increase in the amount of hormone aldosterone your body produces" 3. "This medication is a diuretic but does not make the kidneys excrete potassium" 4. "This will help you excrete larger amounts of ammonia"

3 Spironolactone (Aldactone) is used in clients with ascites that show no improvement with bedrest and fluid restriction. It inhibits sodium reabsorption in the distal tubule and promotes potassium retention by inhibiting aldosterone. The other options do not address this rationale. Core issue: Knowledge of medication effects in a client with ascites.

In caring for the client 4 days post-cholecystectomy, the nurse notices that the drainage from the T-tube is 600mL in 24 hours. Which is the appropriate action by the nurse? 1. Clamp the tube q2h for 30 minutes 2. Place the patient in a supine position 3. Assess drainage characteristics and notify the physician 4. Encourage an increase fluid intake

3 The T-tube may drain 500mL in the first 24 h and decreases steadily thereafter. If there is excessive drainage, the nurse should further assess the drainage to be able to describe it accurately and notify the physician immediately. Option 1 would be contraindicated and 2 and 4 are of no help Core issue: Knowledge of appropriate nursing action following notation of excessive T-tube drainage.

A client has a total gastrectomy. The nurse explains to the client the need for long-term injections of which of the following vitamins? 1. Thiamine 2. Folic Acid 3. Cyanocobalamin 4. Niacin

3 The loss of parietal cells that secrete intrinsic factor results in vitamin B12 (cyanocobalamin) deficiency post gastrectomy, because intrinsic factor is needed for absorption of vitamin B12. For this reason, clients require vitamin B12 injections for life. The other options identify other B-complex vitamins Core issue: Knowledge that gastric surgery results in loss of ability to produce intrinsic factor and subsequent vitamin B12 deficiency.

The nurse evaluates the client's understanding of ileostomy care. Which of the following statements indicates that discharge teaching has been effective? 1. "I should be able to resume weight lifting in 2 weeks." 2. "I can return to work in 2 weeks." 3. "I need to drink at least 3,000 mL a day of fluid." 4. "I will need to avoid getting my stoma wet while bathing."

3. To maintain an adequate fluid balance, the client needs to drink at least 3,000 mL/ day. Heavy lifting should be avoided; the physician will indicate when the client can participate in sports again. The client will not resume working as soon as 2 weeks after surgery. Water does not harm the stoma, so the client does not have to worry about getting it wet.

What teaching would you provide to a client with GERD?

Don't lie down for 2 hours after eating.

Cystic fibrosis, a genetic disorder characterized by pulmonary and pancreatic dysfunction, usually appears in young children but can also affect adults. If the pancreas was functioning correctly, where would the bile and pancreatic enzymes enter the GI system? a) Cecum b) Jejunum c) Duodenum d) Ileum

Duodenum Explanation: The duodenum, which is approximately 10 inches long, is the first region of the small intestine and the site where bile and pancreatic enzymes enter.

Stop the instillation and meausre vital signs

During the enema the client begins to complain of pain. THe nurse notes blood in the return fluid and rectal bleeding. The nurs's next action is to: A. Stop the instillation. B. Slow down the rate of instillation C. Stop the instillation and meausre vital signs D. Tell the client to breathe

What should you be looking for with a pt on TPN?

Hyperglycemia, infection, hyperalimentation, and effective nutrition.

The nurse is caring for a man who has experienced a spinal cord injury. Throughout his recovery, the client expects to gain control of his bowels. The nurse's best response to this client would be which of the following? a) "Over time, the nerve fibers will regrow new tracts, and you can have bowel movements again." b) "It is not going to happen. Your nerve cells are too damaged." c) "Wearing an undergarment will become more comfortable over time." d) "Having a bowel movement is a spinal reflex requiring intact nerve fibers. Yours are not intact."

"Having a bowel movement is a spinal reflex requiring intact nerve fibers. Yours are not intact." Explanation: The act of defecation is a spinal reflex involving the parasympathetic nerve fibers. Normally, the external anal sphincter is maintained in a state of tonic contraction. With a spinal cord injury, the client no longer has this nervous system control and is often incontinent.

A home care nurse is caring for a client with complaints of epigastric discomfort who is scheduled for a barium swallow. Which statement by the client indicates an understanding of the test? a) "I'll take a laxative to clear my bowels before the test." b) "I'll drink full liquids the day before the test." c) "I'll avoid eating or drinking anything 6 to 8 hours before the test." d) "There is no need for special preparation before the test."

"I'll avoid eating or drinking anything 6 to 8 hours before the test." Explanation: The client demonstrates understanding of a barium swallow when he states that he must refrain from eating or drinking for 6 to 8 hours before the test. No other preparation is needed. Before a lower GI series, the client should eat a low-residue or clear liquid diet for 2 days and take a potent laxative and an oral liquid preparation.

A client recently started a new medication to treat a suspected ulcer. She asks the nurse how this medicine is helping. Which of the following would be the best response by the nurse regarding anti-ulcerative medications? a) "This medication helps to digest food more slowly, allowing acid to attach to the food particles." b) "This medication reduces the acid secretion in your stomach." c) "The medication is allowing the acid to be eliminated more quickly in the stool." d) "This medication acts to reduce the volume of acid and foods that can enter the small intestine."

"This medication reduces the acid secretion in your stomach." Explanation: The stomach secretes a highly acidic fluid in response to food. The acid is derived from hydrochloric acid, which is secreted by the glands of the stomach. Ulcer medications inhibit acid secretion in the stomach or inhibit the action of histamine at the receptor sites.

A client who had a colonoscopy with removal of a polyp is being prepared for discharge. Which of the following would the nurse include in the discharge instructions? a) "You might experience some nausea and vomiting for a day or so. This is normal." b) "Call your physician if there is even slight bleeding with your first bowel movement." c) "You might feel some cramping and gas but these usually go away in about a day." d) "Be sure to eat high fiber foods when you get home to help you move your bowels."

"You might feel some cramping and gas but these usually go away in about a day." Explanation: After a colonoscopy, a client may experience mild cramping and flatulence which usually resolve within n 24 hours. If the client has a small growth or polyp removed, there may be a slight amount of bleeding that resolves on its own. The client should notify his physician if he experiences nausea, vomiting, fever, or excessive bleeding. The client also should avoid high-fat and high-fiber foods for at least 24 hours after the procedure.

Clear Liquid Diet

*Indications* ■ Clear liquid diet provided fluids and some electrolytes to prevent dehydration ■ Clear liquid diet is used as an initial feeding after complete bowel rest ■ Clear liquid diet is used initially to feed a malnourished person or a person who has not had any oral intake for some time ■ clear liquid diet is used for bowel preparation for surgery or tests, as well as post-operatively and in clients with fever, vomiting or diarrhea ■ Clear liquid diet is used in gastroenteritis or pancreatitis *Nursing consideration* ■ Clear liquids are deficient in energy (calories) and many *nutrients* ■ Clear liquid diet is easily digested and absorbed ■ Minimal residue is left in the gastrointestinal tract ■ Clients may find a clear liquid diet unappetizing and boring ■ As a transition diet, clear liquids are intended for short-term use ■ Clear liquids and foods that are relatively transparent to light and are liquid at body temperature are considered "clear liquids" such as water, boullon, clear broth, carbonated beverages, gelatin, hard candy, lemonade, popscicles, regular or decaffeinated coffee or tea. ■ By limiting the amount of caffeine intake, upset stomach and sleeplessness may be prevented ■ The client may consume salt and sugar ■ Dairy products and fruit juices with pulp are not clear liquids */!\* Monitor the client's hydration status by assessing intake and output, weight, monitoring for edema, and monitoring for signs of dehydration. */!\*

Soft diet

*Indications* ■ Used for clients who have difficulty chewing or swallowing ■ Used for clients who have ulcerations of the mouth or gums, oral surgery, broken jaw, plastic surgery of the head or neck, or dysphagia, or for the client who has had a stroke *Nursing considerations* ■ Clients with mouth sores should be served foods at cooler temperatures ■ Clients who have difficulty chewing and swallowing because of dry mouth can increase salivary flow by sucking on sour candy ■ Encourage the client to eat a variety of foods ■ Provide plenty of fluids with meals to ease chewing and swallowing of foods ■ Drinking fluids through a straw may be easier than drinking from a cup or glass ■ All foods and seasonings are permitted; however, liquid, chopped, or pureed foods or regular foods with a soft consistency are tolerated best ■ Foods that contain nuts or seeds, which easily can become trapped in the mouth and cause discomfort, should be avoided ■ Raw fruits and vegetables, fried foods, and whole grains should be avoided */!\* Consider the client's disease or illness and how it may impact nutritional status */!\*

Vitamins

*Vitamins* facilitate *metabolism* of proteins, fats, and carbohydrates and act as catalysts for metabolic function. Vitamins promote life and grown processes, and maintain and regulate body functions. *Fat-soluble* vitamins *A, D, E, K* can be stored in the body, so an excess can cause toxicity. *Water-soluble" vitamins *B, C, niacin, folic acid* are not stored in the body and can be excreted in the urine. Vitamin *A* maintains eyesight and epithelial linings. Vitamin *D* regulates blood calcium levels and the rate of deposit and reabsorption of calcium in bone. Vitamin *E* is an antioxidant that protects red blood cells and muscle tissue cells. Vitamin *K* acts as a catalyst for facilitating blood clotting factors, especially prothrombin, and bone development Vitamin *C* is an antioxidant that has immune functions, acts in the production of collagen, a vital component in wound healing, and in iron absorption *Food sources of fat-soluble vitamins* *A*: Liver, egg yolk, whole milk, green or orange vegetables, fruits *D*: Fortified milk, fish oils, cereals *E*: vegetable oils; green, leafy vegetables; cereals; apricots; apples and peaches *K*: green leafy vegetables; cauliflower and cabbage *Food sources of water-soluble vitamins* *Folic acid*: green leafy vegetables; liver, beef, and fish; legumes; grapefruit and oranges *Niacin*: meats, poultry, fish, beans, peanuts, grains *B1 (thiamine)*: pork, nuts, whole-grain cereals and legumes *B2 (riboflavin)*: milk, lean meats, fish, grains *B6 (pyridoxine)*: yeast, corn, meat, poultry, fish *B12 (cobalamin)*: meat, liver *C (ascorbic acid)*: citrus fruits, tomatoes, broccoli, cabbage

Anoscopy, proctoscopy, sigmoidoscopy

- Anoscopy: rigid scope to examine anal canal. pt knee-chest or lt side - Proctoscopy and sigmoidoscopy: flexible scope to examine rectum and sigmoid colon. pt lt side c rt leg bent - Pre: enemas (@ risk for fluid and electrolyte imbalances) - Post: monitor for bleeding

A client with ulcerative colitis is to take sulfasalazine (Azulfidine). Which of the following instructions should the nurse provide for the client about taking this medication at home? Select all that apply. 1. Drink enough fluids to maintain a urine output of at least 1,200- 1,500 mL per day. 2. Discontinue therapy if symptoms of acute intolerance develop and notify the health care provider. 3. Stop taking the medication if the urine turns orange-yellow. 4. Avoid activities that require alertness. 5. If dose is missed, skip and continue with the next dose.

1, 2, 4. Sulfasalazine may cause dizziness and the nurse should caution the client to avoid driving or other activities that require alertness until response to medication is known. If symptoms of acute intolerance (cramping, acute abdominal pain, bloody diarrhea, fever, headache, rash) occur, the client should discontinue therapy and notify the health care provider immediately. Fluid intake should be sufficient to maintain a urine output of at least 1,200- 1,500 mL daily to prevent crystalluria and stone formation. The nurse can also inform the client that this medication may cause orange-yellow discoloration of urine and skin, which is not significant and does not require the client to stop taking the medication. The nurse should instruct the client to take missed doses as soon as remembered unless it is almost time for the next dose.

The nurse is teaching a group of teenage boys who are on a baseball team about the risks of chewing tobacco. Which of the following should the nurse instruct the teenagers to report to their parents and physicians? Select all that apply. 1. Dysphagia. 2. Sensitive teeth. 3. Unexplained mouth pain. 4. Lump in the neck. 5. Decreased saliva. 6. White patch on the mucosa.

1, 3, 4, 6. Chewing tobacco has become a more common practice among teenagers. It is important that they understand that this increases their risk for oral cancer. They should be instructed to inspect their mouth frequently and report any observed lesions or other changes in the oral mucosa. Signs and symptoms that are potential indicators of oral cancer are dysphagia, unexplained mouth pain, a lump in the neck, and white patches on the mucosa (leukoplakia). Other indications may be a painless mouth ulcer, a reddened patch (erythroplasia), and rough patches on the mucosa. Sensitive teeth and decreased saliva are not associated with oral cancer.

The nurse is preparing a community presentation on oral cancer. Which of the following is a primary risk factor for oral cancer that the nurse should include in the presentation? 1. Use of alcohol. 2. Frequent use of mouthwash. 3. Lack of vitamin B12. 4. Lack of regular teeth cleaning by a dentist.

1. Chronic and excessive use of alcohol can lead to oral cancer. Smoking and use of smokeless tobacco are other significant risk factors. Additional risk factors include chronic irritation such as a broken tooth or ill-fitting dentures, poor dental hygiene, overexposure to sun (lip cancer), and syphilis. Use of mouthwash, lack of vitamin B12, and lack of regular teeth cleaning appointments have not been implicated as primary risk factors for oral cancer.

The nurse instructs the client on health maintenance activities to help control symptoms from her hiatal hernia. Which of the following statements would indicate that the client has understood the instructions? 1. "I'll avoid lying down after a meal." 2. "I can still enjoy my potato chips and cola at bedtime." 3. "I wish I didn't have to give up swimming." 4. "If I wear a girdle, I'll have more support for my stomach."

1. A client with a hiatal hernia should avoid the recumbent position immediately after meals to minimize gastric reflux. Bedtime snacks, as well as high-fat foods and carbonated beverages, should be avoided. Excessive vigorous exercise also should be avoided, especially after meals, but there is no reason why the client must give up swimming. Wearing tight, constrictive clothing such as a girdle can increase intra-abdominal pressure and thus lead to reflux of gastric juices.

A client with ulcerative colitis expresses serious concerns about her career as an attorney because of the effects of stress on ulcerative colitis. Which of the following nursing interventions will be most helpful to the client? 1. Review her current coping mechanisms and develop alternatives, if needed. 2. Suggest a less stressful career in which she would still use her education and experience. 3. Suggest that she ask her colleagues to help decrease her stress by giving her the easier cases. 4. Prepare family members for the fact that she will have to work part-time.

1. A client with ulcerative colitis need not curtail career goals. Self-care is the cornerstone of long-term management, and learning to cope with and modify stressors will enable the client to live with the disease. Giving up a desired career could discourage and even depress the client. Placing the responsibility for minimizing stressors at work in the hands of others leads to a feeling of loss of control and decreases the sense of responsibility needed for sound self-care. Working part-time rather than full-time is unnecessary.

The nurse is caring for a client who has just had an upper GI endoscopy. The client's vital signs must be taken every 30 minutes for 2 hours after the procedure. The nurse assigns an unlicensed nursing personnel (UAP) to take the vital signs. One hour later, the UAP reports the client, who was previously afebrile, has developed a temperature of 101.8 ° F (38.8 ° C). What should the nurse do in response to this reported assessment data? 1. Promptly assess the client for potential perforation. 2. Tell the assistant to change thermometers and retake the temperature. 3. Plan to give the client acetaminophen (Tylenol) to lower the temperature. 4. Ask the assistant to bathe the client with tepid water.

1. A sudden spike in temperature following an endoscopic procedure may indicate perforation of the GI tract. The nurse should promptly conduct a further assessment of the client, looking for further indicators of perforation, such as a sudden onset of acute upper abdominal pain; a rigid, boardlike abdomen; and developing signs of shock. Telling the assistant to change thermometers is not an appropriate action and only further delays the appropriate action of assessing the client. The nurse would not administer acetaminophen without further assessment of the client or without a physician's order; a suspected perforation would require that the client be placed on nothing-by-mouth status. Asking the assistant to bathe the client before any assessment by the nurse is inappropriate.

The nurse is obtaining a health history from a client who has a sliding hiatal hernia associated with reflux. The nurse should ask the client about the presence of which of the following symptoms? 1. Heartburn. 2. Jaundice. 3. Anorexia. 4. Stomatitis.

1. Heartburn, the most common symptom of a sliding hiatal hernia, results from reflux of gastric secretions into the esophagus. Regurgitation of gastric contents and dysphagia are other common symptoms. Jaundice, which results from a high concentration of bilirubin in the blood, is not associated with hiatal hernia. Anorexia is not a typical symptom of hiatal hernia. Stomatitis is inflammation of the mouth.

A client has returned from surgery during which her jaws were wired as treatment for a fractured mandible. The client is in stable condition. The nurse is instructing the unliscensed nursing personnel (UAP) on how to properly position the client. Which instructions about positioning would be appropriate for the nurse to give the UAP? 1. Keep the client in a side-lying position with the head slightly elevated. 2. Do not reposition the client without the assistance of a registered nurse. 3. The client can assume any position that is comfortable. 4. Keep the client's head elevated on two pillows at all times.

1. Immediately after surgery the client should be placed on the side with the head slightly elevated. This position helps facilitate removal of secretions and decreases the likelihood of aspiration should vomiting occur. A registered nurse does not need to be present to reposition the client, unless the client's condition warrants the presence of the nurse. Although it is important to elevate the head, there is no need to keep the client's head elevated on two pillows unless that position is comfortable for the client.

The physician prescribes metoclopramide hydrochloride (Reglan) for the client with hiatal hernia. The nurse plans to instruct the client that this drug is used in hiatal hernia therapy to accomplish which of the following objectives? 1. Increase tone of the esophageal sphincter. 2. Neutralize gastric secretions. 3. Delay gastric emptying. 4. Reduce secretion of digestive juices.

1. Metoclopramide hydrochloride (Reglan) increases esophageal sphincter tone and facilitates gastric emptying; both actions reduce the incidence of reflux. Other drugs, such as antacids or histamine receptor antagonists, may also be prescribed to help control reflux and esophagitis and to decrease or neutralize gastric secretions. Reglan is not effective in decreasing or neutralizing gastric secretions.

A client who has had ulcerative colitis for the past 5 years is admitted to the hospital with an exacerbation of the disease. Which of the following factors was most likely of greatest significance in causing an exacerbation of ulcerative colitis? 1. A demanding and stressful job. 2. Changing to a modified vegetarian diet. 3. Beginning a weight-training program. 4. Walking 2 miles every day.

1. Stressful and emotional events have been clearly linked to exacerbations of ulcerative colitis, although their role in the etiology of the disease has been disproved. A modified vegetarian diet or an exercise program is an unlikely cause of the exacerbation.

The nurse is developing standards of care for a client with gastroesophageal reflux disease and wants to review current evidence for practice. Which one of the following resources will provide the most helpful information? 1. A review in the Cochrane Library. 2. A literature search in a database, such as the Cumulative Index to Nursing and Allied Health Literature (CINHAL). 3. An online nursing textbook. 4. The online policy and procedure manual at the health care agency.

1. The Cochrane Library provides systematic reviews of health care interventions and will provide the best resource for evidence for nursing care. CINHAL offers key word searches to published articles in nursing and allied health literature, but not reviews. A nursing textbook has information about nursing care which may include evidence-based practices, but textbooks may not have the most up-to-date information. While the policy and procedure manual may be based on evidence-based practices, the most current practices will be found in evidence-based reviews of literature.

The nurse administers fat emulsion solution during TPN as ordered based on the understanding that this type of solution: 1. Provides essential fatty acids. 2. Provides extra carbohydrates. 3. Promotes effective metabolism of glucose. 4. Maintains a normal body weight.

1. The administration of fat emulsion solution provides additional calories and essential fatty acids to meet the body's energy needs. Fatty acids are lipids, not carbohydrates. Fatty acids do not aid in the metabolism of glucose. Although they are necessary for meeting the complete nutritional needs of the client, fatty acids do not necessarily help a client maintain normal body weight.

The client is scheduled to have an upper gastrointestinal tract series of x-rays. Following the x-rays, the nurse should instruct the client to: 1. Take a laxative. 2. Follow a clear liquid diet. 3. Administer an enema. 4. Take an antiemetic.

1. The client should take a laxative after an upper gastrointestinal series to stimulate a bowel movement. This examination involves the administration of barium, which must be promptly eliminated from the body because it may harden and cause an obstruction. A clear liquid diet would have no effect on stimulating removal of the barium. The client should not have nausea and an antiemetic would not be necessary; additionally, the antiemetic will decrease peristalsis and increase the likelihood of eliminating the barium. An enema would be ineffective because the barium is too high in the gastrointestinal tract.

TPN is ordered for a client with Crohn's disease. Which of the following indicate the TPN soloution is having an intended outcome? 1. There is increased cell nutrition. 2. The client does not have metabolic acidosis. 3. The client is hydrated. 4. The client is in a negative nitrogen balance.

1. The goal of TPN is to meet the client's nutritional needs. TPN is not used to treat metabolic acidosis; ketoacidosis can actually develop as a result of administering TPN. TPN is a hypertonic solution containing carbohydrates, amino acids, electrolytes, trace elements, and vitamins. It is not used to meet the hydration needs of clients. TPN is administered to provide a positive nitrogen balance.

Using a sliding-scale schedule, the nurse is preparing to administer an evening dose of regular insulin to a client who is receiving total parenteral nutrition (TPN). Which action is most appropriate for the nurse to take to determine the amount of insulin to give? 1. Base the dosage on the glucometer reading of the client's glucose level obtained immediately before administering the insulin. 2. Base the dosage on the fasting blood glucose level obtained earlier in the day. 3. Calculate the amount of TPN fluid the client has received since the last dose of insulin and adjust the dosage accordingly. 4. Assess the client's dietary intake for the evening meal and snack and adjust the dosage accordingly.

1. When using a sliding-scale insulin schedule, the nurse obtains a glucometer reading of the client's blood glucose level immediately before giving the insulin and bases the dosage on those findings. The fasting blood glucose level obtained earlier in the day is not relevant to an evening sliding-scale insulin dosage. The nurse cannot calculate insulin dosage by assessing the amount of TPN intake or dietary intake.

Name the 2 types of Pancreatitis

1. Acute: caused by alcohol or gallstones/gallbladder disease 2. Chronic: alcohol

Know the reason supplements are given including iron, folic acid, calcium, magnesium, and potassium. Be familiar with the populations that receive these supplements.

1. Adequate *calcium* intake throughout the lifespan decreases the likelihood of osteoporosis (a condition marked by porous bones). The recommended daily intake of 1200 mg for adults aged 50 years and older is difficult to achieve by diet alone. Deficiencies in iron, calcium, and vitamins A and C are common in Toddlers. Parents need to offer a variety of foods to provide these essential nutrients. It is almost impossible to consume the recommended amount of dietary iron and calcium during pregnancy, so supplements are commonly prescribed, as are supplements of folic acid and calcium. Many older adults may need supplements of calcium, vitamin D, and vitamin B12. 2. *Iron* deficiency causes anemia, the most common nutritional problem worldwide. Vitamin supplements may be prescribed for both breastfed and bottle-fed infants; iron supplements are usually prescribed at age 6 months. 3. *Magnesium* may decrease the risk of hypertension and coronary artery disease in women. Heavy use of alcohol depletes magnesium, B vitamins and folic acid. 4. *Folic acid* functions in cellular metabolism, Neurotransmitter synthesis, Cell division, DNA synthesis, Hemoglobin formation. U.S. Preventive Services Task Force offi cials recommend that women capable of becoming pregnant should take a daily folic acid supplement of 0.4 mg to 0.8 mg (400 to 800 micrograms) to lower the risk for neural tube defects in the fetus (USPSTF, 2009). 5. *Potassium* supplementation is indicated for patients with renal tubular acidosis, gastrointestinal losses from diarrhea or vomiting, 1 or 2 aldosteronism, Cushing's Syndrome or exogenous corticosteroid use. 6. *Vitamin C*. Up to 1/2 of this antioxidant is lost in the cooking water of boiled vegetables. Keeping foods hot longer than 2 hours results in even further loss. Oral contraceptive use lowers the serum level of vitamin C and several B vitamins. Women with marginal nutrient intake may need vitamin supplements. Smokers use vitamin C faster than nonsmokers. Even children exposed to second-hand smoke tend to have lower plasma levels of ascorbate than unexposed children. If the person cannot quit smoking, a vitamin C supplement may help compensate.

Treatment for Peptic Ulcers (medications)

1. Antacids (liquid to coat the stomach) -Take when stomach is empty and at bedtime - when the stomach is empty acid can get on ulcer...take antacid to protect ulcer 2. Proton Pump Inhibitors (decrease acid secretions -Omeprazole (Prilosec), Lansoprazole (Prevacid), Pantoprazole (Protonix), Esomeprazole (Nexium) 3. H2 antagonist -Ranitidine (Zantac), Famotidine (Pepcid) -GI Cocktail (donnatal, viscous lidocaine, Mylanta II) 4. Antibiotics for H. Pylori -Clarithromycin (Biaxin), Amoxicillin (Amoxil), Tetracycline (Panmycin), Metronidazole (Flagyl) 5. Sucrafata (Carafate): forms a barrier over wound so acid can't get on the ulcer

Medications for Crohn's or UC

1. Antidiarrheals -only given with mildly symptomatic UC; does not work well in severe cases 2. Antibiotics 3. Steroids (decrease inflammation)

Name the 2 parts and functions of the pancreas

1. Endocrine: insulin 2. Exocrine: digestive enzymes

The nurse is assigning clients for the evening shift. Which of the following clients are appropriate for the nurse to assign to a licensed practical nurse to provide client care? Select all that apply. 1. A client with Crohn's disease who is receiving total parenteral nutrition (TPN). 2. A client who underwent inguinal hernia repair surgery 3 hours ago. 3. A client with an intestinal obstruction who needs a Cantor tube inserted. 4. A client with diverticulitis who needs teaching about his take-home medications. 5. A client who is experiencing an exacerbation of his ulcerative colitis.

2, 5. The nurse should consider client needs and scope of practice when assigning staff to provide care. The client who is recovering from inguinal hernia repair surgery and the client who is experiencing an exacerbation of his ulcerative colitis are appropriate clients to assign to a licensed practical nurse as the care they require fall within the scope of practice for a licensed practical nurse. It is not within the scope of practice for the licensed practical nurse to administer TPN, insert nasoenteric tubes, or provide client teaching related to medications.

Which of the following should be a priority focus of care for a client experiencing an exacerbation of Crohn's disease? 1. Encouraging regular ambulation. 2. Promoting bowel rest. 3. Maintaining current weight. 4. Decreasing episodes of rectal bleeding.

2. A priority goal of care during an acute exacerbation of Crohn's disease is to promote bowel rest. This is accomplished through decreasing activity, encouraging rest, and initially placing client on nothing-by-mouth status while maintaining nutritional needs parenterally. Regular ambulation is important, but the priority is bowel rest. The client will probably lose some weight during the acute phase of the illness. Diarrhea is nonbloody in Crohn's disease, and episodes of rectal bleeding are not expected.

The nurse finds a client who has been diagnosed with a peptic ulcer surrounded by papers from his briefcase and arguing on the telephone with a coworker. The nurse's response to observing these actions should be based on knowledge that: 1. Involvement with his job will keep the client from becoming bored. 2. A relaxed environment will promote ulcer healing. 3. Not keeping up with his job will increase the client's stress level. 4. Setting limits on the client's behavior is an important nursing responsibility.

2. A relaxed environment is an essential component of ulcer healing. Nurses can help clients understand the importance of relaxation and explore with them ways to balance work and family demands to promote healing. Being involved with his work may prevent boredom; however, this client is upset and argumentative. Not keeping up with his job will probably increase the client's stress level, but the nurse's response is best if it is based on the fact that a relaxed environment is an essential component of ulcer healing. Nurses cannot set limits on a client's behavior; clients must make the decision to make lifestyle changes.

The client with ulcerative colitis is following orders for bed rest with bathroom privileges. When evaluating the effectiveness of this level of activity, the nurse should determine if the client has: 1. Conserved energy. 2. Reduced intestinal peristalsis. 3. Obtained needed rest. 4. Minimized stress.

2. Although modified bed rest does help conserve energy and promotes comfort, its primary purpose in this case is to help reduce the hypermotility of the colon. Remaining on bed rest does not by itself reduce stress, and if the client is having stress, the nurse can plan with the client to use strategies that will help the client manage the stress.

Which of the following factors would most likely contribute to the development of a client's hiatal hernia? 1. Having a sedentary desk job. 2. Being 5 feet, 3 inches tall and weighing 190 lb. 3. Using laxatives frequently. 4. Being 40 years old.

2. Any factor that increases intra-abdominal pressure, such as obesity, can contribute to the development of hiatal hernia. Other factors include abdominal straining, frequent heavy lifting, and pregnancy. Hiatal hernia is also associated with older age and occurs in women more frequently than in men. Having a sedentary desk job, using laxatives frequently, or being 40 years old is not likely to be a contributing factor in development of a hiatal hernia.

In developing a teaching plan for the client with a hiatal hernia, the nurse's assessment of which work-related factors would be most useful? 1. Number and length of breaks. 2. Body mechanics used in lifting. 3. Temperature in the work area. 4. Cleaning solvents used.

2. Bending, especially after eating, can cause gastroesophageal reflux. Lifting heavy objects increases intra-abdominal pressure. Assessing the client's lifting techniques enables the nurse to evaluate the client's knowledge of factors contributing to hiatal hernia and how to prevent complications. Number and length of breaks, temperature in the work area, and cleaning solvents used are not directly related to treatment of hiatal hernia.

Bethanechol (Urecholine) has been ordered for a client with gastroesophageal reflux disease (GERD). The nurse should assess the client for which of the following adverse effects? 1. Constipation. 2. Urinary urgency. 3. Hypertension. 4. Dry oral mucosa.

2. Bethanechol (Urecholine), a cholinergic drug, may be used in GERD to increase lower esophageal sphincter pressure and facilitate gastric emptying. Cholinergic adverse effects may include urinary urgency, diarrhea, abdominal cramping, hypotension, and increased salivation. To avoid these adverse effects, the client should be closely monitored to establish the minimum effective dose.

The nurse finds the client who has had an ileostomy crying. The client explains to the nurse, "I'm upset because I know I won't be able to have children now that I have an ileostomy." Which of the following would be the best response for the nurse? 1. "Many women with ileostomies decide to adopt. Why don't you consider that option?" 2. "Having an ileostomy does not necessarily mean that you can't bear children. Let's talk about your concerns." 3. "I can understand your reasons for being upset. Having children must be important to you." 4. "I'm sure you will adjust to this situation with time. Try not to be too upset."

2. The fact that the client has an ileostomy does not necessarily mean that she cannot get pregnant and bear children. It may be recommended, however, that the number of pregnancies be limited. Women of childbearing age should be encouraged to discuss their concerns with their physician. Discussing their concerns about sexual functioning and pregnancy will help decrease fears and anxiety. Empathizing or telling the woman that she can adopt does not address her concerns. Her current fears may be based on erroneous understanding. Telling the client that she will adjust to the situation ignores her concerns.

Which of the following instructions should the nurse include in the teaching plan for a client who is experiencing gastroesophageal reflux disease (GERD)? 1. Limit caffeine intake to two cups of coffee per day. 2. Do not lie down for 2 hours after eating. 3. Follow a low-protein diet. 4. Take medications with milk to decrease irritation.

2. The nurse should instruct the client to not lie down for about 2 hours after eating to prevent reflux. Caffeinated beverages decrease pressure in the lower esophageal sphincter and milk increases gastric acid secretion, so these beverages should be avoided. The client is encouraged to follow a high-protein, low-fat diet, and avoid foods that are irritating.

A client who was in a motor vehicle accident has a fractured mandible. Surgery has been performed to immobilize the injury by wiring the jaw. In the immediate postoperative phase, the nurse should: 1. Prevent nausea and vomiting. 2. Maintain a patent airway. 3. Provide frequent oral hygiene. 4. Establish a way for the client to communicate.

2. The priority of care in the immediate postoperative phase is to maintain a patent airway. The nurse should observe the client carefully for signs of respiratory distress. If the client becomes nauseated, antiemetics should be administered to decrease the chance of vomiting with obstruction of the airway and aspiration of vomitus. Providing frequent oral hygiene and an alternative means of communication are important aspects of nursing care, but maintaining a patent airway is most important.

Which of the following adverse effects would the nurse expect the client to exhibit in the event of too rapid an infusion of TPN solution? 1. Negative nitrogen balance. 2. Circulatory overload. 3. Hypoglycemia. 4. Hypokalemia.

2. Too rapid infusion of a TPN solution can lead to circulatory overload. The client should be assessed carefully for indications of excessive fluid volume. A negative nitrogen balance occurs in nutritionally depleted individuals, not when TPN fluids are administered in excess. When TPN is administered too rapidly the client is at risk for receiving an excess of dextrose and electrolytes. Therefore, the client is at risk for hyperglycemia and hyperkalemia.

The nurse has been assigned to provide care for four clients at the beginning of the day shift. In what order should the nurse assess these clients? 1. The client awaiting hiatal hernia repair at 11 am. 2. A client with suspected gastric cancer who is on nothing-by-mouth (NPO) status for tests. 3. A client with peptic ulcer disease experiencing sudden onset of acute stomach pain. 4. A client who is requesting pain medication 2 days after surgery to repair a fractured jaw.

3, 4, 2, 1 The client with peptic ulcer disease who is experiencing a sudden onset of acute stomach pain should be assessed first by the nurse. The sudden onset of stomach pain could be indicative of a perforated ulcer, which would require immediate medical attention. It is also important for the nurse to thoroughly assess the nature of the client's pain. The client with the fractured jaw is experiencing pain and should be assessed next. The nurse should then assess the client who is NPO for tests to ensure NPO status and comfort. Last, the nurse can assess the client before surgery.

The nurse notes that the sterile, occlusive dressing on the central catheter insertion site of a client receiving total parenteral nutrition (TPN) is moist. The client is breathing easily with no abnormal breath sounds. The nurse should do the following in order of what priority from first to last? 1. Change dressing per institutional policy. 2. Culture drainage at insertion site. 3. Notify physician. 4. Position rolled towel under client's back, parallel to the spine.

3, 4, 2, 1. A potential complication of receiving TPN is leakage or catheter puncture; notify the physician immediately and prepare for changing of the catheter. If pneumothorax is suspected, position a rolled towel under the client's back. If there is drainage at the insertion site, culture the drainage and change the dressing using sterile technique.

When obtaining a nursing history on a client with a suspected gastric ulcer, which signs and symptoms should the nurse expect to assess? Select all that apply. 1. Epigastric pain at night. 2. Relief of epigastric pain after eating. 3. Vomiting. 4. Weight loss. 5. Melena.

3, 4, 5. Vomiting and weight loss are common with gastric ulcers. The client may also have blood in the stools (melena) from gastric bleeding. Clients with a gastric ulcer are most likely to complain of a burning epigastric pain that occurs about 1 hour after eating. Eating frequently aggravates the pain. Clients with duodenal ulcers are more likely to complain about pain that occurs during the night and is frequently relieved by eating.

What should you teach your patient about cholecystectomy via laparoscopy?

3-4 cm incisions, inflate abd with gas, take out gallbladder, body will absorb gas, shoulder and chest pain is normal (don't worry about MI complaints).

The client asks the nurse, "Is it really possible to lead a normal life with an ileostomy?" Which action by the nurse would be the most effective to address this question? 1. Have the client talk with a member of the clergy about these concerns. 2. Tell the client to worry about those concerns after surgery. 3. Arrange for a person with an ostomy to visit the client preoperatively. 4. Notify the surgeon of the client's question.

3. If the client agrees, having a visit by a person who has successfully adjusted to living with an ileostomy would be the most helpful measure. This would let the client actually see that typical activities of daily living can be pursued postoperatively. Someone who has felt some of the same concerns can answer the client's questions. A visit from the clergy may be helpful to some clients but would not provide this client with the information sought. Disregarding the client's concerns is not helpful. Although the physician should know about the client's concerns, this in itself will not reassure the client about life after an ileostomy.

The nurse should instruct the client with an ileostomy to report which of the following signs and symptoms immediately? 1. Passage of liquid stool from the stoma. 2. Occasional presence of undigested food in the effluent. 3. Absence of drainage from the ileostomy for 6 or more hours. 4. Temperature of 99.8 ° F (37.7 ° C).

3. Any sudden decrease in drainage or onset of severe abdominal pain should be reported to the physician immediately because it could mean that an obstruction has developed. The ileostomy drains liquid stool at frequent intervals throughout the day. Undigested food may be present at times. A temperature of 99.8 ° F is not necessarily abnormal or a cause for concern.

Cimetidine (Tagamet) may also be used to treat hiatal hernia. The nurse should understand that this drug is used to prevent which of the following? 1. Esophageal reflux. 2. Dysphagia. 3. Esophagitis. 4. Ulcer formation.

3. Cimetidine (Tagamet) is a histamine receptor antagonist that decreases the quantity of gastric secretions. It may be used in hiatal hernia therapy to prevent or treat the esophagitis and heartburn associated with reflux. Cimetidine is not used to prevent reflux, dysphagia, or ulcer development.

The nurse is preparing to teach a client with a peptic ulcer about the diet that should be followed after discharge. The nurse should explain that the diet will most likely consist of which of the following? 1. Bland foods. 2. High-protein foods. 3. Any foods that are tolerated. 4. Large amounts of milk.

3. Diet therapy for ulcer disease is a controversial issue. There is no scientific evidence that diet therapy promotes healing. Most clients are instructed to follow a diet that they can tolerate. There is no need for the client to ingest only a bland or high-protein diet. Milk may be included in the diet, but it is not recommended in excessive amounts.

A client's ulcerative colitis signs and symptoms have been present for longer than 1 week. The nurse should assess the client for signs and symptoms of which of the following complications? 1. Heart failure. 2. Deep vein thrombosis. 3. Hypokalemia. 4. Hypocalcemia.

3. Excessive diarrhea causes significant depletion of the body's stores of sodium and potassium as well as fluid. The client should be closely monitored for hypokalemia and hyponatremia. Ulcerative colitis does not place the client at risk for heart failure, deep vein thrombosis, or hypocalcemia.

A client who has had her jaws wired begins to vomit. What should be the nurse's first action? 1. Insert a nasogastric (NG) tube and connect it to suction. 2. Use wire cutters to cut the wire. 3. Suction the client's airway as needed. 4. Administer an antiemetic intravenously.

3. The nurse's first action is to clear the client's airway as necessary. Inserting an NG tube or administering an antiemetic may prevent future vomiting episodes, but these procedures are not helpful when the client is actually vomiting. Cutting the wires is done only as a last resort or in case of respiratory or cardiac arrest.

A client has been taking aluminum hydroxide (Amphojel) 30 mL six times per day at home to treat his peptic ulcer. He tells the nurse that he has been unable to have a bowel movement for 3 days. Based on this information, the nurse would determine that which of the following is the most likely cause of the client's constipation? 1. The client has not been including enough fiber in his diet. 2. The client needs to increase his daily exercise. 3. The client is experiencing an adverse effect of the aluminum hydroxide. 4. The client has developed a gastrointestinal obstruction.

3. It is most likely that the client is experiencing an adverse effect of the antacid. Antacids with aluminum salt products, such as aluminum hydroxide, form insoluble salts in the body. These precipitate and accumulate in the intestines, causing constipation. Increasing dietary fiber intake or daily exercise may be a beneficial lifestyle change for the client but is not likely to relieve the constipation caused by the aluminum hydroxide. Constipation, in isolation from other symptoms, is not a sign of a bowel obstruction.

A client is scheduled for an ileostomy. Which of the following interventions would be most helpful in preparing the client psychologically for the surgery? 1. Include family members in preoperative teaching sessions. 2. Encourage the client to ask questions about managing an ileostomy. 3. Provide a brief, thorough explanation of all preoperative and postoperative procedures. 4. Invite a member of the ostomy association to visit the client.

3. Providing explanations of preoperative and postoperative procedures helps the client prepare and understand what to expect. It also provides an opportunity for the client to share concerns. Including family members in the teaching sessions is beneficial but does not focus on the client's psychological preparation. Encouraging the client to ask questions about managing the ileostomy may be rushing the client psychologically into accepting the change in body image and function. The client may need time to first handle the stress of surgery and then observe the care of the ileostomy by others before it is appropriate to begin discussing self-management. The nurse should gently explore whether the client is ready to ask questions about management throughout the hospitalization. The client should have the opportunity to express concerns and to agree to an ostomy association visitor before an invitation is extended.

A client is to take one daily dose of ranitidine (Zantac) at home to treat her peptic ulcer. The client understands proper drug administration of ranitidine when she says that she will take the drug at which of the following times? 1. Before meals. 2. With meals. 3. At bedtime. 4. When pain occurs.

3. Ranitidine blocks secretion of hydrochloric acid. Clients who take only one daily dose of ranitidine are usually advised to take it at bedtime to inhibit nocturnal secretion of acid. Clients who take the drug twice a day are advised to take it in the morning and at bedtime. It is not necessary to take the drug before meals. The client should take the drug regularly, not just when pain occurs.

Which of the following nursing interventions would most likely promote self-care behaviors in the client with a hiatal hernia? 1. Introduce the client to other people who are successfully managing their care. 2. Include the client's daughter in the teaching so that she can help implement the plan. 3. Ask the client to identify other situations in which he demonstrated responsibility for himself. 4. Reassure the client that he will be able to implement all aspects of the plan successfully.

3. Self-responsibility is the key to individual health maintenance. Using examples of situations in which the client has demonstrated self-responsibility can be reinforcing and supporting. The client has ultimate responsibility for his personal health habits. Meeting other people who are managing their care and involving family members can be helpful, but individual motivation is more important. Reassurance can be helpful but is less important than individualization of care.

During the assessment of a client's mouth, the nurse notes the absence of saliva. The client has pain in the area of the ear. The client has been nothing-by-mouth (NPO) for several days because of the insertion of a nasogastric tube. Based on these findings, the nurse suspects that the client may be developing which of the following mouth conditions? 1. Stomatitis. 2. Oral candidiasis. 3. Parotitis. 4. Gingivitis.

3. The lack of saliva, pain near the area of the ear, and the prolonged NPO status of the client should lead the nurse to suspect the development of parotitis, or inflammation of the parotid gland. Parotitis usually develops in cases of dehydration combined with poor oral hygiene or when clients have been NPO for an extended period. Preventive measures include the use of sugarless hard candy or gum to stimulate saliva production, adequate hydration, and frequent mouth care. Stomatitis (inflammation of the mouth) produces excessive salivation and a sore mouth. Oral candidiasis (thrush) causes bluish white mouth lesions. Gingivitis can be recognized by the inflamed gingiva and bleeding that occur during toothbrushing.

The client has been taking magnesium hydroxide (milk of magnesia) at home in an attempt to control hiatal hernia symptoms. The nurse should assess the client for which of the following conditions most commonly associated with the ongoing use of magnesium-based antacids? 1. Anorexia. 2. Weight gain. 3. Diarrhea. 4. Constipation.

3. The magnesium salts in magnesium hydroxide are related to those found in laxatives and may cause diarrhea. Aluminum salt products can cause constipation. Many clients find that a combination product is required to maintain normal bowel elimination. The use of magnesium hydroxide does not cause anorexia or weight gain.

The nurse is changing the subclavian dressing of a client who is receiving total parenteral nutrition. When assessing the catheter insertion site, the nurse notes the presence of yellow drainage from around the sutures that are anchoring the catheter. Which action should the nurse take first? 1. Clean the insertion site and redress the area. 2. Document assessment findings in the client's chart. 3. Obtain a culture specimen of the drainage. 4. Notify the physician.

3. The nurse should first obtain a culture specimen. The presence of drainage is a potential indication of an infection and the catheter may need to be removed. A culture specimen should be obtained and sent for analysis so that treatment can be promptly initiated. Since removing the catheter will be required in the presence of an infection, the nurse would not clean and redress the area. After the culture report is obtained, the nurse should notify the physician and document all assessments and client care activities in the client's record.

A client who is experiencing an exacerbation of ulcerative colitis is receiving I.V. fluids that are to be infused at 125 mL/ hour. The I.V. tubing delivers 15 gtt/ mL. How quickly should the nurse infuse the fluids in drops per minute to infuse the fluids at the prescribed rate? ________________________ gtt/ minute.

31 gtt/ minute To administer I.V. fluids at 125 mL/ hour using tubing that has a drip factor of 15 gtt/ mL, the nurse should use the following formula: 125 mL/ 60 minutes × 15 gtt/ 1 mL = 31 gtt/ minute.

A client is taking an antacid for treatment of a peptic ulcer. Which of the following statements best indicates that the client understands how to correctly take the antacid? 1. "I should take my antacid before I take my other medications." 2. "I need to decrease my intake of fluids so that I don't dilute the effects of my antacid." 3. "My antacid will be most effective if I take it whenever I experience stomach pains." 4. "It is best for me to take my antacid 1 to 3 hours after meals."

4. Antacids are most effective if taken 1 to 3 hours after meals and at bedtime. When an antacid is taken on an empty stomach, the duration of the drug's action is greatly decreased. Taking antacids 1 to 3 hours after a meal lengthens the duration of action, thus increasing the therapeutic action of the drug. Antacids should be administered about 2 hours after other medications to decrease the chance of drug interactions. It is not necessary to decrease fluid intake when taking antacids. If antacids are taken more frequently than recommended, the likelihood of developing adverse effects increases. Therefore, the client should not take antacids as often as desired to control pain.

A client who has a history of a mitral valve prolapse tells the nurse that she is scheduled to get her teeth cleaned. Which of the following replies by the nurse is most appropriate? 1. "The physician will need to reevaluate the status of your heart condition before your dental appointment." 2. "Be sure to remind your dentist that you have a heart condition." 3. "It is important for you to care for your teeth because your heart condition makes you more susceptible to developing oral infections." 4. "We will prescribe a prophylactic antibiotic for you to take before getting your teeth cleaned."

4. Clients who are at risk for developing infective endocarditis due to cardiac conditions such as mitral valve prolapse must take prophylactic antibiotics before any dental procedure that may cause bleeding. The client is not more susceptible to developing oral infections. Rather, the client is more susceptible to developing endocarditis that results from oral bacteria that enter the circulation during the dental procedure. The physician does not necessarily need to re-evaluate the heart condition of a client who is stable, but antibiotics must be prescribed. It is not enough to simply remind the dentist about the heart condition.

The client with gastroesophageal reflux disease (GERD) complains of a chronic cough. The nurse understands that in a client with GERD this symptom may be indicative of which of the following conditions? 1. Development of laryngeal cancer. 2. Irritation of the esophagus. 3. Esophageal scar tissue formation. 4. Aspiration of gastric contents.

4. Clients with GERD can develop pulmonary symptoms, such as coughing, wheezing, and dyspnea, that are caused by the aspiration of gastric contents. GERD does not predispose the client to the development of laryngeal cancer. Irritation of the esophagus and esophageal scar tissue formation can develop as a result of GERD. However, GERD is more likely to cause painful and difficult swallowing.

Which of the following interventions is most appropriate for a client who has stomatitis? 1. Drinking hot tea at frequent intervals. 2. Gargling with antiseptic mouthwash. 3. Using an electric toothbrush. 4. Eating a soft, bland diet.

4. Clients with stomatitis (inflammation of the mouth) have significant discomfort, which impacts their ability to eat and drink. They will be most comfortable eating soft, bland foods, and avoiding temperature extremes in their food and liquids. Gargling with an antiseptic mouthwash will be irritating to the mucosa. Mouth care should include gentle brushing with a soft toothbrush and flossing.

A client is receiving Total Parenteral Nutrition (TPN) soulution. The nurse should assess a client's ability to metabolize the TPN solution adequately by monitoring the client for which of the following signs? 1. Tachycardia. 2. Hypertension. 3. Elevated blood urea nitrogen concentration. 4. Hyperglycemia.

4. During TPN administration, the client should be monitored regularly for hyperglycemia. The client may require small amounts of insulin to improve glucose metabolism. The client should also be observed for signs and symptoms of hypoglycemia, which may occur if the body overproduces insulin in response to a high glucose intake or if too much insulin is administered to help improve glucose metabolism. Tachycardia or hypertension is not indicative of the client's ability to metabolize the solution. An elevated blood urea nitrogen concentration is indicative of renal status and fluid balance.

A client with peptic ulcer disease is taking ranitidine (Zantac). What is the expected outcome of this drug? 1. Heal the ulcer. 2. Protect the ulcer surface from acids. 3. Reduce acid concentration. 4. Limit gastric acid secretion.

4. Histamine-2 (H2) receptor antagonists, such as ranitidine, reduce gastric acid secretion. Antisecretory, or proton-pump inhibitors, such as omeprazole (Prilosec), help ulcers heal quickly in 4 to 8 weeks. Cytoprotective drugs, such as sucralfate (Carafate), protect the ulcer surface against acid, bile, and pepsin. Antacids reduce acid concentration and help reduce symptoms.

A client has entered a smoking cessation program to quit a two-pack-a-day cigarette habit. He tells the nurse that he has not smoked a cigarette for 3 weeks, but is afraid he is going to "slip up" and smoke because of current job pressures. What would be the most appropriate reply for the nurse to make in response to the client's comments? 1. "Don't worry about it. Everybody has difficulty quitting smoking, and you should expect to as well." 2. "If you increase your self-control, I am sure you will be able to avoid smoking." 3. "Try taking a couple of days of vacation to relieve the stress of your job." 4. "It is good that you can talk about your concerns. Try calling a friend when you want to smoke."

4. It is important for individuals who are engaged in smoking cessation efforts to feel comfortable with sharing their fears of failure with others and seeking support. Although fewer than 5% of smokers successfully quit on their first attempt, it is not helpful to tell a client that he should anticipate failure. Telling the client to exercise more self-control does not provide him with support. Taking a vacation to avoid job pressures does not address the issue of fearing he will smoke a cigarette when in a stressful situation.

A client who has ulcerative colitis says to the nurse, "I can't take this anymore! I'm constantly in pain, and I can't leave my room because I need to stay by the toilet. I don't know how to deal with this." Based on these comments, an appropriate nursing diagnosis for this client would be: 1. Impaired physical mobility related to fatigue. 2. Disturbed thought processes related to pain. 3. Social isolation related to chronic fatigue. 4. Ineffective coping related to chronic abdominal pain.

4. It is not uncommon for clients with ulcerative colitis to become apprehensive and upset about the frequency of stools and the presence of abdominal cramping. During these acute exacerbations, clients need emotional support and encouragement to verbalize their feelings about their chronic health concerns and assistance in developing effective coping methods. The client has not expressed feelings of fatigue or isolation or demonstrated disturbed thought processes.

The nurse should instruct the client to avoid which of the following drugs while taking metoclopramide hydrochloride (Reglan)? 1. Antacids. 2. Antihypertensives. 3. Anticoagulants. 4. Alcohol.

4. Metoclopramide hydrochloride (Reglan) can cause sedation. Alcohol and other central nervous system depressants add to this sedation. A client who is taking this drug should be cautioned to avoid driving or performing other hazardous activities for a few hours after taking the drug. Clients may take antacids, antihypertensives, and anticoagulants while on metoclopramide.

A client with a well-managed ilesostomy calls the nurse to report the sudden onset of abdominal cramps, vomiting, and watery discharge from the ileostomy. The nurse should: 1. Tell the client to take an antiemetic. 2. Encourage the client to increase fluid intake to 3 L/ day to replace fluid lost through vomiting. 3. Instruct the client to take 30 mL of milk of magnesia to stimulate a bowel movement. 4. Advise the client to notify the physcian.

4. Sudden onset of abdominal cramps, vomiting, and watery discharge with no stool from an ileostomy are likely indications of an obstruction. It is imperative that the physician examine the client immediately. Although the client is vomiting, the client should not take an antiemetic until the physician has examined the client. If an obstruction is present, ingesting fluids or taking milk of magnesia will increase the severity of symptoms. Oral intake is avoided when a bowel obstruction is suspected.

Blood urea nitrogen (BUN) norms

5-18 mg/dL (children) 8-21 mg/dL (age 14-adult) 10-31 mg/dL (adult older than 90)

What should be the TPN rate?

40-50 ml/hr, then increase by 25 ml/hr every 6 hours.

A nurse teaches a client experiencing heartburn to take 1 ½ oz of Maalox when symptoms appear. How many milliliters should the client take? ________________________ mL. .

45 mL

A client with liver and renal failure has severe ascites. On initial shift rounds, his primary nurse finds his indwelling urinary catheter collection bag too full to store more urine. The nurse empties more than 2,000 ml from the collection bag. One hour later, she finds the collection bag full again. The nurse notifies the physician, who suspects that a bladder rupture is allowing the drainage of peritoneal fluid. The physician orders a urinalysis to be obtained immediately. The presence of which substance is considered abnormal? a) Albumin b) Chloride c) Urobilinogen d) Creatinine

A) Albumin Albumin is an abnormal finding in a routine urine specimen. Ascites present in liver failure contain albumin; therefore, if the bladder ruptured, ascites containing albumin would drain from the indwelling urinary catheter because the catheter is no longer contained in the bladder. Creatinine, urobilinogen, and chloride are normally found in urine.

When caring for a client with an acute exacerbation of a peptic ulcer, the nurse finds the client doubled up in bed with severe pain to his right shoulder. The intial appropriate action by the nurse is to a) Assess the client's abdomen and vital signs. b) Irrigate the client's NG tube. c) Place the client in the high-Fowler's position. d) Notify the health care provider.

A) Assess the client's abdomen and vital signs Signs and symptoms of perforation includes sudden, severe upper abdominal pain (persisting and increasing in intensity); pain may be referred to the shoulders, especially the right shoulder, because of irritation of the phrenic nerve in the diaphragm. The nurse should assess the vital signs and abdomen prior to notifying the physician. Irrigation of the NG tube should not be performed because the additional fluid may be spilled into the peritoneal cavity, and the client should be placed in a position of comfort, usually on the side with the head slightly elevated.

The nurse is to obtain a stool specimen from a client who reported that he is taking iron supplements. The nurse would expect the stool to be which color? a) Black b) Red c) Dark brown d) Green

A) Black Ingestion of iron can cause the stool to turn black. Meat protein causes stool to appear dark brown. Ingestion of large amounts of spinach may turn stool green while ingestion of carrots and beets may cause stool to turn red.

A client is admitted to the health care facility with a diagnosis of a bleeding gastric ulcer. The nurse expects this client's stools to be: a) black and tarry. b) coffee-ground-like. c) bright red. d) clay-colored.

A) Black and tarry Black, tarry stools are a sign of bleeding high in the GI tract, as from a gastric ulcer, and result from the action of digestive enzymes on the blood. Vomitus associated with upper GI tract bleeding commonly is described as coffee-ground-like. Clay-colored stools are associated with biliary obstruction. Bright red stools indicate lower GI tract bleeding.

During removal of a fecal impaction, which of the following could occur Because of vaginal stimulation? A) Bradycardia B)Atelectasid C) Tachycardia D) Cardiac tamponade

A) Bradycardia Removing a fecal impaction manually may result in stimulation of the vaginal nerve and resulting bradycardia

A nurse is reviewing the history and physical of a client admitted for a hemorrhoidectomy. Which predisposing condition does the nurse expect to see? a) Constipation b) Hypoglycemia c) Lactic acidosis d) Hyperkalemia

A) Constipation Orthostatic hypertension and other conditions associated with persistently high intra-abdominal pressure (such as pregnancy) can lead to hemorrhoids. The passing of hard stools, not diarrhea, can aggravate hemorrhoids. Diverticulosis has no relationship to hemorrhoids. Rectal bleeding is a symptom of hemorrhoids, not a predisposing condition.

Which of the following is a true statement about the effects of medication on bowel illumination? A) Diarrhea commonly occurs with amoxicillin clavulanate use B) Anticoagulants cause a white discoloration of the stool C) Narcotic analgesics increased Gastrointestinal mobility D) Iron salts in pair digestion and cause a green store

A) Diarrhea commonly occurs with amoxicillin clavulanate use Anticoagulants may result in the store having a pink to red to black appearance, whereas iron salts also cause a black stool. Narcotic analgesics decrease gastric mobility.

Which of the following is the primary symptom of achalasia? a) Difficulty swallowing b) Pulmonary symptoms c) Chest pain d) Heartburn

A) Difficulty swallowing The primary symptom of achalasia is difficulty in swallowing both liquids and solids. The patient may also report chest pain and heartburn that may or may not be associated with eating. Secondary pulmonary complications may result from aspiration of gastric contents.

Which of the following would be a common nursing diagnosis for patient with an ileostomy? A) Disturbed body image B) Constipation C) Delayed growth and development D) Excess fluid volume

A) Disturbed body image Constipation does not occur with the Ileostomy because the drainage is liquid. Growth and development are not affected by the formation of an Ileostomy. Excess fluid Byam is unlikely to occur because the drainage is liquid and probably continual

The client cannot tolerate oral feedings due to an intestinal obstruction and is NPO. A central line has been inserted, and the client is being started on parenteral nutrition (PN). The nurse performs the following actions while the client receives PN (select all that apply): a) Document intake and output. b) Use clean technique for all catheter dressing changes. c) Weigh the client every day. d) Cover insertion site with a transparent dressing that is changed daily. e) Check blood glucose level every 6 hours.

A) Document intake and output; C) Weigh the client every day; E) Check blood glucose level every 6 hours When a client is receiving PN through a central line, the nurse weighs the client daily, checks blood glucose level every 6 hours, and documents intake and output. These actions are to ensure the client is receiving optimal nutrition. The nurse also performs activities to prevent infection, such as covering the insertion site with a transparent dressing that is changed weekly and/or prn and using sterile technique during catheter site dressing changes.

The nurse is caring for a patient in the emergency department with complaints of acute abdominal pain, nausea, and vomiting. When the nurse palpates the patient's left lower abdominal quadrant, the patient complains of pain in the right lower quadrant. The nurse will document this as which of the following diagnostic signs of appendicitis? A) Rovsing sign B) Referred pain C) Chvostek's sign D) Rebound tenderness

A) Rovsing sign In patients with suspected appendicitis, Rovsing sign may be elicited by palpation of the left lower quadrant, causing pain to be felt in the right lower quadrant.

Which of the following is caused by improper catheter placement and inadvertent puncture of the pleura? a) Pneumothorax b) Sepsis c) Fluid overload d) Air embolism

A) Pneumothorax A pneumothorax is caused by improper catheter placement and inadvertent puncture of the pleura. Air embolism can occur from a missing cap on a port. Sepsis can be caused by the separation of dressings. Fluid overload is caused by fluids infusing too rapidly.

A nurse is interviewing a client about his past medical history. Which preexisting condition may lead the nurse to suspect that a client has colorectal cancer? a) Polyps b) Weight gain c) Hemorrhoids d) Duodenal ulcers

A) Polyps Colorectal polyps are common with colon cancer. Duodenal ulcers and hemorrhoids aren't preexisting conditions of colorectal cancer. Weight loss — not gain — is an indication of colorectal cancer.

The most common cause of esophageal varices includes which of the following? a) Portal hypertension b) Asterixis c) Jaundice d) Ascites

A) Portal hypertension Esophageal varices are almost always caused by portal hypertension, which results from obstruction of the portal circulation within the damaged liver. Jaundice occurs when the bilirubin concentration in the blood is abnormally elevated. Ascites results from circulatory changes within the diseased liver. Asterixis is an involuntary flapping movement of the hands associated with metabolic liver dysfunction.

The nurse observes dry mucous membranes in a client who is receiving tube feedings after an oral surgery. The client also complains of unpleasant tastes and odors. Which of the following measures should be included in the client's plan of care? a) Provide frequent mouth care. b) Keep the feeding formula refrigerated. c) Ensure adequate hydration with additional water. d) Flush the tube with water before adding the feedings.

A) Provide frequent mouth care Frequent mouth care helps to relieve the discomfort from dryness and unpleasant odors and tastes. It can be done with the help of ice chips and analgesic throat lozenges, gargles, or sprays. Adequate hydration is essential. If urine output is less than less than 500 mL/day, formula and additional water can be given as ordered. Keeping the feeding formula refrigerated and unopened until it is ready for use and flushing the tube with water before adding feedings are measures to protect the client from infections.

A patient with a history of peptic ulcer disease has presented to the emergency department with complaints of severe abdominal pain and a rigid, boardlike abdomen, prompting the health care team to suspect a perforated ulcer. Which of the following actions should the nurse anticipate? A) Providing IV fluids and inserting a nasogastric tube B) Administering oral bicarbonate and testing the patient's gastric pH level C) Performing a fecal occult blood test and administering IV calcium gluconate D) Starting parenteral nutrition and placing the patient in a high-Fowler's position

A) Providing IV fluids and inserting a NG tube A perforated peptic ulcer requires IV replacement of fluid losses and continued gastric aspiration by NG tube. Nothing is given by mouth and gastric pH testing is not a priority. Calcium gluconate is not a medication directly relevant to the patient's suspected diagnosis and parenteral nutrition is not a priority in the short term.

A pt with a history of peptic ulcer disease has presented to the emergency department with complaints of severe abdominal pain and a rigid, boardlike abdomen, prompting the health care team to suspect a perforated ulcer. Which of the following actions should the nurse anticipate? A) Providing IV fluids and inserting a nasogastric tube B) Administering oral bicarbonate and testing the patient's gastric pH level C) Performing a fecal occult blood test and administering IV calcium gluconate D) Starting parenteral nutrition and placing the patient in a high-Fowler's position

A) Providing IV fluids and inserting a nasogastric tube A perforated peptic ulcer requires IV replacement of fluid losses and continued gastric aspiration by NG tube. Nothing is given by mouth and gastric pH testing is not a priority. Calcium gluconate is not a medication directly relevant to the patient's suspected diagnosis and parenteral nutrition is not a priority in the short term.

A patient scheduled to undergo an abdominal ultrasonography is advised to do which of the following? a) Restrict eating of solid food for 6 to 8 hours before the test. b) Do not consume anything sweet for 24 hours before the test c) Do not undertake any strenuous exercise for 24 hours before the test d) Avoid exposure to sunlight for at least 6 to 8 hours before the test

A) Restrict eating of solid food for 6 to 8 hours before the test For a patient who is scheduled to undergo an abdominal ultrasonography, the patient should restrict herself from solid food for 6 to 8 hours to avoid having images of her test obscured with gas and intestinal contents. Ultrasonography records the reflection of sound waves. Strenuous exercises, the consumption of sweets, and exposure to sunlight do not affect the results of the test in any way.

A barium Enema should be done before an upper gastrointestinal series because of which of the following? A) Retained barium may cloud the colon B)barium Can cause lower Gastrointestinal bleeding C) The physicians orders are in that sequence D)barium Is absorbed readily in the lower intestine

A) Retained barium may cloud the colon The barium And I'm should always perceive the upper gastrointestinal series because retained barium from the latter may take several days to pass through the gastrointestinal tract and may cloud anatomic detail on the barium enema studies

A client with acute liver failure exhibits confusion, a declining level of consciousness, and slowed respirations. The nurse finds him very difficult to arouse. The diagnostic information which best explains the client's behavior is: a) subnormal serum glucose and elevated serum ammonia levels. b) subnormal clotting factors and platelet count. c) elevated liver enzymes and low serum protein level. d) elevated blood urea nitrogen and creatinine levels and hyperglycemia.

A) Subnormal serum glucose and elevated serum ammonia levels In acute liver failure, serum ammonia levels increase because the liver can't adequately detoxify the ammonia produced in the GI tract. In addition, serum glucose levels decline because the liver isn't capable of releasing stored glucose. Elevated serum ammonia and subnormal serum glucose levels depress the level of a client's consciousness. Elevated liver enzymes, low serum protein level, subnormal clotting factors and platelet count, elevated blood urea nitrogen and creatine levels, and hyperglycemia aren't as directly related to the client's level of consciousness.

Blood shed in sufficient quantities into the upper GI tract, produces which color of stool? a) Tarry-black b) Milky white c) Green d) Bright red

A) Tarry-black Blood shed in sufficient quantities into the upper GI tract produces a tarry-black stool. Blood entering the lower portion of the GI tract or passing rapidly through it will appear bright or dark red. A milky white stool is indicative "of" a patient who received barium. A green stool is indicative of a patient who has eaten spinach.

A client with dysphagia is being prepared for discharge. Which outcome indicates that the client is ready for discharge? a) The client is free from esophagitis and achalasia. b) The client doesn't exhibit rectal tenesmus. c) The client has normal gastric structures. d) The client reports diminished duodenal inflammation.

A) The client is free from esophagitis and achalasia Dysphagia may be the reason why a client with esophagitis or achalasia seeks treatment. Therefore, when the client is free of esophagitis or achalasia, he is ready for discharge. Dysphagia isn't associated with rectal tenesmus, duodenal inflammation, or abnormal gastric structures.

A client with dysphagia is being prepared for discharge. Which outcome indicates that the client is ready for discharge? a) The client is free from esophagitis and achalasia. b) The client reports diminished duodenal inflammation. c) The client has normal gastric structures. d) The client doesn't exhibit rectal tenesmus.

A) The client is free from esophagitis and achalasia. Dysphagia may be the reason why a client with esophagitis or achalasia seeks treatment. Therefore, when the client is free of esophagitis or achalasia, he is ready for discharge. Dysphagia isn't associated with rectal tenesmus, duodenal inflammation, or abnormal gastric structures.

A nurse enters the room of a client who has returned to the unit after having a radical neck dissection. Which assessment finding requires immediate intervention? a) The client lying in a lateral position, with the head of bed flat b) Foley catheter bag containing 500 ml of amber urine c) Serosanguineous drainage on the dressing d) A piggyback infusion of levofloxacin (Levaquin)

A) The client lying in a lateral position, with the head of bed flat A client who has had neck surgery is at risk for neck swelling. To prevent respiratory complications, the head of the bed should be at least at a 30-degree angle. This position gives the lungs room to expand and decreases swelling by promoting venous and lymphatic drainage. This position also minimizes the risk of aspiration. Serosanguineous drainage on the dressing, a Foley bag containing amber urine, and levofloxacin infusing aren't causes for concern.

The nurse is preparing to insert a nasogastric tube into a 68-year-old patient with an abdominal mass and suspected bowel obstruction. The patient asks the nurse why this procedure is necessary. Which of the following responses is most appropriate? A) "The tube will help to drain the stomach contents and prevent further vomiting." B) "The tube will push past the area that is blocked, and thus help to stop the vomiting." C) "The tube is just a standard procedure before many types of surgery to the abdomen." D) "The tube will let us measure your stomach contents, so that we can plan what type of IV fluid replacement would be best."

A) The tube will help drain the stomach contents and prevent further vomiting The nasogastric tube is used to decompress the stomach by draining stomach contents, and thereby prevent further vomiting.

When administered a dose of metoclopramide (Reglan), a patient complains of nausea. The nurse would teach the patient to report which of the following potential adverse effects? A) Tremors B) Constipation C) Double vision D) Numbness in the fingers and dose

A) Tremors (Extrapyramidal side effects, including tremors and dyskinesias, may occur as a result of metoclopramide (Reglan) administration.)

A client with GERD develops espophagitis. Which diagnostic test would the nurse expect the physician to order to confirm the diagnosis? a) Upper endoscopy with biopsy b) Stool testing for occult blood c) 24-hour esophageal pH monitoring d) Barium swallow

A) Upper endoscopy with biopsy Upper endoscopy with biopsy confirms esophagitis. Barium-swallow would reveal inflammation or stricture formation from chronic esophagitis. Tests of stool may show positive findings of blood. Ambulatory 24-hour esophageal pH monitoring allows for observation of the frequency of reflux episodes and their associated symptoms.

The nurse inserts a nasoduodenal tube for feeding of the client. To check best for placement, the nurse a) Verifies location with an abdominal x-ray b) Aspirates contents and checks the color of the aspirate c) Auscultates when injecting air d) Adds 8 to 10 inches of the tube after inserting to the xiphoid process

A) Verifies location with an abdominal x-ray Initially, an x-ray should be used to confirm placement of the nasoduodenal tube. It is the most accurate method to verify tube placement. Adding 8 to 10 inches to the length of the tube after measuring from nose to earlobe to xiphoid process is not supported, because it does not indicate that the tube will be in the correct position. Intestinal aspirate is usually clear and yellow to bile-colored. Gastric aspirate is usually cloudy and green, tan, off-white, or brown. Food particles may be present. The traditional method of injecting air through the tube while auscultating the epigastric area with a stethoscope to detect air insufflation is also an unreliable indicator.

Initially, which diagnostic should be completed following placement of a NG tube? a) X-ray b) Measurement of tube length c) pH measurement of aspirate d) Visual assessment of aspirate

A) X-ray Instead of auscultation, a combination of three methods is recommended: measurement of tube length, visual assessment of aspirate, and pH measurement of aspirate.

Which of the following are included in the nursing plan of care to prevent adverse effects when caring for patients with a nasogastric tube in place for gastric decompression's? Select all that apply. A) Irrigation with Saline B) Measure the length of exposed tube C) Measure the pH of the aspirated tube contents D) Administer frequent oral hygiene

A,B,C,D After checking placement, NGT should be Irrigated with 30 to 60 mL of normal saline to maintain patency. The frequency is determined by facility policy, medical order, and nursing judgment. The length of the exposed tubes should be measured after insertion and routinely thereafter, as part of the assessment to verify placement and ensure the tube has not dislodged. Measuring the pH of aspirated two contents is one way to validate to placement in the intestinal tract. The other methods is to visually Assess aspirate to confirm gastric contents. Patients with NGTs Often experience discomfort related to irritation to nasal and throat mucosa And drying of the mucous membranes. Frequent oral hygiene should be administered as well as applying lubricant to the lips

When caring for a patient with liver disease, the nurse recognizes the need to prevent bleeding resulting from altered clotting factors and rupture of varices. Which of the following nursing interventions would be appropriate to achieve this outcome (select all that apply)? A) Use smallest gauge possible when giving injections or drawing blood. B) Teach patient to avoid straining at stool, vigorous blowing of nose, and coughing. C) Advise patient to use soft-bristle toothbrush and avoid ingestion of irritating food. D) Apply gentle pressure for the shortest possible time period after performing venipuncture. E) Instruct patient to avoid aspirin and NSAIDs to prevent hemorrhage when varices are present.

A,B,C,E Using the smallest gauge for injections will minimize the risk of bleeding into the tissues. Avoiding straining, nose blowing, and coughing will reduce the risk of hemorrhage at these sites. The use of a soft-bristle toothbrush and avoidance of irritating food will reduce injury to highly vascular mucous membranes. The nurse should apply gentle but prolonged pressure to venipuncture sites to minimize the risk of bleeding.

What are risk factors for peritonitis "Hot Belly"?

Abd surgery, ectopic pregnancy, any "scope" procedure, perforation of: ulcer, appendix, diverticulum, or trauma caused.

A 24-year-old athlete is admitted to the trauma unit following a motor-vehicle collision. The client is comatose and has developed ascites as a result of the accident. You are explaining the client's condition to his parents. In your education, what do you indicate is the primary function of the small intestine? a) Digest fats b) Digest proteins c) Absorb water d) Absorb nutrients

Absorb nutrients Explanation: The primary function of the small intestine is to absorb nutrients from the chyme.

Which of the following is the primary function of the small intestine? a) Secretion b) Peristalsis c) Absorption d) Digestion

Absorption Explanation: Absorption is the primary function of the small intestine. Digestion occurs in the stomach. Peristalsis occurs in the colon. The duodenum secretes enzymes.

What is the recommended colon cancer screening for non-high-risk clients?

Annual fecal testing > age 50.

When assisting with preparing a client scheduled for a barium swallow, which of the following would be appropriate to include? a) Take three cleansing enemas before the procedure. b) Take vitamin K before the procedure. c) Avoid the intake of red meat before the procedure. d) Avoid smoking for at least a day before the procedure.

Avoid smoking for at least a day before the procedure. Explanation: The nurse should instruct the client to avoid smoking for at least a day before the procedure of barium swallow because smoking stimulates gastric motility. The client is advised to take vitamin K before a liver biopsy and instructed to take three cleansing enemas before a barium enema. Instruction to avoid red meat would be appropriate for a client who is having a Hemoccult test.

The results of a patient's recent endoscopy indicate the presence of peptic ulcer disease (PUD). Which of the following teaching points should the nurse provide to the patient in light of his new diagnosis? A) "You'll need to drink at least two to three glasses of milk daily." B) "It would likely be beneficial for you to eliminate drinking alcohol." C) "Many people find that a minced or pureed diet eases their symptoms of PUD." D) "Your medications should allow you to maintain your present diet while minimizing symptoms."

B) "It would likely be beneficial for you to eliminate drinking alcohol" Although there is no specific recommended dietary modification for PUD, most patients find it necessary to make some sort of dietary modifications to minimize symptoms. Milk may exacerbate PUD and alcohol is best avoided because it can delay healing.

A client who is recovering from anesthesia following oral surgery for lip cancer is experiencing difficulty breathing deeply and coughing up secretions. Which of the following measures will help ease the client's discomfort? a) Positioning the client flat on the abdomen or side. b) Keeping the head of the bed elevated. c) Turning the client's head to the side. d) Providing a tracheostomy tray near the bed.

B) Keeping the head of the bed elevated It is essential to position the client with the head of the bed elevated because it is easier for the client to breathe deeply and cough up secretions after recovering from the anesthetic. Positioning the client flat either on the abdomen or side with the head turned to the side will facilitate drainage from the mouth. A tracheostomy tray is kept by the bed for respiratory distress or airway obstruction. When mouth irrigation is carried out, the nurse should turn the client's head to the side to allow the solution to run in gently and flow out.

A 74-year-old client is on the hospital unit where you practice nursing. She will be undergoing rhinoplasty and you are completing her admission assessment and paperwork. She reports medications she uses on a daily basis, which you record for her chart. Which of her daily medications will result in constipation? a) Acetaminophen b) Laxative c) NSAIDs d) Multivitamin without iron

B) Laxative Constipation may also result from chronic use of laxatives ("cathartic colon") because such use can cause a loss of normal colonic motility and intestinal tone. Laxatives also dull the gastrocolic reflex.

A nurse is providing dietary instructions to a client with a history of pancreatitis. Which instruction is correct? a) "Maintain a high-sodium, high-calorie diet." b) "Maintain a high-carbohydrate, low-fat diet." c) "Maintain a high-fat, high-carbohydrate diet." d) "Maintain a high-fat diet and drink at least 3 L of fluid a day."

B) Maintain a high carbohydrate, low fat diet A client with a history of pancreatitis should avoid foods and beverages that stimulate the pancreas, such as fatty foods, caffeine, and gas-forming foods; should avoid eating large meals; and should eat plenty of carbohydrates, which are easily metabolized. Therefore, the only correct instruction is to maintain a high-carbohydrate, low-fat diet. An increased sodium or fluid intake isn't necessary because chronic pancreatitis isn't associated with hyponatremia or fluid loss.

If a client has abdominal surgery and a portion of the small intestine is removed, the client is at risk for which of the following? a) Gastric ulcers b) Malabsorption syndrome c) Constipation d) Cirrhosis

B) Malabsorption syndrome Absorption is the primary function of the small intestine. Vitamins and minerals are absorbed essentially unchanged. Nutrients are absorbed at specific locations in the small intestine.

The nurse should administer a prn dose of magnesium hydroxide (MOM) after noting which of the following while reviewing a patient's medical record? A) Abdominal pain and bloating B) No bowel movement for 3 days C) A decrease in appetite by 50% over 24 hours D) Muscle tremors and other signs of hypomagnesemia

B) No bowel movement for 3 days MOM is an osmotic laxative that produces a soft, semisolid stool usually within 15 minutes to 3 hours. This medication would benefit the patient who has not had a bowel movement for 3 days.

A client with a disorder of the oral cavity cannot tolerate tooth brushing or flossing. Which of the following strategies can the nurse employ to assist this client? a) Regularly wipe the outside of the client's mouth to prevent germs from entering. b) Provide the client with an irrigating solution of baking soda and warm water. c) Recommend that the client drink a small glass of alcohol at the end of the day to kill germs. d) Urge the client to regularly rinse the mouth with tap water.

B) Provide the client with an irrigating solution of baking soda and warm water If a client cannot tolerate brushing or flossing, an irrigating solution of 1 tsp of baking soda to 8 oz of warm water, half strength hydrogen peroxide, or normal saline solution is recommended.

When a central venous catheter dressing becomes moist or loose, what should a nurse do first? a) Notify the physician. b) Remove the dressing, clean the site, and apply a new dressing. c) Remove the catheter, check for catheter integrity, and send the tip for culture. d) Draw a circle around the moist spot and note the date and time.

B) Remove dressing, clean the site, and apply a new dressing. A nurse maintaining a central venous catheter should change the dressing every 72 hours or when it becomes soiled, moist, or loose. After removing the soiled dressing, the nurse should use sterile technique to clean around the site in accordance with facility policy. After the cleaning solution has dried, the nurse should cover the site with a transparent semipermeable dressing. A nurse who notes drainage on a wound dressing should draw a circle around the moist spot and note the date and time. She should notify the physician if she observes any catheter-related complications. Only a nurse with the appropriate qualifications may remove a central venous catheter, and a moist or loose dressing isn't a reason to remove the catheter.

A nurse is providing postprocedure instructions for a client who had an esophagogastroduodenoscopy. The nurse should perform which action? a) Tell the client to call back in the morning so she can give him instructions over the phone. b) Review the instructions with the person accompanying the client home. c) Tell the client there aren't specific instructions for after the procedure. d) Give instructions to the client immediately before discharge.

B) Review the instructions with the person accompanying the client home A client who undergoes esophagogastroduodenoscopy receives sedation during the procedure, and his memory becomes impaired. Clients tend not to remember instructions provided after the procedure. The nurse's best course of action is to give the instructions to the person who is accompanying the client home. It isn't appropriate for the nurse to tell the client to call back in the morning for instructions. The client needs to be aware at discharge of potential complications and signs and symptoms to report to the physician.

A client is admitted with a diagnosis of acute appendicitis. When assessing the abdomen, the nurse would expect to find rebound tenderness at which location? a) Left upper quadrant b) Right lower quadrant c) Left lower quadrant d) Right upper quadrant

B) Right lower quadrant The pain of acute appendicitis localizes in the right lower quadrant (RLQ) at McBurney's point, an area midway between the umbilicus and the right iliac crest. Often, the pain is worse when manual pressure near the region is suddenly released, a condition called rebound tenderness.

Nursing assessment of a client with peritonitis reveals hypotension, tachycardia, and signs and symptoms of dehydration. The nurse also expects to find: a) tenderness and pain in the right upper abdominal quadrant. b) severe abdominal pain with direct palpation or rebound tenderness. c) jaundice and vomiting. d) rectal bleeding and a change in bowel habits.

B) Severe abdominal pain with direct palpation or rebound tenderness Peritonitis decreases intestinal motility and causes intestinal distention. A classic sign of peritonitis is a sudden, diffuse, severe abdominal pain that intensifies in the area of the underlying causative disorder (such as appendicitis, diverticulitis, ulcerative colitis, or a strangulated obstruction). The client may also have rebound tenderness. Tenderness and pain in the right upper abdominal quadrant suggest cholecystitis. Jaundice and vomiting are signs of cirrhosis of the liver. Rectal bleeding or a change in bowel habits may indicate colorectal cancer.

Which of the following medications would the nurse expect the physician to order for a client with cirrhosis who develops portal hypertension? a) Kanamycin (Kantrex) b) Spironolactone (Aldactone) c) Cyclosporine (Sandimmune) d) Lactulose (Cephulac)

B) Spironlactone (Aldactone) For portal hypertension, a diuretic usually an aldosterone antagonist such as spironolactone (Aldactone) is ordered. Kanamycin (Kantrex) would be used to treat hepatic encephalopathy to destroy intestinal microorganisms and decrease ammonia production. Lactulose would be used to reduce serum ammonia concentration in a client with hepatic encephalopathy. Cyclosporine (Sandimmune) would be used to prevent graft rejection after a transplant.

A client with acute liver failure exhibits confusion, a declining level of consciousness, and slowed respirations. The nurse finds him very difficult to arouse. The diagnostic information which best explains the client's behavior is: a) elevated blood urea nitrogen and creatinine levels and hyperglycemia. b) subnormal serum glucose and elevated serum ammonia levels. c) subnormal clotting factors and platelet count. d) elevated liver enzymes and low serum protein level.

B) Subnormal serum glucose and elevated serum ammonia levels In acute liver failure, serum ammonia levels increase because the liver can't adequately detoxify the ammonia produced in the GI tract. In addition, serum glucose levels decline because the liver isn't capable of releasing stored glucose. Elevated serum ammonia and subnormal serum glucose levels depress the level of a client's consciousness. Elevated liver enzymes, low serum protein level, subnormal clotting factors and platelet count, elevated blood urea nitrogen and creatine levels, and hyperglycemia aren't as directly related to the client's level of consciousness.

The nurse is preparing to examine the abdomen of a client complaining of a change in his bowel pattern. The nurse would place the client in which position? a) Lithotomy b) Supine with knees flexed c) Knee-chest d) Left Sim's lateral

B) Supine with knees flexed When examining the abdomen, the client lies supine with his knees flexed. This position assists in relaxing the abdominal muscles. The lithotomy position commonly is used for a female pelvic examination and to examine the rectum. The knee-chest position can be used for a variety of examinations, most commonly the anus and rectum. The left Sim's lateral position may be used to assess the rectum or vagina and to administer an enema.

A client with appendicitis is experiencing excruciating abdominal pain. An abdominal X-ray film reveals intraperitoneal air. The nurse should prepare the client for: a) colonoscopy. b) surgery. c) nasogastric (NG) tube insertion. d) barium enema.

B) Surgery The client should be prepared for surgery because his signs and symptoms indicate bowel perforation. Appendicitis is the most common cause of bowel perforation in the United States. Because perforation can lead to peritonitis and sepsis, surgery wouldn't be delayed to perform other interventions, such as colonoscopy, NG tube insertion, or a barium enema. These procedures aren't necessary at this point.

A client receives a local anesthetic to suppress the gag reflex for a diagnostic procedure of the upper GI tract. Which of the following nursing interventions is advised for this patient? a) The client should be monitored for any breathing related disorder or discomforts b) The client should not be given any food and fluids until the gag reflex returns c) The client should be monitored for cramping or abdominal distention d) The client's fluid output should be measured for at least 24 hours after the procedure

B) The client should not be given any food and fluids until the gag reflex returns. For a client receiving a local anesthetic that suppresses the gag reflex, the nurse is advised to withhold food and fluids until the reflex returns.

Which patient teaching component is important for the nurse to communicate regarding pain management prior to or during diagnostic testing for a disorder of the GI system? a) The patient should not expel gas and test fluids from the bowel when he or she experiences the urge during the procedure. b) The patient should inform the test personnel if he or she experiences pressure or cramping during the instillation of test fluids. c) The patient should take a sedative before the procedure to avoid the possibility of experiencing any discomfort. d) The patient should lie down in a supine position for at least 3 hours before the test to reduce any discomfort during the test.

B) The patient should inform the test personnel if he or she experiences pressure or cramping during instillation of test fluids To ensure that a patient who is to undergo a diagnostic test for a disorder of the gastrointestinal system experiences no or minimal discomfort during the test, the patient should be instructed to inform the test personnel if he or she experiences pressure or cramping during the instillation of test fluids. The test personnel can slow the instillation or take other measures to relieve discomfort. The patient should also be advised to expel gas and test fluids from the bowel when he or she experiences the urge. Ignoring the urge to expel the bowel contents increases pain and discomfort. The patient should be advised not to take any sedative or analgesic before the test, unless prescribed. Lying down in a supine position is not known to have any consequence on the level of discomfort experienced by a patient during a diagnostic test for a GI disorder.

Nurses should recommend avoiding the habitual use of laxatives. Which of the following is the rationale for this? A) They will cause a fecal impaction B) They will cause chronic constipation C) They change the pH of the Gastrointestinal track D) They inhibit the intestinal enzymes

B) They will cause chronic constipation Habitual use of laxatives is the most common cause of chronic constipation

Dietary fiber

Dietary fiber, a polysaccharide, is the indigestible "fi brous skeleton" of plant foods. Humans do not have the enzymes to digest fiber; thus, it provides no usable glucose.

A nurse is caring for a client who had gastric bypass surgery 2 days ago. Which assessment finding requires immediate intervention? a) The client states he has been passing gas. b) The client states he is nauseated. c) The client's right lower leg is red and swollen. d) The client complains of pain at the surgical site.

C) Client's right lower leg is red and swollen A red, swollen extremity is a possible sign of a thromboembolism, a common complication after gastric surgery because of the fact that the clients are obese and tend to ambulate less than other surgical clients. The nurse should inform the physician of the finding. Pain at the surgical site should be investigated, but the red, swollen leg is a higher priority. It isn't unusual for a client to be nauseated after gastric bypass surgery. The nurse should follow up with the finding, but only after she has notified the physician about the possible thromboembolism. Passing gas is normal and a sign that the client's intestinal system is beginning to mobilize.

A client is scheduled for several diagnostic tests to evaluate her gastrointestinal function. After teaching the client about these tests, the nurse determines that the client has understood the teaching when she identifies which test as not requiring the use of a contrast medium? a) Computer tomography b) Small bowel series c) Colonoscopy d) Upper GI series

C) Colonoscopy A colonoscopy is a direct visual examination of the entire large intestine. It does not involve the use of a contrast agent. Contrast medium may be used with a small bowel series, computed tomography, and upper GI series.

A client with a gastrojejunostomy is beginning to take solid food. Which finding would lead the nurse to suspect that the client is experiencing dumping syndrome? a) Slowed heart beat b) Hyperglycemia c) Diarrhea d) Dry skin

C) Diarrhea Clients with a gastrojejunostomy are at risk for developing the dumping syndrome when they begin to take solid food. This syndrome produces weakness, dizziness, sweating, palpitations, abdominal cramps, and diarrhea, which result from the rapid emptying (dumping) of large amounts of hypertonic chyme (a liquid mass of partly digested food) into the jejunum. This concentrated solution in the gut draws fluid from the circulating blood into the intestine, causing hypovolemia. The drop in blood pressure can produce syncope. As the syndrome progresses, the sudden appearance of carbohydrates in the jejunum stimulates the pancreas to secrete excessive amounts of insulin, which in turn causes hypoglycemia.

A client is admitted to the healthcare facility suspected of having acute pancreatitis and undergoes laboratory testing. Which of the following would the nurse expect to find? a) Decreased white blood cell count b) Decreased liver enzyme levels c) Elevated urine amylase levels d) Increased serum calcium levels

C) Elevated urine amylase levels Elevated serum and urine amylase, lipase, and liver enzyme levels accompany significant pancreatitis. If the common bile duct is obstructed, the bilirubin level is above normal. Blood glucose levels and white blood cell counts can be elevated. Serum electrolyte levels (calcium, potassium, and magnesium) are low.

The nurse determines that a patient has experienced the beneficial effects of medication therapy with famotidine (Pepcid) when which of the following symptoms is relieved? A) Nausea B) Belching C) Epigastric pain D) Difficulty swallowing

C) Epigastric Pain Famotidine is an H2-receptor antagonist that inhibits parietal cell output of HCl acid and minimizes damage to gastric mucosa related to hyperacidity, thus relieving epigastric pain.

The nurse determines that a pnt has experienced the beneficial effects of medication therapy with famotidine (Pepcid) when which of the following symptoms is relieved? A) Nausea B) Belching C) Epigastric pain D) Difficulty swallowing

C) Epigastric pain Famotidine is an H2-receptor antagonist that inhibits parietal cell output of HCl acid and minimizes damage to gastric mucosa related to hyperacidity, thus relieving epigastric pain.

Which of the following surgical procedures for obesity utilizes a prosthetic device to restrict oral intake? a) Vertical-banded gastroplasty b) Roux-en-Y gastric bypass c) Gastric banding d) Biliopancreatic diversion with duodenal switch

C) Gastric banding In gastric banding, a prosthetic device is used to restrict oral intake by creating a small pouch of 10 to 15 milliliters that empties through the narrow outlet into the remainder of the stomach. Roux-en-Y gastric bypass uses a horizontal row of staples across the fundus of the stomach to create a pouch with a capacity of 20 to 30 mL. Vertical-banded gastroplasty involves placement of a vertical row of staples along the lesser curvature of the stomach, creating a new, small gastric pouch. Biliopancreatic diversion with duodenal switch combines gastric restriction with intestinal malabsorption.

The nurse is caring for a man who has experienced a spinal cord injury. Throughout his recovery, the client expects to gain control of his bowels. The nurse's best response to this client would be which of the following? a) "Over time, the nerve fibers will regrow new tracts, and you can have bowel movements again." b) "Wearing an undergarment will become more comfortable over time." c) "Having a bowel movement is a spinal reflex requiring intact nerve fibers. Yours are not intact." d) "It is not going to happen. Your nerve cells are too damaged."

C) Having a bowel movement is a spinal reflex requiring intact nerve fibers. Yours are not intact The act of defecation is a spinal reflex involving the parasympathetic nerve fibers. Normally, the external anal sphincter is maintained in a state of tonic contraction. With a spinal cord injury, the client no longer has this nervous system control and is often incontinent.

A nurse is preparing a presentation for a local community group about hepatitis. Which of the following would the nurse include? a) Hepatitis B is transmitted primarily by the oral-fecal route. b) Hepatitis A is frequently spread by sexual contact. c) Hepatitis C increases a person's risk for liver cancer. d) Infection with hepatitis G is similar to hepatitis A.

C) Hep C increases a person's risk for liver cancer Infection with hepatitis C increases the risk of a person developing hepatic (liver) cancer. Hepatitis A is transmitted primarily by the oral-fecal route; hepatitis B is frequently spread by sexual contact and infected blood. Hepatitis E is similar to hepatitis A whereas hepatitis G is similar to hepatitis C.

Which of the following is a protrusion of the intestine through a weakened area in the abdominal wall? a) Tumor b) Adhesion c) Hernia d) Volvulus

C) Hernia A hernia is a protrusion of intestine through a weakened area in the abdominal muscle or wall. A tumor that extends into the intestinal lumen, or a tumor outside the intestine causes pressure on the wall of the intestine. Volvulus occurs when the bowel twists and turns on itself. An adhesion occurs when loops of intestine become adherent to areas that heal slowly or scar after abdominal surgery.

A client is diagnosed with a hiatal hernia. Which statement indicates effective client teaching about hiatal hernia and its treatment? a) "I'll lie down immediately after a meal." b) "I'll eat three large meals every day without any food restrictions." c) "I'll eat frequent, small, bland meals that are high in fiber." d) "I'll gradually increase the amount of heavy lifting I do."

C) I'll eat frequent, small, bland meals that are high in fiber In hiatal hernia, the upper portion of the stomach protrudes into the chest when intra-abdominal pressure increases. To minimize intra-abdominal pressure and decrease gastric reflux, the client should eat frequent, small, bland meals that can pass easily through the esophagus. Meals should be high in fiber to prevent constipation and minimize straining on defecation (which may increase intra-abdominal pressure from the Valsalva maneuver). Eating three large meals daily would increase intra-abdominal pressure, possibly worsening the hiatal hernia. The client should avoid spicy foods, alcohol, and tobacco because they increase gastric acidity and promote gastric reflux. To minimize intra-abdominal pressure, the client shouldn't recline after meals, lift heavy objects, or bend.

Crohn's disease is a condition of malabsorption caused by which of the following pathophysiological processes? a) Gastric resection b) Infectious disease c) Inflammation of all layers of intestinal mucosa d) Disaccharidase deficiency

C) Inflammation of all layers of intestinal mucosa Crohn's disease is also known as regional enteritis and can occur anywhere along the GI tract, but most commonly at the distal ileum and in the colon. Infectious disease causes problems such as small bowel bacterial overgrowth leading to malabsorption. Disaccharidase deficiency leads to lactose intolerance. Postoperative malabsorption occurs after gastric or intestinal resection.

The client is receiving a 25% dextrose solution of parenteral nutrition. The infusion machine is beeping, and the nurse determines the intravenous (IV) bag is empty. The nurse finds there is no available bag to administer. It is most important for the nurse to a) Request a new bag from the pharmacy department. b) Flush the line with 10 mL of sterile saline. c) Infuse a solution containing 10% dextrose and water. d) Catch up with the next bag when it arrives.

C) Infuse a solution containing 10% dextrose and water If the parenteral nutrition solution runs out, a solution of 10% dextrose and water is infused to prevent hypoglycemia. The nurse would then order the next parenteral nutrition bag from the pharmacy. Flushing a peripherally inserted catheter is usually prescribed every 8 hours or per hospital established protocols. It is not the most important activity at this moment. The infusion rate should not be increased to compensate for fluids that were not infused, because hyperglycemia and hyperosmolar diuresis could occur.

When assisting with the plan of care for a client receiving tube feedings, which of the following would the nurse include to reduce the client's risk for aspiration? a) Administering 15 to 30 mL of water every 4 hours. b) Aspirating for residual contents every 4 to 8 hours. c) Keeping the client in a semi-Fowler's position at all times. d) Giving the feedings at room temperature.

C) Keeping the client in a semi Fowler's position at all times With continuous tube feedings, the nurse needs to keep the client in a semi-Fowler's position at all times to reduce regurgitation and the risk for aspiration. Aspirating for residual contents helps to ensure adequate nutrition and prevent overfeeding. Administering 15 to 30 mL of water every 4 hours helps to maintain tube patency. Giving the feedings at room temperature reduces the risk for diarrhea.

While palpating a client's right upper quadrant (RUQ), the nurse would expect to find which structure? a) Spleen b) Appendix c) Liver d) Sigmoid colon

C) Liver The RUQ contains the liver, gallbladder, duodenum, head of the pancreas, hepatic flexure of the colon, portions of the ascending and transverse colon, and a portion of the right kidney. The sigmoid colon is located in the left lower quadrant; the appendix, in the right lower quadrant; and the spleen, in the left upper quadrant.

When bowel sounds are heard about every 15 seconds, the nurse would record that the bowel sounds are a) absent. b) hypoactive. c) normal. d) sluggish.

C) Normal Normal bowel sounds are heard every 5 to 20 seconds. Hypoactive bowel sound is the description given to auscultation of one to two bowel sounds in 2 minutes. Sluggish is not a term a nurse would use to accurately describe bowel sounds. The nurse records that bowel sounds are absent when no sound is heard in 3 to 5 minutes.

The nurse asks a client to point to where she feels pain. The client asks why this is important. The nurse's best response would be which of the following? a) "This determines the pain medication to be ordered." b) "If the doctor massages over the exact painful area, the pain will disappear." c) "Often the area of pain is referred from another area." d) "The area may determine the severity of the pain."

C) Often the area of pain is referred from another area Pain can be a major symptom of disease. The location and distribution of pain can be referred from a different area. If a client points to an area of pain and has other symptoms associated with a certain disease, this is valuable information for treatment.

When explaining the action of a hyper tonic solution enema, the nurse incorporates which of the following as the basis for action? A) bowel mucosa irritations B) Diffusion of water out of colon C) Osmosis of water into colon D) Softening of fecal contents

C) Osmosis of water into colon Hypertonic solutions draw water into the colon a By osmosis that's stimulating the defecation reflex. Orrills solutions soften fecal contents, and soup solutions distend the intestines and irritate the bowel mucosa

The nurse recognizes that the patient diagnosed with a duodenal ulcer will likely experience a) weight loss. b) vomiting. c) pain 2 to 3 hours after a meal. d) hemorrhage.

C) Pain 2 to 3 hours after a meal The patient with a gastric ulcer often awakens between 1 to 2 with pain, and ingestion of food brings relief. Vomiting is uncommon in the patient with duodenal ulcer. Hemorrhage is less likely in the patient with duodenal ulcer than the patient with gastric ulcer. The patient with a duodenal ulcer may experience weight gain.

A client with hepatitis who has not responded to medical treatment is scheduled for a liver transplant. Which of the following most likely would be ordered? a) Chenodiol b) Ursodiol c) Tacrolimus d) Interferon alfa-2b, recombinant

C) Tacrolimus In preparation for a liver transplant, a client receives immunosuppressants to reduce the risk for organ rejection. Tacrolimus or cyclosporine are two immunosuppresants that may be used. Chenodiol and ursodiol are agents used to dissolve gall stones. Recombinant interferon alfa-2b is used to treat chronic hepatitis B, C, and D to force the virus into remission.

When caring for a client with acute pancreatitis, the nurse should use which comfort measure? a) Encouraging frequent visits from family and friends b) Administering frequent oral feedings c) Positioning the client on the side with the knees flexed d) Administering an analgesic once per shift, as ordered, to prevent drug addiction

C) Postitioning the client on the side with the knees flexed The nurse should place the client with acute pancreatitis in a side-lying position with knees flexed; this position promotes comfort by decreasing pressure on the abdominal muscles. The nurse should administer an analgesic, as needed and ordered, before pain becomes severe, rather than once each shift. Because the client needs a quiet, restful environment during the acute disease stage, the nurse should discourage frequent visits from family and friends. Frequent oral feedings are contraindicated during the acute stage to allow the pancreas to rest.

A client is being treated for prolonged diarrhea. Which of the following foods should the nurse encourage the client to consume? a) Protein-rich foods b) High-fiber foods c) Potassium-rich foods d) High-fat foods

C) Potassium rich foods The nurse should encourage the client with diarrhea to consume potassium-rich foods. Excessive diarrhea causes severe loss of potassium. The nurse should also instruct the client to avoid high-fiber or fatty foods because these foods stimulate gastrointestinal motility. The intake of protein foods may or may not be appropriate depending on the client's status.

The nurse observes dry mucous membranes in a client who is receiving tube feedings after an oral surgery. The client also complains of unpleasant tastes and odors. Which of the following measures should be included in the client's plan of care? a) Ensure adequate hydration with additional water. b) Keep the feeding formula refrigerated. c) Provide frequent mouth care. d) Flush the tube with water before adding the feedings.

C) Provide frequent mouth care Frequent mouth care helps to relieve the discomfort from dryness and unpleasant odors and tastes. It can be done with the help of ice chips and analgesic throat lozenges, gargles, or sprays. Adequate hydration is essential. If urine output is less than less than 500 mL/day, formula and additional water can be given as ordered. Keeping the feeding formula refrigerated and unopened until it is ready for use and flushing the tube with water before adding feedings are measures to protect the client from infections.

The nurse is caring for a client with cirrhosis. Which assessment findings indicate that the client has deficient vitamin K absorption caused by this hepatic disease? a) Dyspnea and fatigue b) Ascites and orthopnea c) Purpura and petechiae d) Gynecomastia and testicular atrophy

C) Purpura and petechiae A hepatic disorder, such as cirrhosis, may disrupt the liver's normal use of vitamin K to produce prothrombin (a clotting factor). Consequently, the nurse should monitor the client for signs of bleeding, including purpura and petechiae. Dyspnea and fatigue suggest anemia. Ascites and orthopnea are unrelated to vitamin K absorption. Gynecomastia and testicular atrophy result from decreased estrogen metabolism by the diseased liver.

Following administration of a dose of metoclopramide (Reglan) to the patient, the nurse determines that the medication has been effective when which of the following is noted? A) Decreased blood pressure B) Absence of muscle tremors C) Relief of nausea and vomiting D) No further episodes of diarrhea

C) Relief of nausea and vomiting Metoclopramide is classified as a prokinetic and antiemetic medication. If it is effective, the patient's nausea and vomiting should resolve.

A client is admitted to the health care facility with abdominal pain, a low-grade fever, abdominal distention, and weight loss. The physician diagnoses acute pancreatitis. What is the primary goal of nursing care for this client? a) Maintaining adequate nutritional status b) Preventing fluid volume overload c) Relieving abdominal pain d) Teaching about the disease and its treatment

C) Relieving abdominal pain The predominant clinical feature of acute pancreatitis is abdominal pain, which usually reaches peak intensity several hours after onset of the illness. Therefore, relieving abdominal pain is the nurse's primary goal. Because acute pancreatitis causes nausea and vomiting, the nurse should try to prevent fluid volume deficit, not overload. The nurse can't help the client achieve adequate nutrition or understand the disease and its treatment until the client is comfortable and no longer in pain.

A nurse is teaching a client with malabsorption syndrome about the disorder and its treatment. The client asks which part of the GI tract absorbs food. The nurse tells the client that products of digestion are absorbed mainly in the: a) rectum. b) stomach. c) small intestine. d) large intestine.

C) Small intestine The small intestine absorbs products of digestion, completes food digestion, and secretes hormones that help control the secretion of bile, pancreatic juice, and intestinal secretions. The stomach stores, mixes, and liquefies the food bolus into chyme and controls food passage into the duodenum; it doesn't absorb products of digestion. Although the large intestine completes the absorption of water, chloride, and sodium, it plays no part in absorbing food. The rectum is the portion of the large intestine that forms and expels feces from the body; its functions don't include absorption.

After teaching a group of students about the various organs of the upper gastrointestinal tract and possible disorders, the instructor determines that the teaching was successful when the students identify which of the following structures as possibly being affected? a) Large intestine b) Ileum c) Stomach d) Liver

C) Stomach The upper gastrointestinal (GI) tract begins at the mouth and ends at the jejunum. Therefore, the stomach would be a component of the upper GI tract. The lower GI tract begins at the ileum and ends at the anus. The liver is considered an accessory structure.

A client has noticed increased incidence of constipation since he broke his ankle and cannot complete his daily three-mile walk. As his home care nurse, you complete your assessment and discuss the potential causes. During your client education session, what do you explain as the mechanical cause of his constipation? a) No known cause b) Ingesting excessive fiber c) Stool remaining in the large intestine too long. d) Drinking excessive water

C) Stool remaining in the large intestine too long Whenever stool remains stationary in the large intestine, moisture continues to be absorbed from the residue. Consequently, retention of stool, for any number of reasons, causes stool to become dry and hard.

The family of a patient newly diagnosed with hepatitis A asks the nurse what they can do to prevent becoming ill themselves. Which of the following responses by the nurse is most appropriate? A) "The hepatitis vaccine will provide immunity from this exposure and future exposures." B) "I am afraid there is nothing you can do since the patient was infectious before admission." C) "You will need to be tested first to make sure you don't have the virus before we can treat you." D) "An injection of immunoglobulin will need to be given to prevent or minimize the effects from this exposure."

D) An injection of immunoglobulin will need to be given to prevent or minimize the effects from this exposure Immunoglobulin provides temporary (1-2 months) passive immunity and is effective for preventing hepatitis A if given within 2 weeks after exposure. It may not prevent infection in all persons, but it will at least modify the illness to a subclinical infection. The hepatitis vaccine is only used for preexposure prophylaxis.

Which of the following is an appropriate nursing action to promote regular bowel habits? A) Encourage the patient to avoid moving his bowels until a certain time of day B) Encourage the patient to avoid excess fluid intake and too much fiber C) Avoid strenuous exercise to limit stress on the abdominal muscles and impair peristalsis D) Assisting the patient to a normal position as possible to defecate

D) Assisting the patient to a normal position as possible to defecate Sitting upright on a toilet or commode promotes defecation. If the patient must use a bedpan, raise the head of the bed 30 to 45°. Patient should be encouraged to move their bowels at their usual time of the day. However, the patient should not be encouraged to put off defecation if the urge arises before or after their usual time. Patient should be encouraged to consume 2000 to 3000 mL of fluid, preferably water, and increase fiber, to promote regular defecation. Regular exercise improves gastrointestinal activities and aids in defecation

Which of the following would a nurse expect to assess in a client with peritonitis? a) Decreased pulse rate b) Deep slow respirations c) Hyperactive bowel sounds d) Board-like abdomen

D) Board-like abdomen The client with peritonitis would typically exhibit a rigid, board-like abdomen, with absent bowel sounds, elevated pulse rate, and rapid, shallow respirations.

To ensure patency of central venous line ports, diluted heparin flushes are used in which of the following situations? a) Before drawing blood b) With continuous infusions c) When the line is discontinued d) Daily when not in use

D) Daily when not in use Daily instillation of dilute heparin flush when a port is not in use will maintain the port. Continuous infusion maintains the patency of each port. Heparin flushes are used after each intermittent infusion. Heparin flushes are used after blood drawing in order to prevent clotting of blood within the port. Heparin flush of ports is not necessary if a line is to be discontinued.

The health care provider orders lactulose for a patient with hepatic encephalopathy. The nurse will monitor for effectiveness of this medication for this patient by assessing which of the following? A) Relief of constipation B) Relief of abdominal pain C) Decreased liver enzymes D) Decreased ammonia levels

D) Decreased ammonia levels Hepatic encephalopathy is a complication of liver disease and is associated with elevated serum ammonia levels. Lactulose traps ammonia in the intestinal tract. Its laxative effect then expels the ammonia from the colon, resulting in decreased serum ammonia levels and correction of hepatic encephalopathy.

The most common symptom of esophageal disease is a) nausea. b) odynophagia. c) vomiting. d) dysphagia.

D) Dysphagia This symptom may vary from an uncomfortable feeling that a bolus of food is caught in the upper esophagus to acute pain on swallowing. Nausea is the most common symptom of gastrointestinal problems in general. Vomiting is a nonspecific symptom that may have a variety of causes. Odynophagia refers specifically to acute pain on swallowing.

As the nurse prepares to assist Mrs. P with her newly created Ileostomy, She is aware of which of the following? A) An appliance will not be required on the continual basis B) The size of the stoma stabilizes within two weeks C) Irrigation is necessary for regulation D) Fecal drainage will be liquid

D) Fecal drainage will be liquid And appliance is usually required on a continual basis because the fecal drainage is liquid. Stomas size usually stabilizes within 4 to 6 weeks, and Ileostomy Irrigation is not necessary because fecal matter is liquid

A client has a gastrointestinal tube that enters the stomach through a surgically created opening in the abdominal wall. The nurse documents this as which of the following? a) Jejunostomy tube b) Nasogastric tube c) Orogastric tube d) Gastrostomy tube

D) Gastrostomy tube A gastrostomy tube enters the stomach through a surgically created opening into the abdominal wall. A jejunostomy tube enters jejunum or small intestine through a surgically created opening into the abdominal wall. A nasogastric tube passes through the nose into the stomach via the esophagus. An orogastric tube passes through the mouth into the stomach.

A nurse is applying an ostomy appliance to the ileostomy of a client with ulcerative colitis. Which action is appropriate? a) Maintaining wrinkles in the faceplate so it doesn't irritate the skin b) Scrubbing fecal material from the skin surrounding the stoma c) Cutting the faceplate opening no more than 2? larger than the stoma d) Gently washing the area surrounding the stoma using a facecloth and mild soap

D) Gently washing the area surrounding the stoma using a facecloth and mild soap For a client with an ostomy, maintaining skin integrity is a priority. The nurse should gently wash the area surrounding the stoma using a facecloth and mild soap. Scrubbing the area around the stoma can damage the skin and cause bleeding. The faceplate opening should be no more than 1/8? to 1/6? larger than the stoma. This size protects the skin from exposure to irritating fecal material. The nurse can create an adequate seal and prevent leakage of fecal material from under the faceplate by applying a thin layer of skin barrier and smoothing out wrinkles in the faceplate. Eliminating wrinkles in the faceplate also protects the skin surrounding the stoma from pressure.

The nurse is caring for a postoperative patient with a colostomy. The nurse is preparing to administer a dose of famotidine (Pepcid) when the patient asks why the medication was ordered since the patient does not have a history of heartburn or gastroesophageal reflux disease (GERD). Which of the following would be the most appropriate response by the nurse? A) "This will prevent air from accumulating in the stomach, causing gas pains." B) "This will prevent the heartburn that occurs as a side effect of general anesthesia." C) "The stress of surgery is likely to cause stomach bleeding if you do not receive it." D) "This will reduce the amount of HCl in the stomach until the nasogastric tube is removed, and you can eat a regular diet again."

D) This will reduce the amount of HCl in the stomach until the nasogastric tube is removed, and you can eat a regular diet again Famotidine is an H2-receptor antagonist that inhibits gastric HCl secretion and thus minimizes damage to gastric mucosa while the patient is not eating a regular diet after surgery.

Why are antacids administered regularly, rather than as needed, in peptic ulcer disease? a) To increase pepsin activity b) To maintain a regular bowel pattern c) To promote client compliance d) To keep gastric pH at 3.0 to 3.5

D) To keep gastric pH at 3.0 to 3.5 To maintain a gastric pH of 3.0 to 3.5 throughout each 24-hour period, regular (not as needed) doses of an antacid are needed to treat peptic ulcer disease. Frequent administration of an antacid tends to decrease client compliance rather than promote it. Antacids don't regulate bowel patterns, and they decrease pepsin activity.

A client with cholelithiasis has a gallstone lodged in the common bile duct. When assessing this client, the nurse expects to note: a) black, tarry stools. b) circumoral pallor. c) light amber urine. d) yellow sclerae.

D) Yellow sclerae Yellow sclerae are an early sign of jaundice, which occurs when the common bile duct is obstructed. Urine normally is light amber. Circumoral pallor and black, tarry stools don't occur in common bile duct obstruction; they are signs of hypoxia and GI bleeding, respectively.

A nurse is assisting with a percutaneous liver biopsy. Place the steps involved in care in the correct sequence from first to last. Ensure that the biopsy equipment is assembled and in order. Help the client assume a supine position. Make sure that the specimen container is labeled and delivered to the laboratory. While the physician inserts the needle, instruct the client to take a deep breath and hold it to keep the liver as near to the abdominal wall as possible. Place a rolled towel beneath the client's right lower ribs.

Ensure that the biopsy equipment is assembled and in order. Help the client assume a supine position. Place a rolled towel beneath the client's right lower ribs. While the physician inserts the needle, instruct the client to take a deep breath and hold it to keep the liver as near to the abdominal wall as possible. Make sure that the specimen container is labeled and delivered to the laboratory. When assisting with a percutaneous liver biopsy, the nurse ensures that the biopsy equipment is assembled and in order. He or she helps the client assume a supine position with a rolled towel beneath the right lower ribs. Before the physician inserts the needle, the nurse instructs the client to take a deep breath and hold it to keep the liver as near to the abdominal wall as possible. After specimen cells are obtained, they are placed in a preservative. The nurse makes sure that the specimen container is labeled and delivered to the laboratory.

A few hours after eating hot and spicy chicken wings, a client presents with lower chest pain. He wonders if he is having a heart attack. How should the nurse proceed first? a) Further investigate the initial complaint. b) Explain that fatty foods can mimic chest pain. c) Administer an over-the-counter antacid tablet. d) Call for an immediate electrocardiogram.

Further investigate the initial complaint. Explanation: While fatty foods can cause discomfort similar to chest pain, the nurse must fully assess all the client's symptoms. Investigation of chief complaint begins with a complete history. The underlying cause of pain influences the characteristics, duration, pattern, location, and distribution of pain.

If you have a client walking in the hallway and they think their wound may have opened where should you assess them?

Get them back to bed. DO NOT assess them in the hallway.

What should diet look like for a person with CROHN's disease?

High protein (tissue building) and increased calorie.

What is the most appropriate diet for a patient with ulcerative colitis?

High protein, low residue.

What diet changes would you recommend to decrease GERD?

Patient will decrease intake of fats; patient will eat small, frequent meals.

A client undergoing a diagnostic examination for gastrointestinal disorder was given polyethylene glycol/electrolyte solution as a part of the test preparation. Which of the following measures should the nurse take once the solution is administered? a) Allow the client to ingest fat-free meal. b) Instruct the client to have low-residue meals. c) Provide saline gargles to the client. d) Permit the client to drink only clear liquids.

Permit the client to drink only clear liquids. Explanation: After polyethylene glycol/electrolyte solution is administered, the client should have clear liquids as this ensures watery stools, which are necessary for procedures like a barium enema. Allowing the client to ingest a fat-free meal is used in preparation for oral cholecystography. Instructing the client to have low-residue meals is a pretest procedure for barium enema. A client is offered saline gargles after esophagogastroduodenoscopy.

A client diagnosed with acute pancreatitis is being transferred to another facility. The nurse caring for the client completes the transfer summary, which includes information about the client's drinking history and other assessment findings. Which assessment findings confirm his diagnosis? a) Recent weight loss and temperature elevation b) Presence of easy bruising and bradycardia c) Adventitious breath sounds and hypertension d) Presence of blood in the client's stool and recent hypertension

Recent weight loss and temperature elevation Assessment findings associated with pancreatitis include recent weight loss and temperature elevation. Inflammation of the pancreas causes a response that elevates temperature and leads to abdominal pain that typically occurs with eating. Nausea and vomiting may occur as a result of pancreatic tissue damage that's caused by the activation of pancreatic enzymes. The client may experience weight loss because of the lost desire to eat. Blood in stools and recent hypertension aren't associated with pancreatitis; fatty diarrhea and hypotension are usually present. Presence of easy bruising and bradycardia aren't found with pancreatitis; the client typically experiences tachycardia, not bradycardia. Adventitious breath sounds and hypertension aren't associated with pancreatitis.

A client is scheduled for bowel resection with anastomosis involving the large intestine. Because of the surgical site, the nurse formulates the nursing diagnosis of Risk for infection. To complete the nursing diagnosis statement, the nurse should add which "related-to" phrase? a) Related to the presence of bacteria at the surgical site b) Related to malnutrition secondary to bowel resection with anastomosis c) Related to major surgery required by bowel resection d) Related to the presence of a nasogastric (NG) tube postoperatively

Related to the presence of bacteria at the surgical site Explanation: The nurse should add "Related to the presence of bacteria at the surgical site" to the diagnosis of Risk for infection. The large intestine normally contains bacteria because its alkaline environment permits growth of organisms that putrefy and break down remaining proteins and indigestible residue. These organisms include Escherichia coli, Aerobacter aerogenes, Clostridium perfringens, and Lactobacillus. Although bowel resection with anastomosis is considered major surgery, it poses no greater risk of infection than any other type of major surgery. Malnutrition seldom follows bowel resection with anastomosis because nutritional absorption (except for some water, sodium, and chloride) is completed in the small intestine. An NG tube is placed through a natural opening, not a wound, and therefore doesn't increase the client's risk of infection.

How should you teach your patient to irrigate their colostomy?

Remove soiled colostomy appliance, apply irrigation sleeve, place 750-1000 cc lukewarm water in the irrigation container and hang 18-20 inches above stoma, lub catheter with water soluble, insert using a ROTATING MOTION, no more than 3 inches, open clamp and let water flow for 5-10 mins, hold for 10 sec than remove, wait 15 mins for return, and observe emptying. DON'T USE FORCE WHEN INSERTING CATHETER BC COULD INJURE BOWEL.

A nurse is providing postprocedure instructions for a client who had an esophagogastroduodenoscopy. The nurse should perform which action? a) Tell the client there aren't specific instructions for after the procedure. b) Tell the client to call back in the morning so she can give him instructions over the phone. c) Give instructions to the client immediately before discharge. d) Review the instructions with the person accompanying the client home.

Review the instructions with the person accompanying the client home. Explanation: A client who undergoes esophagogastroduodenoscopy receives sedation during the procedure, and his memory becomes impaired. Clients tend not to remember instructions provided after the procedure. The nurse's best course of action is to give the instructions to the person who is accompanying the client home. It isn't appropriate for the nurse to tell the client to call back in the morning for instructions. The client needs to be aware at discharge of potential complications and signs and symptoms to report to the physician.

What does pain relief after having presented with appendicitis like pain mean? Intervention?

Rupture. Place in high fowlers bc stool in lower abdomen.

Which populations should limit their intake of sodium, control their protein intake, limit their intake of potassium-rich foods, and avoid foods containing gluten.

Some health conditions require modifi cation of dietary intake. The following are the most common diets: ■ Calorie-restricted. For clients requiring weight reduction ■ Sodium-restricted. For clients with blood pressure or fluid balance problems ■ Fat-restricted. For clients with elevated cholesterol or triglyceride levels; may also be ordered for general weight loss. ■ Diabetic. To manage calories and carbohydrate intake for clients with diabetes mellitus ■ Renal diet. To manage electrolytes and fluid for clients with renal and liver disease. Controlled amounts of protein, sodium, phosphorus, calcium, potassium, and fluids; may also need modification in fiber, cholesterol, and fat based on individual requirements. Most patients on dialysis need to restrict fluids. ■ Protein-controlled diet. To manage liver and kidney disease ■ Antigen-avoidance diets. For clients allergic to or intolerant of certain foods, such as a gluten-free diet for clients with celiac disease ■ Calorie-protein push. Used when there is a need to heal wounds, maintain or increase weight, or promote growth. If the person cannot consume enough kcal by adding fats and proteins to his regular diet, high-calorie, high-protein supplements may be used.

What is characteristic of an ileostomy?

Stool is always liquid, never irrigate, always wear a bag, watch for skin breakdown, empty q3hrs, at risk for fluid and electrolyte imbalance.

What is characteristic of a transverse loop colostomy?

Stool is normal, irrigate (give enema same time every day), after training A BAG DOESN'T HAVE TO BE WORN.

What is characteristic of an end colostomy?

Stool is normal, irrigate (give enema same time everyday), PERMANENT, (after training A BAG DOESN'T HAVE TO BE WORN).

The nurse is preparing to examine the abdomen of a client complaining of a change in his bowel pattern. The nurse would place the client in which position? a) Lithotomy b) Knee-chest c) Supine with knees flexed d) Left Sim's lateral

Supine with knees flexed Explanation: When examining the abdomen, the client lies supine with his knees flexed. This position assists in relaxing the abdominal muscles. The lithotomy position commonly is used for a female pelvic examination and to examine the rectum. The knee-chest position can be used for a variety of examinations, most commonly the anus and rectum. The left Sim's lateral position may be used to assess the rectum or vagina and to administer an enema.

Which of the following should the nurse advise a client due to undergo a diagnostic test for a disorder of the GI system, to ensure that the client experiences none or minimal discomfort? a) The client should not expel gas and test fluids from the bowel when he or she experiences the urge during the procedure b) The client should take a sedative before the procedure to avoid the possibility of experiencing any discomfort c) The client should lie down in a supine position for at least three hours before the test to reduce any discomfort during the test d) The client should inform the test personnel if he or she experiences pressure or cramping during the instillation of test fluids

The client should inform the test personnel if he or she experiences pressure or cramping during the instillation of test fluids Explanation: To ensure that a client who is to undergo a diagnostic test for a disorder of the gastrointestinal system experiences none or minimal discomfort during the test, the client should be instructed to inform the test personnel if the client experiences pressure or cramping during the instillation of test fluids. The test personnel can slow the instillation or take other measures to relieve discomfort.

Which patient teaching component is important for the nurse to communicate regarding pain management prior to or during diagnostic testing for a disorder of the GI system? a) The patient should inform the test personnel if he or she experiences pressure or cramping during the instillation of test fluids. b) The patient should not expel gas and test fluids from the bowel when he or she experiences the urge during the procedure. c) The patient should take a sedative before the procedure to avoid the possibility of experiencing any discomfort. d) The patient should lie down in a supine position for at least 3 hours before the test to reduce any discomfort during the test.

The patient should inform the test personnel if he or she experiences pressure or cramping during the instillation of test fluids. Explanation: To ensure that a patient who is to undergo a diagnostic test for a disorder of the gastrointestinal system experiences no or minimal discomfort during the test, the patient should be instructed to inform the test personnel if he or she experiences pressure or cramping during the instillation of test fluids. The test personnel can slow the instillation or take other measures to relieve discomfort. The patient should also be advised to expel gas and test fluids from the bowel when he or she experiences the urge. Ignoring the urge to expel the bowel contents increases pain and discomfort. The patient should be advised not to take any sedative or analgesic before the test, unless prescribed. Lying down in a supine position is not known to have any consequence on the level of discomfort experienced by a patient during a diagnostic test for a GI disorder.

Patient has C. diff. How does the nurse prevent it from spreading?

Wash hands with soap & water; wear protective clothing. (Note: alcohol gel will NOT kill C. diff.)

What are the S&S of dumping syndrome?

Weakness, Dizziness, Diaphoresis, epigastric fullness, tachycardia, and abdominal cramping occurring within 15 mins after eating.

What is characteristic of a gastric ulcer?

Weight loss, burn left epigastric area, food frequently aggravates pain, decreased pain at night.


Conjuntos de estudio relacionados

NU142- Chapter 1: Health Care Delivery and Evidence-Based Nursing Practice

View Set

Chapter 1 Introduction to Statistics

View Set

Ch. 10-12 Urinary System, Spleen, Retroperitoneum

View Set

2.1: Deterministic Finite Automata

View Set

Geology Chapter 14: Energy Resources - Fossil Fuels

View Set

Intermediate Micro Econ Test 3 Study Guide

View Set